You are on page 1of 288

i

1. Um pouco de clculo
1.1 Introduo aos vetores................................................................................1
1.2 Introduo s derivadas..............................................................................9
1.3 Integrao ...............................................................................................15
1.4 Interpretao cinemtica das derivadas e integrais...................................19
Exerccios................................................................................................21

2. Movimento unidimensional
2.1 Introduo..................................................................................................25
2.2 Classificao dos movimentos unidimensionais.......................................30
2.3 Determinao de x(t) a partir de v(t) e de v(t) a partir de a(t)...................30
2.4 Acelerao constante.................................................................................32
Exerccios................................................................................................34

3. Movimentos bi e tridimensional
3.1 Introduo..................................................................................................35
3.2 Decomposio de movimentos..................................................................37
3.3 O movimento acelerado.............................................................................38
3.4 Movimentos planos descritos por coordenadas polares............................43
Exerccios................................................................................................45

4. As leis de Newton
4.1 Introduo..................................................................................................49
4.2 Referenciais...............................................................................................53
4.3 Aplicaes das leis de Newton..................................................................54
4.4 Movimento circular...................................................................................63
4.5 Fora retardada proporcional velocidade...............................................67
4.6 Foras observadas na natureza..................................................................69
4.7 Foras inerciais..........................................................................................75
Exerccios................................................................................................79

ndice

ii
5. Trabalho e energia
5.1 Trabalho e energia cintica.......................................................................85
5.2 Potncia.................................................................................................... 90
5.3 Energia potencial.......................................................................................90
5.4 Foras conservativas..................................................................................92
5.5 Determinao da fora a partir da energia potencial.................................94
5.6 Foras dissipativas.....................................................................................95
5.7 Conservao de energia.............................................................................96
5.8 Corpo so sob a ao de um potencial arbitrrio.....................................100
Exerccios..............................................................................................101

6. Sistema de partculas. Conservao de momentum
6.1 Centro de massa ......................................................................................107
6.2 Movimento do centro de massa...............................................................109
6.3 Sistemas onde a massa varia....................................................................112
Exerccios..............................................................................................116

7. Colises
7.1 Impulso....................................................................................................119
7.2 Transporte de momentum para uma superfcie. Presso de um gs........121
7.3 Coliso e conservao de momentum.....................................................123
Exerccios..............................................................................................127

8. Dinmica do corpo rgido
8.1 Introduo ...............................................................................................131
8.2 Rotao em torno de um eixo fixo...........................................................131
8.3 Energia rotacional e momento de inrcia................................................134
8.4 Dinmica da rotao em torno de um eixo fixo.......................................142
8.5 Equilbrio esttico de um corpo rgido....................................................145
8.6 Acelerao constante...............................................................................147
8.7 Momentum angular..................................................................................152
8.8 Torque e momentum angular de um sistema de partculas.....................154
8.9 Relao trabalho-energia rotacional........................................................158
8.10 Conservao do momentum angular.....................................................159
8.11 Combinao de translao e rotao.....................................................162
Exerccios..............................................................................................167

9. Oscilaes
9.1 O movimento harmnico simples............................................................175










iii
9.2 O sistema massa-mola.............................................................................178
9.3 O sistema massa-mola com gravidade.....................................................181
9.4 O pndulo matemtico.............................................................................182
9.5 O pndulo fsico.......................................................................................184
9.6 Oscilao de dois corpos.........................................................................185
9.7 O sistema mola-cilindro...........................................................................186
9.8 Oscilaes amortecidas............................................................................187
9.9 Oscilaes foradas.................................................................................188
Exerccios..............................................................................................192

10. Movimento ondulatrio
10.1 Introduo..............................................................................................195
10.2 Propagao de pulsos numa corda.........................................................197
10.3 Ondas sonoras........................................................................................198
10.4 Ondas harmnicas..................................................................................201
10.5 Efeito Doppler.......................................................................................202
10.6 Ondas estacionrias...............................................................................205
10.7 Funes de onda no caso estacionrio...................................................209
10.8 Interferncia...........................................................................................210
Exerccios..............................................................................................212

11. Gravitao
11.1 Introduo..............................................................................................215
Exerccios..............................................................................................221

12. Mecnica dos fluidos
12.1 Introduo..............................................................................................225
12.2 Hidrosttica............................................................................................226
12.3 Princpio de Arquimedes.......................................................................229
12.4 Dinmica dos fludos.............................................................................231
12.5 Teorema de Bernouilli...........................................................................233
12.6 Viscosidade............................................................................................238
Exerccios..............................................................................................241

13. Termologia e termodinmica
13.1 Introduo..............................................................................................245
13.2 Medida da temperatura..........................................................................247
13.3 Equao de estado.................................................................................249

iv
13.4 Interpretao microscpica da temperatura...........................................250
13.5 Dilatao trmica...................................................................................252
13.6 Calor e trabalho.....................................................................................255
13.7 Transmisso de calor.............................................................................257
Exerccios..............................................................................................262

14. Termodinmica do gs ideal
14.1 Introduo..............................................................................................265
14.2 Capacidade trmica...............................................................................268
14.3 Tipos de expanses................................................................................268
14.4 Mtodo de Rchhardt para determinao de .....................................270
Exerccios..............................................................................................272


S. C. Zilio e V. S. Bagnato Mecnica, calor e ondas







Um pouco de clculo 1








1.1 Introduo aos vetores
Existem grandezas fsicas que podem ser especificadas fornecendo-se
apenas um nmero. Assim, por exemplo, quando dizemos que a temperatura
de uma sala de 20
0
C temos a informao completa, no sendo necessrio
nenhum dado adicional. Grandezas deste tipo so conhecidas como escalares.
Por outro lado, se estivermos discutindo o deslocamento de um corpo,
necessrio indicar a distncia percorrida entre dois pontos, a direo e o
sentido do deslocamento. A grandeza que descreve este movimento
denominada de vetor e ser o objeto de estudo desta seo. Existem ainda
grandezas chamadas tensores que necessitam de um nmero maior de
informaes, em geral dadas na forma de matrizes, que fogem abrangncia
deste texto.
Geometricamente, os vetores so representados por uma seta, cujo
comprimento chamado de mdulo (escolhendo-se uma determinada escala).
A direo e o sentido da seta fornecem a direo e sentido do vetor.
Usualmente, ele representado por uma letra em negrito (a, AB, etc.) ou com
uma seta sobre a letra ( a
r
,
AB

, etc.). Por outro lado, o mdulo do vetor


representado apenas por uma letra ou com o vetor colocado entre barras (a,
a
r
, (
AB

(, etc.)
Consideremos uma partcula deslocando-se de A para B. Este
deslocamento representado por uma seta indo de A at B, como a mostrada
na Fig. 1.1(a). O caminho efetivamente seguido pela partcula pode no
coincidir com o seu deslocamento (vetor), conforme ilustra a Fig. 1.1(b). Se
considerarmos pontos intermedirios (P), tais como o mostrado na Fig. 1.1(c),
1
UM POUCO DE
CLCULO

S. C. Zilio e V. S. Bagnato Mecnica, calor e ondas






2 Um pouco de clculo


poderemos eventualmente mapear o trajeto, porm a soma resultante ser
sempre o vetor
AB

, caracterizado pelo seu mdulo (comprimento), direo e


sentido. As grandezas vetoriais combinam-se segundo determinadas regras.
Assim, no deslocamento da Fig. 1.1 definimos a operao soma de vetores,
AB PB AP

= + , que veremos com mais detalhes a seguir.






Fig. 1.1 - (a) Vetor descrevendo o deslocamento de uma partcula entre os pontos A e
B, (b) trajetria real da partcula e (c) soma de deslocamentos.
Consideremos os vetores a
r
e b
r
mostrados na Fig. 1.2. O resultado da
adio destes dois vetores a resultante r
r
, denotada por b a r
r
r r
+ = . O
procedimento empregado para efetuar a adio geomtrica de vetores pode ser
intudo a partir da Fig. 1.1 e o seguinte: traa-se (em escala) o vetor a
r
e em
seguida o vetor b
r
com a origem na extremidade de a
r
. Une-se a extremidade
final de b
r
com a origem de a
r
e assim temos o vetor soma r
r
, como ilustrado
na Fig. 1.2.




Fig. 1.2 - Adio geomtrica dos vetores a
r
e b
r
.
Usando este procedimento geomtrico para a adio de vetores, vemos
que esta satisfaz as propriedades comutativa: a b b a
r
r r
r
+ = + e associativa:
) c b ( a c ) b a (
r
r
r r
r
r
+ + = + + , como indicado na Fig. 1.3.
A
B B
A
B
A
P
(a) (b) (c)
a
r

b
r

r
r



S. C. Zilio e V. S. Bagnato Mecnica, calor e ondas







Um pouco de clculo 3


A subtrao de vetores facilmente introduzida definindo-se o
negativo de um vetor como sendo o vetor com sentido oposto ao original.
Assim, ) b ( a b a
r
r
r
r
+ = , como ilustrado na Fig. 1.4. Note que tanto a adio
como a subtrao podem ser representadas simultaneamente pela construo
do paralelogramo representado na Fig. 1.5.






(a) (b)
Fig. 1.3 - Propriedades (a) comutativa e (b) associativa.




Fig. 1.4 - Subtrao geomtrica dos vetores a
r
e b
r
.





Fig. 1.5 - Regra do paralelogramo para a adio e subtrao geomtrica dos vetores
a
r
e b
r
.
A adio geomtrica de vetores tridimensionais muito mais difcil e para
evit-la costuma-se utilizar o mtodo analtico, que consiste na decomposio
espacial dos vetores e na manipulao individual de seus componentes. A
decomposio de um vetor s pode ser efetuada com relao a um sistema de
a
r

b
r


c
r

b a
r
r
+
c b a
r
r
r
+ +
c b
r
r
+
a
r

b
r

r
r

a
r

b
r


a
r

b
r

b
r

a
r

b a
r
r

b
r

a
r
b a
r
r

a
r


b
r

a
r

b
r


b a
r
r
+


S. C. Zilio e V. S. Bagnato Mecnica, calor e ondas






4 Um pouco de clculo


coordenadas de orientao conhecida no espao. Considere a decomposio de
um vetor no plano, conforme mostra a Fig. 1.6, onde o ngulo entre a
r
e
o semi-eixo positivo x. Dependendo do ngulo , as componentes podem ser
positivas ou negativas. Por definio, este ngulo aumenta quando o vetor
roda no sentido anti-horrio. O conhecimento dos componentes de um vetor
suficiente para especific-lo completamente, alm de possibilitar a
manipulao matemtica simultnea de vrios vetores. De acordo com a Fig.
1.6 temos a
x
= a cos e a
y
= a sen, de onde sai que:
2
y
2
x
a a a a + = =
r

tg = a
y
/a
x








Fig. 1.6 - Decomposio do vetor a
r
num sistema de coordenadas cartesianas.
Muitas vezes conveniente a introduo de um vetor de mdulo
unitrio, chamado versor, na direo de um determinado vetor, que pode ento
ser escrito como
a
e a a =
r
. Assim separamos o mdulo do vetor (a) de sua
direo e sentido (
a
e ). Da mesma forma, conveniente traar versores
paralelos aos eixos do sistema de coordenadas escolhido, como mostra a Fig.
1.7. Normalmente, no sistema de coordenadas cartesianas eles so chamados
de i

, j

e k

.
Costumamos dizer que estes versores formam uma base completa
porque qualquer vetor pode ser expresso como combinao linear deles, da
forma:
y

a
y
a
x


x

a
r


S. C. Zilio e V. S. Bagnato Mecnica, calor e ondas







Um pouco de clculo 5


k

a j

a i

a a
z y x
+ + =
r









Fig. 1.7 - Versores no sistema de coordenadas cartesianas.
onde k

a e j

a , i

a
z y x
so denominadas de componentes vetoriais do vetor a
r
.
Note que se estivermos tratando com vetores contidos no plano xy, temos a
z
=
0. A soma analtica de vetores pode ser efetuada da forma:
( ) ( ) k

b j

b i

b k

a j

a i

a b a r
z y x z y x
+ + + + + = + =
r
r r

( ) ( ) ( ) k

r j

r i

r k

b a j

b a i

b a
z y x z z y y x x
+ + = + + + + + =
Assim, r
x
= a
x
+ b
x
, r
y
= a
y
+ b
y
, r
z
= a
z
+ b
z
. Logo: O vetor resultante tem como
componentes a soma das respectivas componentes dos vetores individuais.
Como exemplo, considere 3 vetores coplanares dados por: j

1 i

2 a =
r
,
j

2 i

3 b + =
r
e i

1.5 c =
r
. As componentes do vetor resultante so: r
x
= 2 + 3 -
1.5 = 3.5 e r
y
= -1 + 2 + 0 = 1, de modo que j

1 i

5 . 3 r + =
r
. O ngulo pode
ser encontrado de acordo com:
tg = r
y
/r
x
= 1/3.5

=

0.286 = 15.9
0

e o mdulo :
( ) 3.64 1 3.5 r r
2
= + = =
r

Uma operao que veremos aparecer com freqncia nos prximos
captulos a multiplicao envolvendo vetores, que pode ser de trs tipos:
k


x
y
z

S. C. Zilio e V. S. Bagnato Mecnica, calor e ondas






6 Um pouco de clculo


a) Multiplicao de um vetor por um escalar - resulta num outro vetor paralelo
ao primeiro, porm com o mdulo multiplicado por uma constante. Se esta
constante for negativa existe a inverso do sentido do vetor.
b) Produto escalar - o produto escalar entre a
r
e b
r
resulta num nmero (e no
num vetor) que definido como cos ab b . a =
r
r
, onde o ngulo entre eles.
Geometricamente, temos o produto do mdulo de um vetor pela projeo do
outro sobre si. Este tipo de produto aparece no clculo do trabalho mecnico,
potncia de uma fora, etc.





Fig. 1.8 - Produto escalar entre dois vetores a
r
e b
r
.
c) Produto vetorial representado por b a c
r
r r
= . O vetor resultante tem o
mdulo dado por c = ab sen, e direo perpendicular ao plano que contm a
r

e b
r
. Novamente, o ngulo entre a
r
e b
r
. O sentido de c
r
pode ser
determinado pela regra da mo direita, ilustrada na Fig. 1.9. Usa-se a seguinte
receita: Empurre com as pontas dos dedos o vetor a
r
no sentido de superp-
lo ao vetor b
r
. O polegar indicar o sentido do vetor c
r
.







Fig. 1.9 - Regra da mo direita para a realizao do produto vetorial.

a
r
b
r
c
r
b
r
a
r

S. C. Zilio e V. S. Bagnato Mecnica, calor e ondas







Um pouco de clculo 7


Ao contrrio do produto escalar, o produto vetorial no comutativo,
isto , ele muda de sinal ao mudarmos a ordem dos vetores, isto ,
a b b a
r
r r
r
= . Este fato pode ser comprovado pela regra da mo direita.
Algumas propriedades interessantes dos produtos escalar e vetorial so:
1. distributiva (escalar): c . a b . a ) c b .( a
r r
r
r r
r
r
+ = +
2. distributiva (vetorial): c a b a ) c b ( x a
r r
r
r r
r
r
+ = +
3. produto misto: ) b a .( c ) a c ( . b ) c b ( . a
r
r r r r
r
r
r
r
= =
4. duplo produto vetorial: c ) b . a ( b ) c . a ( ) c b ( x a
r
r
r
r
r r r
r
r
=
Para o clculo do produto vetorial, notamos que: j

= =
0 k

= = , pois o ngulo entre dois vetores iguais nulo e


i

, k

= = e j

= , como pode ser visto pela regra da mo


direita. Vejamos a seguir alguns exemplos de multiplicao vetorial.
(i) k

8 b a j

2 b e i

4 a = = =
r
r
r
r

(ii) j

- i

b e j

3 i

2 a
2
1
= + =
r
r
= b a
r
r
( ) =
|

\
|
+ j

3 i

2
2
1

k

- j

3 - i

2 - i

2
7
2
3
= + = .
Uma outra maneira de se fazer o produto vetorial pelo uso de
matrizes. Considere k

3 i

2 a + =
r
e k

2 j

b + =
r
. Podemos calcular o
vetor resultante pela co-fatora da matriz:
( ) ( ) ( ) ) k

5( k

3 2 j

1 4 i

1 6
2 1 - 1
1 - 3 2
k

b a = + + = =
r
r

Este mesmo resultado pode ser encontrado utilizando-se a propriedade
distributiva (vetorial).
A variao dos vetores um fato extremamente importante. Vamos
analisar, por exemplo, o movimento circular uniforme, esquematizado na Fig.
1.10.

S. C. Zilio e V. S. Bagnato Mecnica, calor e ondas






8 Um pouco de clculo









Fig. 1.10 - Representao do movimento circular.
Durante um intervalo de tempo t extremamente curto (infinitesimal), a
distncia percorrida s = r t. O vetor velocidade dado por:
t / s v
r r
=
e para calcul-lo tomamos, de acordo com a Fig. 1.10:
( ) ( ) j

t t sen r i

t t cos r r r s
1 2
+ + + = =
r r r

| | i

t sen t sen t cos t cos r j

t sen r i

t cos r =
| | j

t sen r i

t cos r j

t sen t cos t cos t sen r + +


Para t muito pequeno ( 0 t ) temos 1 t cos e
t t sen , e assim,
j

t cos t r i

t sen t r s + =
r
j

t cos r i

t sen r v + =
r

Desta forma, a variao temporal do vetor posio r
r
nos leva a um
vetor velocidade v
r
que tangencial rbita do movimento circular. Note que
se definirmos um vetor k

=
r
, podemos escrever
j

t cos r i

t sen r
0 t rsen t rcos
0 0
k

v + =

=
r

t
r
r
x
y
s
r
s
r
1
t
r
2

S. C. Zilio e V. S. Bagnato Mecnica, calor e ondas







Um pouco de clculo 9


Como vemos, o conhecimento de como as grandezas fsicas variam to
importante quanto o conhecimento da prpria grandeza. Como o vetor
caracterizado pelo mdulo, direo e sentido, ele apresentar variao sempre
que um destes elementos mudar. Podemos ter:
a) Variao do mdulo, como indicado na Fig. 1.11:


1 2
v - v v
r r r
=

Fig. 1.11 Variao do mdulo de um vetor .
b) Variao da direo, como no movimento circular visto anteriormente:

2 1
a a
r r
=

1 2
a a a
r r r
=

Fig. 1.12 - Variao da direo de um vetor .
Este tipo de clculo que fizemos, considerando a variao do vetor em
intervalos pequenos, extremamente til em Fsica e nos leva ao chamado
clculo infinitesimal (vlido quando 0 t ). Abordaremos este tpico a
seguir.
1.2 Introduo s derivadas
Em Fsica, a manipulao matemtica das vrias grandezas to
importante quanto o conhecimento da prpria grandeza. Nem sempre as
operaes elementares de lgebra so suficientes para tais manipulaes,
sendo necessria a introduo de novas operaes e conceitos matemticos.
Dentre estes, so de extrema importncia os de derivada e integral.
Como ilustrao, consideremos um corpo que se desloca a uma
distncia d num intervalo de tempo t. Com estes dados, o mximo que
1
v
r

2
v
r

v
r

1
a
r

2
a
r

a
r


S. C. Zilio e V. S. Bagnato Mecnica, calor e ondas






10 Um pouco de clculo


podemos fazer calcular a velocidade mdia do corpo no intervalo
mencionado. Se quisermos conhecer a velocidade instantnea do corpo num
determinado ponto de sua trajetria, deveremos analisar seu comportamento
nas vizinhanas deste ponto e to mais exata ser a resposta quanto mais
limitada for a vizinhana. comum nesta situao que descrevemos
encontrarmos divises de nmeros quase nulos e, neste caso, tais divises
devem ser feitas de uma maneira especial.
Vamos iniciar a abordagem deste assunto pelo conceito intuitivo de
limite. Consideremos a funo ( ) 1 x 4 x f
2
+ = . Queremos estudar seu
comportamento quando a varivel x assume valores cada vez mais prximos
de 1. Para isto, vamos construir a seguinte tabela:

x f(x) x f(x)
0.6
0.7
0.8
0.9
0.95
0.99
2.44
2.96
3.56
4.24
4.61
4.92
1.4
1.3
1.2
1.1
1.01
1.001
8.84
7.76
6.76
5.84
5.08
5.008

Ela mostra claramente que quando x tende a 1, f(x) tende a 5 e estar
mais prximo de 5 quanto menor for a diferena entre x e 1. Este fato
expresso matematicamente da seguinte forma:
( ) 5 x f lim
1 x
=


que quer dizer que o limite da funo f(x) quando x tende a 1 5. Outros
exemplos que podemos citar so:
1 1 x 2 lim
1 x
=



S. C. Zilio e V. S. Bagnato Mecnica, calor e ondas







Um pouco de clculo 11


=

x
1
lim
0 x

( ) 1 x / 1 1 lim
x
= +


Para funes polinomiais, isto , funes que tenham dependncia do
tipo x
n
, vale a seguinte propriedade:
( ) ( ) c f x f lim
c x
=


Existem outros limites que so um pouco mais difceis de serem
demonstrados e que so melhor discutidos nos livros de Clculo. Por exemplo
temos:
1
x
x sen
lim
0 x
= |

\
|


( ) ... 718 . 2 e x / 1 1 lim
x
x
= = +


Vamos a seguir usar o conceito de limite para introduzir a operao de
diferenciao (derivadas). Seja a funo f(x) definida num intervalo do eixo x,
no qual o ponto x
0
est contido, como mostra a Fig. 1.13. Chamaremos de
razo incremental da funo f(x) relativa ao ponto x
0
, a quantidade:
( ) ( )
0
0
x x
x f x f










Fig. 1.13 - Definio da razo incremental.
x
f(x)
x
0
x
f(x)-f(x
0
)

S. C. Zilio e V. S. Bagnato Mecnica, calor e ondas






12 Um pouco de clculo


A razo incremental da funo f(x) representa o quanto a funo
incrementada quando x variado de x
0
a x. Esta razo pode ser positiva,
negativa ou nula dependendo se a funo crescente, decrescente ou constante
no intervalo considerado. A derivada de uma funo definida como:
( ) ( )
(

=

0
0
x x 0
x x
x f x f
lim ) x ( ' f
0

tambm comum escrevermos dx / df ) x ( ' f
0
= . Fazendo x = x
0
+
, x temos:
( ) ( )
(

+
=

x
x f x x f
lim ) x ( ' f
0 0
o x 0

A derivada da funo num ponto representa a taxa de variao da
funo ao nos afastarmos deste ponto. Vamos, a seguir, obter a derivada de
algumas funes.

1) f(x) = x
2
+ 3x
( ) ( ) ( )
x
x 3 x x x 3 x x
x
) x ( f x x f
2
2

+ + +
=

+

x 3 x 2
x
x 3 x x 3 x 3 x x x 2 x
2 2 2
+ + =

+ + + +
=
Logo: ( ) ( ) 3 x 2 x 3 x 2 lim x ' f
0 x
+ = + + =


2) ( )
( ) ( )
x
x x x
x
x f x x f
x x f

+
=

+
=
( ) ( )
( ) ( ) x x x
1
x x x x
x x x
x x x
x x x
x
x x x
+ +
=
+ +
+
=
+ +
+ +

+
=

E assim,
x 2
1
x x x
1
lim ) x ( ' f
o x
=
+ +
=



S. C. Zilio e V. S. Bagnato Mecnica, calor e ondas







Um pouco de clculo 13


3)
( ) ( ) ( )
x
x cos x x cos
x
x f x x f
x cos ) x ( f

+
=

+
=
( )
( ) 2
2
x
x
2
x
sen
x sen

\
|
+ =
onde utilizamos cos(a+b) - cos(a-b) = -2 sena senb, com a = x + x/2 e b =
x/2. Desta forma temos:
( )
( )
x sen
sen
x sen lim ) x ( f
2
2
x
x
2
x
0 x
'
=
(

\
|
+ =



Geometricamente, podemos verificar que a derivada da funo f(x)
num determinado ponto x
0
representa a tangente do ngulo formado pela reta
tangente curva em x
0
com o eixo das abcissas (x). Este fato est ilustrado na
Fig. 1.14. fcil verificar quando fazemos x tender a x
0
, a reta que passa por
estes dois pontos confunde-se cada vez mais com a tangente curva no ponto
x
0
. Logo:
( )
( )
=

=

tg
x x
) x ( f x f
lim x ' f
0
0
0 x 0








Fig. 1.14 Interpretao geomtrica da derivada.
Uma vez visto o significado matemtico da derivada, passemos a
apresentao de certas regras que facilitam bastante os clculos:
1) funo constante: ( ) 0
dx
df
c x f = =
x
f(x)
f(x)
f(x
0
)
x
0
x
tangente


S. C. Zilio e V. S. Bagnato Mecnica, calor e ondas






14 Um pouco de clculo


2) funo potncia: ( )
1 n n
nx x ' f x ) x ( f

= = (regra do tombo)
3) funo soma: f(x) = u(x) + v(x) f(x) = u(x) + v(x)
Ex.: f(x) = x
4
x
3
+ 3x
2
+ 1 f(x) = 4x
3
3x
2
+ 6x
4) funo produto: f(x) = u(x). v(x) f(x) = u(x) v(x) + u(x). v(x)
Ex.: f(x) = 3x
2
(4x+1) f(x) = 6x (4x+1) + 3x
2
(4)
5) funo quociente: ( ) ) x ( v / ) x ( u x f =
( ) ( ) ( ) ( )
( )
2
x v
x ' v x u x v x ' u
) x ( ' f

=

6) funes trigonomtricas:
( ) ( ) x cos x ' f x sen x f = =
f(x) = cos x f(x) = - sen x
f(x) = tg x f(x) = sec
2
x
7) funo exponencial: f(x) = a
x
f(x) = a
x
lna
Todas estas propriedades que acabamos de mencionar podem ser
demonstradas a partir da definio da derivada em termos da razo
incremental. Demonstraremos aqui apenas uma delas, a da funo produto f(x)
= u(x) v(x), e deixaremos as outras para o curso de Clculo. Neste caso
temos:
( ) ( ) ( ) ( ) ( )
x
x v x u x x v x x u
x
) x ( f x x f

+ +
=

+

( ) ( ) ( ) ( ) ( ) ( ) ( ) ( )
x
x v x x u x v x x u x v x u x x v x x u

+ + + + +
=
( ) ( ) ( ) ( ) ( ) ( ) | |
x
x u x x u x v ] x v x x v [ x x u

+ + + +
=
Tomando o limite para x tendendo a zero:

S. C. Zilio e V. S. Bagnato Mecnica, calor e ondas







Um pouco de clculo 15


( )
( ) | |
( )
( ) ( ) | |
)
`

+
+
)
`

+
+ =


x
x u x x u
x v lim
x
x v ) x x ( v
x x u lim ) x ( ' f
0 x
o x

de onde obtemos: ( ) ( ) ( ) ( ) ( ) x ' u x v x ' v x u x ' f + =
Regra da cadeia: Muitas vezes, durante o uso de derivadas em Fsica,
encontramos a situao em que ) y ( g ) x ( F = , com y = f(x), o que
corresponde chamada funo composta, isto , funo de uma outra funo.
Por exemplo, F(x) = sen (x
2
), de onde temos g(y) = siny e y = x
2
.

Neste caso,
devemos usar a regra da cadeia, dada por:
dx
dy
dy
dg
dx
dF
=
No presente exemplo F(x) = sen x
2
, com g(y) = siny e y = x
2
.

Logo,
y cos dy / dg = e ) x ( cos x 2 ) x ( ' F x 2 dx / dy
2
= =
Tomemos um outro exemplo onde
4 3 2
) x 3 x 2 1 ( ) x ( F + + = .
Chamando x 3 x 2 1 y
3 2
+ + = , temos g(y) = y
4
de forma que a derivada :
F(x) = 4y
3
(4x + 9x
2
) = 4(1+2x
2
+ 3x
3
)
3
(4x + 9x
2
)
1.3 Integrao
Como acabamos de ver, conhecendo-se a funo f(x) possvel
calcular sua taxa de variao f(x) (derivada). Uma pergunta lgica a ser feita
neste ponto : conhecendo-se f(x) possvel encontrar-se f(x), ou em outras
palavras, existe a operao inversa, ou anti-derivada? A resposta sim e a
operao inversa denominada integrao ser discutida a seguir de uma forma
bastante intuitiva, deixando-se o rigor matemtico para o curso de Clculo.
Vamos considerar a funo f(x) mostrada na Fig. 1.15 e supor
conhecidas as derivadas em todos os pontos x (x
0
, x
1
, x
2
, ...). Pela definio de
taxa de variao (ou razo incremental) temos:


S. C. Zilio e V. S. Bagnato Mecnica, calor e ondas






16 Um pouco de clculo











Fig. 1.15 Funo f(x) usada para a demonstrao da operao inversa da derivada.
1 taxa
x x
) x ( f ) x ( f
0 1
0 1
=


tal que f(x
1
) = f(x
0
) + taxa 1.(x
1
x
0
). Assim, conhecendo-se a taxa de
variao e a funo no ponto x
0
, temos condies de determinar a funo no
ponto x
1
. Da mesma forma, conhecendo-se a funo no ponto x
1
e a taxa 2,
que a taxa entre x
1
e x
2
, podemos determinar a funo em x
2
. Se dividirmos o
eixo x em vrios intervalos sucessivos nos quais conhecemos a taxa de
variao da funo f(x), podemos mostrar que:
f(x
n
) = f(x
0
) + taxa 1.(x
1
x
0
) + taxa 2.(x
2
x
1
) + ... taxa n.(x
n
x
n-1
)
de forma que podemos encontrar a funo f(x) e sabermos as vrias taxas de
variao ao longo do eixo x. Vamos, a seguir, tomar todos os intervalos com o
mesmo tamanho, ou seja:
x
1
x
0
= x
2
x
1
= ... = x
n
x
n-1
= x
de modo que:
f(x
n
) = f(x
0
) + (taxa1 + taxa 2 + ... + taxa n). x
Tomando o limite em que x tende a zero, as vrias taxas de variao
transformam-se nas derivadas, de modo que:
x
f(x)
x
1
x
2
x
3
x
0
= 0
taxa 1
taxa 3
taxa 2

S. C. Zilio e V. S. Bagnato Mecnica, calor e ondas







Um pouco de clculo 17


( ) ( ) ( ) x
dx
df
x f x f
s ' s todos
0 n
+ =


Como fizemos x 0, temos agora um nmero infinito de intervalos
e, consequentemente, infinitos termos na somatria. Alm disto, estamos
somando nmeros df/dx que variam continuamente. Neste caso, ao invs de
usarmos a soma

de nmeros discretos, introduzimos a operao

,
denominada integrao, que representa uma soma contnua. A partir desta
definio, podemos escrever:
( )

+ =
n
0
x
x
0 n
dx
dx
df
) x ( f ) x ( f
onde usamos dx x como notao no caso em que x 0. Como vemos,
esta operao permite encontrar-se f(x) a partir de f(x) e por isso dizemos que
a integrao a operao inversa da diferenciao. Se quisermos, por
exemplo, calcular a integral:
( ) ( )

+
+
=
+
= =
+
+
C
1 m
x
dx x
dx
d
1 m
1
dx x I
1 m
1 m m

onde a constante C est representando f(x
0
), que deve ser conhecido. A regra
acima bastante importante na integrao de polinmios. Alguns exemplos
simples so:

+ = C
3
x
dx x
3
2

( )

+ + + = + + C x
2
x
3
x
dx 1 x x
2 3
2

( )

+ + = + C x 4 x
8
5
dx x 8 x 5
2 8 7

A integral de uma determinada funo tambm possui uma
interpretao geomtrica como no caso da derivada. Para vermos tal

S. C. Zilio e V. S. Bagnato Mecnica, calor e ondas






18 Um pouco de clculo


interpretao, vamos considerar

n
0
x
x
. dx ) x ( g Para cada ponto x, multiplicamos
o valor da funo g(x) por uma largura dx mostrada na Fig. 1.16
(infinitesimalmente pequena) e somamos todos os produtos. Em cada ponto
temos a rea de um retngulo infinitesimal de base dx e altura g(x). Baseados
neste fato, podemos interpretar geometricamente a integral de uma funo g(x)
como sendo a rea sob a curva, isto , ( )

=
n
0
x
x
dx x g rea sob a funo g(x)
entre os pontos x
0
e x
n
.








Fig. 1.16 - Interpretao geomtrica da integral.
Podemos verificar este fato calculando a integral de g(x) = 4x entre 0
e 1, e comparando o valor obtido com a rea da funo neste intervalo. Temos:
( )

= = = =
1
0
1
0
1
0
2
2 0 1 . 2
2
x
4 dx x 4 dx x 4
Nesta ltima passagem introduzimos os limites de integrao,
substituindo a constante de integrao C.
( )

= =
b
a
b
a
) a ( F ) b ( F x F dx ) x ( g
Calculando a rea do tringulo sombreado da Fig. 1.17 obtemos: rea
= .4.1 = 2, que coincide com o resultado obtido por integrao.


g(x)
dx

x
n
x
0

g(x)
x

S. C. Zilio e V. S. Bagnato Mecnica, calor e ondas







Um pouco de clculo 19









Fig. 1.17 rea da funo g(x) = 4x entre 0 e 1.
Algumas propriedades importantes das integrais so:
(1) c g(x) dx = c g(x) dx onde c uma constante
(2) [g
1
(x) + g
2
(x)] = g
1
(x) dx + g
2
(x) dx
(3) senx dx =
dx
d
(-cos x) dx = - cosx + C
(4) cosx dx =
dx
d
(senx) dx = senx + C
1.4 Interpretao cinemtica das derivadas e
integrais
Na cinemtica encontramos vrias aplicaes do clculo de derivadas
e integrais. Analisando o movimento de um corpo, estas idias fluem
espontaneamente dos argumentos fsicos. Vamos considerar um corpo
deslocando-se numa trajetria S, conforme mostra a figura abaixo. Chamamos
de i e f os pontos inicial e final do movimento. O conhecimento especfico da
trajetria no suficiente para predizermos a velocidade do corpo para cada
posio. necessrio o conhecimento das posies sucessivas S(t) com o
decorrer do tempo. Suponha que a trajetria do corpo seja dividida em
pedaos s
r
, como mostra a Fig. 1.18. Um s
r
particular liga o ponto S
j
ao
ponto S
j+1
e o intervalo de tempo decorrido para que o corpo execute este
deslocamento t. A velocidade mdia neste intervalo de tempo
t / s v =
r r
. Esta velocidade ser to mais prxima da velocidade real
0.0 0.5 1.0 1.5 2.0
0
1
2
3
4
x
g(x)

S. C. Zilio e V. S. Bagnato Mecnica, calor e ondas






20 Um pouco de clculo


(instantnea) do corpo na posio S
j
quanto mais prximos forem os pontos j e
j +1. Isto ocorre porque neste caso s
r
confunde-se cada vez mais com a
trajetria real do corpo. No limite em que t (e consequentemente, s
r
) tende
a zero, temos a definio da velocidade instantnea:
dt
s d
t
s
lim v
0 t i
r r
r
=
|

\
|

=


que derivada da posio em relao ao tempo. Suponha agora que queremos
encontrar a distncia total percorrida pelo corpo. Isto pode ser feito dividindo-
se a trajetria em pequenos segmentos S
j
e realizando a soma S
j
.








Fig. 1.18 - Corpo deslocando-se numa trajetria S.
bvio que quanto menores forem os segmentos S
j
, mais a soma
acima se aproximar da distncia real percorrida pelo corpo, porque,
novamente, quanto menores forem os S
j
, melhor eles se encaixam na
trajetria. No limite em que S
j
0 eles se confundem completamente com a
trajetria e assim:
distncia percorrida = lim
Sj 0


S
j

usual no caso em que S
j
0 definirmos S = ds e substituirmos a
somatria pela integral:
distncia percorrida =

j
i
S
S
ds
x
y
i
f
s
j
s
j+1 s
r

S
j
S
j+1
S
j

S. C. Zilio e V. S. Bagnato Mecnica, calor e ondas







Um pouco de clculo 21


Exerccios
1 Uma sala tem dimenses 3 x 4 x 5 m
3
. Uma mosca parte de um de seus
cantos e voa para o canto diametralmente oposto. Qual o mdulo do
deslocamento? Poderia sua trajetria ser menor do que este deslocamento?
Escolha um sistema de coordenadas convenientes e escreva este
deslocamento na forma vetorial.
2 Considere os vetores . k

b j

b i

b b e k

a j

a i

a a
z y x z y x
+ + = + + =
r
r

Mostre que
z z y y x x
b a b a b a b . a + + =
r
r
e que ( ) i

b a b a b a
y z z y
=
r
r

( ) ( ) k

b a b a j

b a b a
x y y x z x z x
+ + .
3 Podemos combinar dois vetores de mdulos diferentes e ter resultante
nula? E no caso de 3 vetores?
4 Considere um corpo em movimento cujo vetor posio dado (em cm) por
. j

t sen 4 i

t cos 3 r + =
r
Usando procedimento semelhante ao utilizado
no texto para o movimento circular, a) mostre num grfico em escala o
vetor r
r
num determinado instante t; b) aps um intervalo de tempo t
pequeno, mostre no mesmo grfico o novo vetor r
r
; c) calcule o
deslocamento ) t ( r ) t t ( r s
r r r
+ = sofrido pelo corpo no intervalo t;
d) calcule t / s v =
r r
e verifique sua orientao para t = 0, /2, e 3/2;
e) calcule v . r
r r
e discuta o resultado; f) calcule v r
r r
e discuta o resultado.
5 Considere os vetores . k

3 j

2 i

b e k

4 j

3 i

2 a + = + + =
r
r

a) determine: . b a e b a , b a , b . a
r
r
r
r
r
r
r
r
+
b) qual a componente de a
r
paralela a b
r
?
c) qual a componente de a
r
perpendicular a b
r
?
6 Considere o vetor a
r
do problema anterior.
a) faa um grfico em escala mostrando o vetor e os ngulos e ,
definidos na Fig. 1.19.
b) calcule o mdulo do vetor e os valores de e .
c) calcule a componente de a
r
paralela ao versor ( ) 3 / k

e + + = .
d) calcule a componente perpendicular a este vetor.

S. C. Zilio e V. S. Bagnato Mecnica, calor e ondas






22 Um pouco de clculo









Fig. 1.18
7 Faa a adio e subtrao geomtrica dos seguintes vetores:
j

3 b e j

2 a
2
3
2
1
+ = =
r
.
8 Faa os produtos escalar e vetorial dos vetores: k

3 j

2 i

a + + =
r

e k

2 j

4 i

2 b + =
r
.
9 Encontre a projeo do vetor k

3 j

2 i

a + + =
r
na direo paralela ao versor
( ) . 3 / k

2 j

2 i

e + = Faa o mesmo para a projeo perpendicular.


10 Mostre que o produto vetorial r v
r r
um vetor constante quando o
movimento circular.
11 Mostre que 0 r . v =
r r
para o movimento circular. O que isto significa?
12 Calcule a derivada das seguintes funes:
a) f(x) = 3x
2
+ 1
b) f(x) = senx/x
2

c) f(x) = e
x
(1+ x
2
+ x
3
)
d) f(x) = (x
2
+ 2)/(x
3
+ 3)
13 Calcule a derivada das funes acima nos pontos:
a) x = 0
b) x =
c) x = 0
d) x = 1

y
y
z
z
a
r
r
x
x

P

S. C. Zilio e V. S. Bagnato Mecnica, calor e ondas







Um pouco de clculo 23


14 Procure num handbook de matemtica:
a) a derivada de f(x) = lnx
b) a integral de f(x) = 1/x
15 Determinar a derivada das seguintes funes:
a) y = 4x
5

b) y = 2x
3
+ 4x
2
5x 2
c) y = sen x + cos x
d) y = x
2
+ 1
e) y = x sen x
f) y = 1/x
2

g) y = ( ) 1 x / x 2
2
+
h) y = x e
x

i) y = cotg x
j) y = x
k) y = x / 1
16 Calcule as derivadas das funes:
a) f(x) = tgx
b) f(x) = e
ax
(no ponto x = 0)
c) f(x) = sen2x (no ponto x = )
d) f(x) = x
n
+ cosx
e) f(x) = sen (cosx)
f) f(x) = e
senx
(no ponto x = 0)
17 Calcule

+
1
0
2
x 1
dx
. Sugesto: Faa x = tg 1 + x
2
= 1 + tg
2
=
sec
2
. Por outro lado, dx/d = sec
2
dx = sec
2
d. Como x = tg, os
limites de integrao ficam: quando x = 0 = 0 e quando x = 1 =
4

.
18 Calcule as seguintes integrais indefinidas:
a) I =

dx x 3
b) I = ( )

+ dx 2 x 4 x 7
3 2


S. C. Zilio e V. S. Bagnato Mecnica, calor e ondas






24 Um pouco de clculo


c) I = ( ) dx x 8 x 15
2 10

+
19 Calcule as integrais definidas:
a) I = ( )

+
0
dx x cos x sen 3
b) I = dx ) x 2 5 (
1
1
2

+
c) I =

1
0
x 2
dx e
d) I =

4
0
dx x cos x sen
20 - Considere a parbola y = 2x
2
+x-3.
a) Usando o conceito de derivada, encontre a posio x
0
que corresponde
ao extremo (mximo ou mnimo);
b) Substituta o valor de x
0
na equao da parbola para encontrar o valor
de y
0
;
c) Complete quadrados para encontrar os pontos do vrtice, x
0
e y
0
;
d) Encontre os pontos para os quais a parbola cruza o eixo x;
e) Faa um esboo (grfico com poucos detalhes) da parbola;
f) Usando integrao, encontre a rea sob a parbola compreendida entre
os pontos 1 e 2.







Movimento unidimensional 25


S. C. Zilio e V. S. Bagnato Mecnica, calor e ondas








2.1 Introduo
Dentre os vrios movimentos que iremos estudar, o movimento
unidimensional o mais simples, j que todas as grandezas vetoriais que
descrevem o movimento so paralelas. Como o movimento ocorre em apenas
uma dimenso, necessria apenas uma coordenada para especificar a posio
de um corpo em cada instante de tempo.
Consideremos um corpo que no instante t
1
encontra-se na posio x
1
.
Aps um intervalo de tempo t = t
2
t
1
, o corpo estar na posio x
2
no
instante de tempo t
2
. Definimos o deslocamento como sendo x = x
2
x
1
e a
velocidade mdia do corpo neste intervalo de tempo como:
1 2
1 2
t t
x x
t
x
v

=
O sentido do deslocamento do corpo dado pelo sinal do prprio
deslocamento ou da velocidade mdia (so proporcionais). Geometricamente,
a velocidade mdia entre os pontos x
2
e x
1
corresponde inclinao da reta
quer passa por estes pontos, conforme mostra a Fig. 2.1.



tg = t / x v =



Fig. 2.1 - Posio de um corpo com funo do tempo.
2
MOVIMENTO
UNIDIMENSIONAL
t
1
t
2


x
t
t
x








Movimento unidimensional

S. C. Zilio e V. S. Bagnato Fsica Bsica Mecnica, calor e ondas


26
Quanto menor for o intervalo de tempo considerado, isto , quanto
mais prximos estiverem os pontos x
1
e x
2
, mais fielmente v representar a
velocidade real do corpo naquele intervalo de tempo. Logo, a velocidade
instantnea (real) definida como:
( )
dt
dx
t
x
lim t v
0 t
=

=


que nada mais do que a derivada da posio com relao ao tempo.
Geometricamente, se tivermos um grfico de posio contra tempo, a
velocidade instantnea corresponde inclinao da reta tangente curva num
determinado instante de tempo, como ilustra a Fig. 2.2.


tg
1
= v(t
1
)
tg
2
= v(t
2
)



Fig. 2.2 - Interpretao geomtrica da velocidade instantnea.
Quando a velocidade instantnea constante num determinado
intervalo de tempo, dizemos que o movimento uniforme e que v ) t ( v = . Por
outro lado, quando a velocidade no constante no tempo, o movimento
chamado de acelerado. Neste caso, a variao da velocidade com o tempo
caracterizada por uma grandeza denominada acelerao. Se a velocidade do
corpo no instante t
1

1
v e no instante t
2

2
v , a acelerao mdia definida
como:
t
v
t t
v v
a
1 2
1 2

=
e no grfico de velocidade contra tempo ela corresponde inclinao da reta
que passa pelos pontos v
1
e v
2
. Quando consideramos o limite em que t tende
x
t

2
t
1
t
2








Movimento unidimensional 27


S. C. Zilio e V. S. Bagnato Mecnica, calor e ondas


a zero, surge a idia de acelerao instantnea, grandeza esta que caracteriza
localmente a variao da velocidade do corpo. Logo:
( )
dt
dv
t
v
lim t a
0 t
=

=


Geometricamente, a acelerao a inclinao da reta tangente curva
no grfico de velocidade, como mostra a Fig. 2.3.


tg = a(t)



Fig. 2.3 Interpretao geomtrica da acelerao instantnea.
O movimento do corpo pode ser classificado de acordo com a maneira
em que a acelerao se comporta no tempo. Quando a acelerao constante,
o movimento chamado de uniformemente acelerado e se constitui numa
classe importante de situaes que analisaremos. Antes de prosseguirmos,
vamos mostrar alguns exemplos dos conceitos que acabamos de ver.
Exemplo 1 : Seja um corpo deslocando-se de tal forma que sua
posio dada por x(t) = 4t
2
, com t dado em s e x em cm. Na Fig. 2.4(a)
vemos o grfico desta funo. A velocidade do corpo em cada instante de
tempo pode ser encontrada tomando-se a derivada de x(t) e assim,







Fig. 2.4 - Posio (a) e velocidade (b) de um corpo como funo do tempo.

t
t
v(t)
t (s)
x(t)
36
27
18
4 3 2 1
9
0
(cm)
t (s)
v(t)
32
24
16
4
3 2 1
8
0
(cm/s)
0
0








Movimento unidimensional

S. C. Zilio e V. S. Bagnato Fsica Bsica Mecnica, calor e ondas


28
( ) t 8
dt
dx
t v = = (em cm/s)
que a equao da linha reta mostrada na Fig. 2.4(b). Se quisermos calcular a
acelerao como funo do tempo, devemos tomar a derivada de v(t) que
obviamente uma constante.
( )
2
s / cm 8
dt
dv
t a = =
A velocidade mdia do corpo entre os instantes t = 1s e t = 3s pode ser
calculada atravs da expresso:
( ) ( )
s / cm 16
2
4 36
1 3
1 x 3 x
t
x
v =

=
Este mesmo resultado poderia ser obtido da seguinte forma:
( ) ( )
s / cm 16
2
8 24
2
1 v 3 v
v =
+
=
+
=
ou seja: A velocidade mdia a mdia das velocidades nos instantes
considerados. Este um resultado que s vale para um movimento cuja
acelerao constante.
Exemplo 2: O movimento de um corpo descrito por x(t) = 3t
2
+ 4t +
1, sendo esta funo mostrada na Fig. 2.5. A posio inicial do corpo x
0
= 1
cm e pelo grfico vemos que nos instantes iniciais do movimento, o
deslocamento se d no sentido positivo do eixo x, at atingir um ponto
mximo a partir do qual o movimento se inverte, ocorrendo a partir da no
sentido negativo do eixo x.
Queremos responder seguinte pergunta: quanto tempo o corpo leva
para voltar posio inicial? Para isto fazemos x(t) = 1, isto ,
-3t
2
+ 4t + 1 = 1 -3t
2
+ 4t = 0 t (-3t + 4) = 0
de onde tiramos que o corpo est na posio x = 1 nos instantes t = 0 (posio
inicial) t = 4/3 s, que corresponde ao tempo necessrio para a partcula voltar
posio inicial.







Movimento unidimensional 29


S. C. Zilio e V. S. Bagnato Mecnica, calor e ondas













Fig. 2.5 - Posio de um corpo como funo do tempo.
A velocidade dada por v(t) = dx/dt = -6t + 4 (cm/s), que est
mostrada na Fig. 2.6. Notamos que: v > 0 para t < 2/3 s, v = 0 para t = 2/3 s
e v < 0 para t > 2/3 s. O grfico da velocidade do corpo corresponde uma
reta com coeficiente angular negativo. O tempo t = 2/3 s define o ponto de
retorno. A acelerao dada por:
2
s / cm 6
dt
dv
a = =
e no sentido oposto ao da velocidade na fase inicial (t < 2/3 s).










Fig. 2.6 - Velocidade de um corpo como funo do tempo.
0
x(cm)
-2
2
1
1.5 2.0 1.0 0.5
-1
0
2.5
t (s)
0
v (cm/s)
-4
4
2
2/3 1/3
-2
0
1
t (s)








Movimento unidimensional

S. C. Zilio e V. S. Bagnato Fsica Bsica Mecnica, calor e ondas


30
2.2 Classificao dos movimentos unidimensionais
O movimento unidimensional classificado de acordo com as
variaes da posio, velocidade e acelerao com o decorrer do tempo.
Assim, temos os seguintes tipos de movimentos:
Progressivo: x(t) aumenta com o tempo;
Retrgrado: x(t) diminui com o tempo;
Acelerado: v(t) e a (t) tem o mesmo sentido;
Retardado: v(t) e a(t) tem sentidos opostos.
No exemplo anterior (Exemplo 2), a classificao do movimento : t <
2/3s movimento progressivo e retardado e t > 2/3x movimento
retrgrado e acelerado.
2.3 Determinao de x(t) a partir de v(t) e de
v(t) a partir de a(t)
Como vimos anteriormente, o conhecimento de x(t) permite o clculo
de v(t) atravs de uma derivao e tambm a(t) atravs de outra derivao. O
problema inverso consiste na determinao de x(t) a partir de v(t) ou a(t). Para
isto, temos que realizar uma integrao, pois estamos procurando a funo
cuja derivada conhecida. Assim,
( ) ( ) ( )

+ = + =
t
t
t
t
0 0
0 0
dt t v x dt
dt
dx
x t x
Conhecendo-se a velocidade do corpo, determinamos sua posio
como funo do tempo atravs de uma integrao simples. Lembre-se que o
que estamos fazendo nada mais do que dividir o intervalo de tempo total em
pequenos intervalos dt nos quais a velocidade considerada constante. O
produto vdt fornece a pequena distncia percorrida (ou deslocamento sofrido)
em dt e a soma deles, que a operao de integrao, fornece o deslocamento
total do corpo. Num grfico de v(t) contra t, o deslocamento do corpo a rea
sob a curva, como mostrado na Fig. 2.7. Note que rea negativa indica
deslocamento no sentido negativo do eixo x.








Movimento unidimensional 31


S. C. Zilio e V. S. Bagnato Mecnica, calor e ondas











Fig. 2.7 - Clculo da posio a partir da velocidade de um corpo.
Exemplo 1: A velocidade de um corpo dada por: v(t) = 3t + 4 e
sabemos que para t = 0 ele localiza-se em x
0
= 1. Vamos calcular x(t). Assim,
( ) ( ) 1 t 4 t dt 4 t 3 1 t x
2
t
0
2
3
+ + = + + =


Exemplo 2: Dado a(t) = 3t, calcular v(t) e x(t)
( )
2
t
0
0 0
t v dt t 3 v t v
2
3
+ = + =


Vemos que para conhecer v(t) precisamos saber a velocidade inicial. Para
achar x(t) fazemos:
( ) ( ) ( )
2 2
3
3
0 0
t
0
2
0 0
t
0
0
t
t v x dt t v x dt t v x t x + + = + + = + =


Deste exemplo podemos concluir que para a determinao de v(t) a
partir de a(t) necessrio o conhecimento do valor inicial v
0
da velocidade. A
determinao precisa de x(t) a partir de v(t) implica no conhecimento da
posio x
0
inicial. x
0
e v
0
so denominados de condies iniciais do
movimento.
t
v(t)
t
0
t
rea = x(t)








Movimento unidimensional

S. C. Zilio e V. S. Bagnato Fsica Bsica Mecnica, calor e ondas


32
2.4 Acelerao constante
Este caso envolve um nmero grande de problemas e, assim, devemos
trata-lo em particular. Sendo a acelerao constante, podemos calcular a
velocidade como:
( ) at v dt a v dt a v t v
t
0
0
t
0
0 0
+ = + = + =


e o deslocamento atravs de outra integrao:
( ) ( ) ( )

+ + = + + = + =
t
0
2
0 0 0 0
t
0
0
at
1
t v x dt at v x dt t v x t x
2

Podemos eliminar t da primeira equao: ( ) a / v v t
0
= e substitu-
lo na segunda:
( ) ( )
( )

+ + =
2
2
0
0
0
0
a
v v
a
2
1
v v
a
v
x t x
( ) ( )
2
v
2
v
vv 2 v v
2
1
v v v x x a
2
0
2
0
2
0
2 2
0 0 0
= + + =
Logo: ( )
0
2
0
2
x x a 2 v v + = , que conhecida como equao de Torricelli,
vlida apenas quando a acelerao constante.
Um caso especial do movimento uniformemente acelerado ocorre para
a = 9.81 m/s
2
= g, que corresponde ao movimento vertical de corpos sujeitos
ao campo gravitacional da Terra, prximos superfcie. Neste caso, comum
tratar o deslocamento como altura (h) e adotar o sentido positivo de h como
sendo oposto ao de g.
Exemplo: Uma bola lanada para cima, com velocidade inicial v
0

como mostra a Fig. 2.8. Assim, usando a equao de Torricelli temos:
( ) ( ) gh 2 v h v gh 2 v h v
2
0
2
0
2
= =
Para um determinado h, existem duas solues para v. A positiva
representa o corpo em ascenso e a negativa o corpo est na descendente.
Vemos tambm que o ponto de retorno (v = 0) ocorre para uma altura mxima







Movimento unidimensional 33


S. C. Zilio e V. S. Bagnato Mecnica, calor e ondas


h
max
= g 2 / v
2
0
mostrada na Fig. 2.9. Por outro lado, a dependncia temporal
dada por v(t) = v
0
gt e h(t) = gt
2









Fig. 2.8 Lanamento vertical de uma bola.
Ao atingir o ponto mximo da trajetria, v = 0 e t
max
= v
0
/g. Logo: h
max

= g 2 / v
2
0
como obtido anteriormente. Para a obteno do tempo total da
trajetria fazemos h(t
f
) = 0 0 = t (v
0
- gt
2
1
) que nos d duas solues: t
i
= 0
(incio do movimento) e t
f
= 2v
0
2
/g que o dobro do tempo gasto para que a
bola atinja h
max
.











Fig. 2.9 Dependncia da velocidade com a altura no lanamento vertical.
v(h)
v
0
2
2g
h

+h v
0
g
r









Movimento unidimensional

S. C. Zilio e V. S. Bagnato Fsica Bsica Mecnica, calor e ondas


34
Exerccios
1 O maquinista de um trem movendo-se com velocidade v
1
, v, a uma
distncia d sua frente, um trem cargueiro movendo-se no mesmo sentido
com velocidade v
2
. Ele aciona os freios, transmitindo ao trem uma
acelerao -a. Mostre que se: d > (v
1
- v
2
)
2
/2a no haver coliso e se d <
(v
1
- v
2
)
2
/2a haver coliso.
2 Gotas de gua caem de um chuveiro sobre o piso situado a 2 m abaixo. As
gotas caem em intervalos regulares e quando a primeira atinge o cho, a
quarta est comeando a cair. Determine a posio de todas as gotas no
instante em que uma tinge o cho.
3 A posio de uma partcula que se desloca ao longo do eixo x depende do
tempo de acordo com a equao: x = at
2
bt
3
, x em cm, t em s.
a) em que ponto x mximo?
b) qual a velocidade e em que instante ela nula?
c) qual a acelerao e em que instante ela nula?

4 Um avio com velocidade v
0
aterriza num porta-avies com uma
acelerao negativa t A a = . Qual o comprimento mnimo da pista?
5 Dois corpos localizam-se na origem do eixo x quando t = 0 s. O corpo A
tem velocidade constante de 2 m/s. O corpo B est inicialmente em
repouso mas sujeito a uma acelerao constante de 1 m/s
2
.
a) represente esquematicamente, num mesmo grfico, as posies dos
corpos A e B como funo do tempo.
b) qual o instante de tempo em que ocorrer a coliso?
c) qual a posio x em que isto ocorrer?
d) qual a velocidade do corpo B no instante da coliso?
e) em que instante de tempo as velocidades dos dois corpos sero iguais?







Movimentos bi e tridimensional 35


S. C. Zilio e V. S. Bagnato Mecnica, calor e ondas








3.1 Introduo
O movimento unidimensional que vimos no captulo anterior um
caso particular de uma classe mais ampla de movimentos que ocorrem em
duas ou trs dimenses. Se o movimento de um corpo est completamente
restrito a um plano, ele denominado movimento plano ou bidimensional.
Neste caso, a posio especificada atravs de coordenadas polares (r, ) ou
cartesianas (x, y), como indicadas na Fig. 3.1.

2 2
y x r + =
x = r cos
y = r sen
tg = y/x
Fig. 3.1 Posio de um corpo no plano xy.
Para o caso do movimento no espao (3 dimenses) a posio do
corpo especificada em coordenadas esfricas (r, , ) ou cartesianas(x, y, z),
indicadas na Fig. 3.2.

=
=
=
cos r z
sen sen r y
cos sen r x


x / y tg
z / y x tg
z y x r
2 2
2 2 2
=
+ =
+ + =

Fig. 3.2 - Posio de um corpo no espao.
3
MOVIMENTOS BI E
TRIDIMENSIONAL

x
x
y
r
P y

y
y
z
z
r
x
x

P







Movimentos bi e tridimensional


S. C. Zilio e V. S. Bagnato Fsica Bsica Mecnica, calor e ondas


36
Para movimentos planos e espaciais, as grandezas cinemticas
( a e v , r
r r r
) no so necessariamente paralelas como acontece no movimento
unidimensional. Desta forma, de importncia fundamental tratar estas
grandezas vetorialmente.
Se no tempo t
1
a posio do corpo for descrita pelo vetor posio
1
r
r
e
no tempo t
2
, pelo vetor posio
2
r
r
, podemos dizer que o deslocamento sofrido
pelo corpo dado por
1 2
r r r
r r r
= onde r
r
no necessariamente a
distncia percorrida pelo corpo. Havendo um deslocamento r
r
num intervalo
de tempo t = t
2
t
1
, podemos definir as velocidades mdia ( )
m
v
r
e instantnea
( ) v
r
da forma:
t
r
v
m

=
r
r

dt
r d
t
r
lim v
0 t
r r
r
=

=


Vemos que a velocidade sempre existir quando houver mudanas no
mdulo e/ou direo do vetor posio. A variao temporal de um vetor pode
ser analisada atravs da variao temporal de suas componentes, da forma:
k

dt
dz
j

dt
dy
i

dt
dx
v k

z i

y i

x r + + = + + =
r r

e isto pode ser feito porque os versores k

e j

, i

no variam com o tempo.


Exemplo: Vamos determinar a velocidade de um corpo cujo vetor
posio dado por: j

t 3 i

t 4 r
2
+ =
r
. Tomando-se as derivadas temporais das
componentes de r
r
temos:
j

3 i

t 8 dt / r d v + = =
r r

Vamos usar este exemplo para demonstrar uma relao importante. Podemos
escrever:






Movimentos bi e tridimensional 37


S. C. Zilio e V. S. Bagnato Mecnica, calor e ondas


( ) ( ) ( ) ( ) i

t 4 j

t 3 i

t t 8 j

t 3 i

t 4 j

t t 3 i

t t 4 t t r
2 2 2
+ + + + = + + + = +
r
No caso em que t muito pequeno, (t)
2
<< t e o termo (t)
2
pode
ser desprezado. Assim,
( ) ( ) ( ) t v t r r t r t t r + = + = +
r r r r r

e dizemos que esta uma aproximao de primeira ordem em t, j que o
termo (t)
2
foi desprezado. A acelerao do corpo definida como:
dt
v d
t
v
lim a
0 t
r r
r
=

=


e, portanto, sempre haver acelerao quando houver mudanas do vetor
velocidade, seja em mdulo, direo ou sentido.
Exemplo: A velocidade de um corpo dada por ( ) k

t j

t i

t 3 t v
3 2
+ + =
r
.
Logo, a acelerao dada por ( ) k

t 3 j

t 6 t a
2
+ + =
r

3.2 Decomposio de movimentos
Do fato que k

dt
dz
j

dt
dy
i

dt
dx
v + + =
r
tiramos que dt / dx v
x
= , v
y
=
dy/dt e dt / dz v
z
= , de modo que se olharmos para cada componente, o
movimento do corpo pode ser analisado independentemente, ou seja, a
velocidade na direo x s depende da variao da coordenada x com o tempo,
etc. Este resultado pode ser generalizado e o movimento espacial de um corpo
pode ser tratado independentemente em cada uma das trs direes.
Resumindo, temos o chamado princpio da independncia dos movimentos ou
princpio de Galileu:
Quando um corpo se encontra sob a ao simultnea de dois ou mais
movimentos, cada um se processa como se os demais no existissem.
Em outras palavras, a posio do mvel depois de um intervalo de
tempo sob a ao do movimento composto a mesma que resultaria se o
mvel se deslocasse por etapas em cada direo. Como um exemplo tpico,







Movimentos bi e tridimensional


S. C. Zilio e V. S. Bagnato Fsica Bsica Mecnica, calor e ondas


38
consideremos o caso de um barco com velocidade v
b
atravessando um rio cuja
correnteza tem velocidade v
r
. O barco percorrer uma trajetria que consiste
em deslocar-se v
r
t na direo do rio e v
b
t na direo perpendicular, como
mostra a Fig. 3.3. Assim, j

v i

v v e j

t v i

t v r
b r b r
+ = + =
r
.









Fig. 3.3 - Movimento de um barco num rio com correnteza.
3.3 Movimento acelerado
Podemos generalizar o que vimos para o movimento unidimensional
escrevendo:
( ) dt t v r r
t
0
0

+ =
r r r

( ) ( ) dt t a v t v
t
0
0

+ =
r r r

A integrao de vetores pode ser executada componente a
componente, como no caso da derivao. Portanto,
( )dt t v r r
t
0
z
0
z z

+ =
e assim por diante. No caso da acelerao ser constante temos:
t a v v
0
r r r
+ = e
2
0 0
t a t v r r
2
1
r r r r
+ + =
i


v
r
v
r
t
v
b
t
j








Movimentos bi e tridimensional 39


S. C. Zilio e V. S. Bagnato Mecnica, calor e ondas


Podemos analisar este movimento atravs do sistema de equaes:
Para a velocidade: Para a posio:
t a v v
t a v v
t a v v
z
0
z z
y
0
y y
x
0
x x
+ =
+ =
+ =

2
z 2
1
0
z
0
z z
2
y 2
1
0
y
o
y y
2
x 2
1
0
x
0
x x
t a t v r r
t a t v r r
t a t v r r
+ + =
+ + =
+ + =

Vamos em seguida ver alguns exemplos de movimento acelerado.
a) Lanamento de projtil
Um caso importante de movimento plano aquele onde temos:
j

g a =
r
(com g = 9.8 m/s
2
) que corresponde ao movimento de um corpo
atirado de maneira arbitrria. Neste caso, o movimento ser acelerado na
direo y e no acelerado nas demais. Vamos imaginar a situao em que o
corpo lanado obliquamente de maneira a formar um ngulo com a
superfcie, como mostrado na Fig. 3.4

=
=
sen v v
cos v v
0
0
y
0
0
x




Fig. 3.4 Lanamento oblquo de um projtil.
Tomando-se o eixo x paralelo superfcie e o eixo y na vertical, a
velocidade inicial v
0
pode ser decomposta em cos v v
0
0
x
= e
= sen v v
0
o
y
. Na direo x no existe acelerao, porm na direo y
temos a
y
= -g de modo que:
( )
( )

+ = + =
= =
t cos v x t v x t x
cos v v t v
0 0
0
x 0
0
0
x x


v
0
y
x







Movimentos bi e tridimensional


S. C. Zilio e V. S. Bagnato Fsica Bsica Mecnica, calor e ondas


40
( )

+ =
= =
2
2
1
0
y 0
o
0
y y
t g t v y t y
t g sen v t g v ) t ( v

Eliminando-se o tempo do primeiro conjunto de equaes
( ) ( )
0
x 0
v / x x t = e substituindo no segundo obtemos:
( )
2
0
x
0
0
x
0
0
y 0
v
x x
g
v
x x
v y y
2
1
|

\
|

+ =
que representa uma trajetria parablica como indicada na Fig. 3.5. A altura
mxima pode ser calculada tomando-se dy/dx = 0. Assim,
( )
g
v v
x x 0
v
x x
g
v
v
0
x
0
y
0 max
0
x
0
0
x
0
y
2
+ = =


e substituindo em y(t) tiramos:
( )
g
v
y y
2
0
y
0 max
2
1
+ =






Fig. 3.5 - Movimento parablico decorrente do lanamento oblquo.
Vamos tomar x
0
= y
0
= 0 e calcular qual o alcance do projtil ao
longo do eixo x. Para isto fazemos y = 0 e assim obtemos:
( )
2
0
x
2
0
x
0
y
v
R g
R
v
v
0
2
1
=
0
v
r


y
max
x
max
x
0 R
x
y
y
0
0







Movimentos bi e tridimensional 41


S. C. Zilio e V. S. Bagnato Mecnica, calor e ondas


Descartando a soluo R = 0, que corresponde ao incio do movimento, temos
R = g / v v 2
0
x
0
y
, e usando-se = = cos v v e sen v v
0
0
x 0
0
y
obtemos:
( )
g
2 sen v
R
2
0

=
de onde conclumos que o ngulo que apresenta o maior alcance = 45
o

b) Movimento circular
Este deslocamento caracterizado pelo fato de que o mdulo do
deslocamento permanece constante. Assim, imaginamos o raio vetor que
descreve o movimento entre t e t + t. O ngulo varrido pelo raio vetor
durante o intervalo de tempo t permite o clculo da velocidade angular como
ilustrado na Fig. 3.6.
t
lim
dt
d
0 t

=









Fig. 3.6 Movimento circular.
Quando constante, temos

= =
t
0
t dt e assim podemos
escrever: x = r cost e y = r sent, ou em notao vetorial:

r j

t sen r i

t cos r
dt
v d
a
j

t cos r i

t sen r
dt
r d
v
j

t rsen i

t cos r r
2 2 2
r
r
r
r
r
r
= = =
+ = =
+ =


t
t+t

x
y







Movimentos bi e tridimensional


S. C. Zilio e V. S. Bagnato Fsica Bsica Mecnica, calor e ondas


42
que sempre oposta a direo radial. Portanto, r / v r a a
2 2
= = =
r
visto
que = r v
r
e esta acelerao conhecida como centrpeta por estar
dirigida ao ponto central do movimento e uma caracterstica importante do
movimento circular uniforme.
c) Movimento ciclide
o movimento de um ponto da borda de um disco rodando, conforme
mostra a Fig. 3.7. Considerando um sistema de eixos no qual x paralelo ao
cho, temos a combinao de um movimento translacional uniforme com um
movimento circular uniforme. Para o movimento translacional, x
t
= x
0
+ v
x
t e,
para o movimento circular, x
0
= r cost e y
0
= r sent.









Fig. 3.7 - Movimento ciclide.
Desta forma,
t sen r y y
t cos r t v x x
0
x 0
+ =
+ + =

Ao utilizarmos a notao vetorial e fazendo x
0
= y
0
= 0,
( ) j

t sen r i

t cos r t v r
x
+ + =
r

r
x






Movimentos bi e tridimensional 43


S. C. Zilio e V. S. Bagnato Mecnica, calor e ondas


( )
c
2 2 2
x
r j

t sen r i

t cos r
dt
v d
a
j

t cos r i

t sen r v
dt
r d
v
r
r
r
r
r
= = =
+ = =

Exemplo: Considere um disco descendo um plano inclinado,
formando um ngulo com a horizontal, como mostrado na Fig. 3.8. Vamos
determinar x(t) e y(t) de um ponto localizado na borda do disco. Escolhendo o
eixo x da maneira indicada na figura, temos a
x
= g sen e a
y
= 0. Ento, x = x
t

+ x
c
, y = y
t
+ y
c
+ + = cos r t sen g
2
1
t v x
2 0
x
e + = sen r t v y
0
y
,
onde t (movimento acelerado) o ngulo que o disco rodou.









Fig. 3.8 Disco descendo um plano inclinado
3.4 Movimentos planos descritos por coordenadas
polares
Vamos considerar um movimento circular no qual o corpo percorre
um comprimento de arco s, que est associado a um ngulo de acordo com: s
= r, sendo r o raio da trajetria. A velocidade tangencial :
=

= = r
dt
d
r
dt
ds
v

r
P
x







Movimentos bi e tridimensional


S. C. Zilio e V. S. Bagnato Fsica Bsica Mecnica, calor e ondas


44
Para representar v
r
, vamos introduzir os versores r e

, que so
adequados para se trabalhar com coordenadas polares. O versor r tem a
mesma direo e sentido do vetor posio r
r
. O versor

perpendicular a r
r

e tangente ao crculo, apontando para a direo em que e s crescem como
indica a Fig. 3.9. Desta forma, podemos escrever r
r
e v
r
em coordenadas
polares da seguinte maneira:

= =
=

dt
d
r

v v
r r r
r
r







Fig. 3.9 Movimento plano descrito por coordenadas polares.
Devemos notar que r e

so versores que variam com o tempo. Para


encontrar esta variao em termos dos versores i

e j

que so fixos vemos


que j

sen i

cos r + = e j

cos i

sen

+ = . Desta forma,
( )
( ) r
dt
d
j

sen i

cos
dt
d
dt

dt
d
j

cos i

sen
dt
d
j

cos
dt
d
i

sen
dt
d
dt
r d

= +

= +

=

Uma vez que conhecemos a maneira pela qual r e

variam com o
tempo, podemos encontrar v
r
e a
r
a partir de r
r
.
i



y
x

r
r
r
j








Movimentos bi e tridimensional 45


S. C. Zilio e V. S. Bagnato Mecnica, calor e ondas


r
dt
d
r
dt

d
dt
d
r
dt
v d
a

dt
d
r
dt
r d
r
dt
r d
v
r r r
2
|

\
|
=

= =

= = =
=
r
r
r
r
r

onde foi suposto que = d/dt constante. Como d/dt = v/r, temos
( ) r r r r / v a
2 2
= =
r
, que a acelerao centrpeta no movimento circular
uniforme.
Se o movimento for uniformemente acelerado, isto , se d/dt = =
constante, a expresso para a acelerao se modifica. Tomando a derivada de
=

r v
r
temos:
r r

r
dt

dt
d
r a
2
=
|
|

\
|

+

=
r

de onde vemos que alm da acelerao centrpeta surge uma acelerao
tangencial dada por

r .
A descrio de um movimento retilneo atravs de coordenadas
polares feita baseando-se na Fig. 3.10. Podemos relacionar v
r
e v

da
seguinte forma:
v
x
= v
r
cos - v

sen
v
y
= v
r
cos + v

sen
ou
v
r
= v
x
cos + v
y
sen
v

= -v
x
sen + v
y
cos





Fig. 3.10 Descrio de um movimento retilneo atravs de coordenadas polares.

y
x

r
r
r v
r


v
y
v
x
v
r
v








Movimentos bi e tridimensional


S. C. Zilio e V. S. Bagnato Fsica Bsica Mecnica, calor e ondas


46
Para o caso que estamos tratando, v
x
= v e v
y
= 0. Portanto, v
r
= v
cos e v

= v sen, ou seja:
=

sen v r cos v v
r

Exerccios
1 Considere um cilindro de raio R rolando sem deslizar num plano
horizontal. O centro de massa do cilindro possui acelerao a. Qual a
acelerao angular do cilindro? Qual o ngulo que o cilindro roda
como funo do tempo?
2 Dois corpos A e B esto em movimentos circular uniformes de trajetrias
concntricas com raios r
a
e r
b
e velocidades angulares
a
e
b
. Determine
a velocidade relativa entre os dois corpos.
3 Determinar a acelerao de um corpo que desliza pela rosca de um
parafuso com passo h e raio R. Despreze o atrito e considere que o corpo
partiu do repouso.
4 necessrio lanar da terra uma bola por cima de uma parede de altura H
que se encontra a uma distncia S (Fig. 3.11). Qual a menor velocidade
inicial com que a bola pode ser lanada?







Fig. 3.11 Lanamento de projtil sobre uma parede de altura H.
0
v
r
H
S






Movimentos bi e tridimensional 47


S. C. Zilio e V. S. Bagnato Mecnica, calor e ondas


5 Uma bala disparada de um canho com velocidade v
0
. Determine a
regio geomtrica onde a bala certamente no cair.
6 Um plano inclinado forma um ngulo com o plano xy, conforme mostra
a Fig. 3.12. Um corpo lanado com velocidade v
0
, formando um ngulo
com o eixo y. Desprezando o atrito calcule: x
max
, z
max
e o tempo que o
projtil demora para retornar ao eixo y.
7 Uma pedra lanada com velocidade inicial de 20 m/s. Sabendo-se que ela
ficou 2 s no ar, calcule:
a) o ngulo de lanamento (com a horizontal)
b) a altura mxima atingida
c) o alcance
d) outro ngulo de lanamento para o qual a pedra ter o mesmo alcance.
(Neste caso o tempo ser diferente de 2 s).







Fig. 3.12 Lanamento oblquo num plano inclinado.
8 Um corpo translada com velocidade v = 5 m/s sobre um plano horizontal
sem atrito. Subitamente ele encontra pela frente um plano inclinado
(tambm sem atrito) de ngulo = 30
0
e altura H = 0,8 m, conforme
mostra a Fig. 3.13. Tomando-se g = 10 m/s, pergunta-se:
a) a que distncia d do final do plano inclinado o corpo cair?
b) qual a altura mxima que o corpo atingir?


0
v
r

y
x
z







Movimentos bi e tridimensional


S. C. Zilio e V. S. Bagnato Fsica Bsica Mecnica, calor e ondas


48






Fig. 3.13 - Lanamento oblquo de um corpo por meio de uma rampa.
9 Um pequeno corpo lanado da origem com velocidade v
0
= 100/ 3 m/s
formando um ngulo = 60
0
com a horizontal. Outro corpo lanado 1
segundo depois, com a mesma velocidade v
0
, porm na horizontal e de
uma altura H, como mostra a Fig. 3.14. Suponha que haja uma coliso
entre os dois corpos e que g = 10 m/s
2
.
a) Em que instante de tempo ocorre a coliso?
b) Qual deve ser o valor de H para que a coliso ocorra?
c) Quais as coordenadas x e y da coliso?
3.10 Um pequeno corpo lanado da origem com velocidade v
0
segundo um
ngulo com a horizontal. Outro corpo lanado com a mesma
velocidade v
0
, porm na horizontal e de uma altura H, como mostra a
Fig. 3.14. Qual deve ser o valor de H tal que eles atinjam o mesmo
ponto no eixo Ox?






Fig. 3.14 - Lanamento de dois corpos.

x
H


y
max
d
v
r
v
0
H
v
0
O

x








Movimentos bi e tridimensional 49


S. C. Zilio e V. S. Bagnato Mecnica, calor e ondas


3.11 - Mostre que o movimento de um projtil lanado com v
0
e descrito
pela parbola: y x
v
g
g x
v
v
g
y
x
y
( ) =
|
\

|
0
2
0
0
2
2 2
, com v
0x
= v
0
cos e v
0y
= v
0
sen. b) Encontre o ngulo que a trajetria faz com a horizontal para
qualquer x (tg = dy/dx), c) Encontre x
max
correspondente ao topo da trajetria
(tg = 0). d) Encontre o alcance R, fazendo =








Movimentos bi e tridimensional


S. C. Zilio e V. S. Bagnato Fsica Bsica Mecnica, calor e ondas


50








As leis de Newwton



S. C. Zilio e V. S. Bagnato Mecnica, calor e ondas


49






4.1 Introduo
At o momento estudamos vrios tipos de movimento sem no entanto
nos preocuparmos com suas causas. J sabamos intuitivamente que para se
modificar o movimento de um corpo necessria a ao de um agente
externo. De fato, na ausncia completa de ao externa, o corpo permanece
num estado de movimento constante. A maneira pela qual o agente externo
age sobre o corpo atravs da atuao de uma fora. Portanto, a fora nada
mais do que a quantificao da ao de um corpo sobre outro.
A fora pode ser definida como uma grandeza fsica capaz de alterar o
estado de movimento de um corpo ou a forma deste corpo. O estado de
movimento de um corpo caracterizado pelo seu momentum linear, que
definido como:
v m p
r r
=
de forma que a existncia de uma fora produz alteraes em p
r
.
O comportamento de um corpo quando sujeito a foras externas
regido pelas leis de Newton, expressas como:
Lei I - Todo corpo permanece em repouso ou em movimento retilneo
uniforme, a menos que seja obrigado a modificar seu estado de movimento
pela ao de foras externas.
Lei II - A modificao do movimento proporcional fora atuante, ou
seja, dt / p d F
r
r
= .
Lei III - A toda ao corresponde uma reao igual e oposta ou, as aes
mtuas de dois corpos so sempre dirigidas em sentidos opostos.
4
AS LEIS DE
NEWTON







As leis de Newwton


S. C. Zilio e V. S. Bagnato Mecnica, calor e ondas


50
A primeira lei estabelece justamente o que havamos dito
anteriormente, isto , para modificarmos p
r

(grandeza que quantifica o estado
de movimento do corpo) necessrio um agente externo exercendo uma fora
sobre o corpo. Suponha por exemplo, um cometa movendo-se em movimento
retilneo uniforme. Ele continuar neste estado at chegar nas proximidades de
um planeta, que atravs da fora gravitacional, modificar seu estado de
movimento fazendo com que o momentum p
r
mude em mdulo e direo. Esta
idia que acabamos de apresentar, embora bastante lgica, no o era na poca
de Galileu, pois se acreditava que para manter um corpo em movimento
retilneo uniforme era necessria a ao de agentes externos. O nico estado
natural e espontneo para um corpo era o repouso!
A fora tambm necessria para alterar a forma de um corpo.
Durante a deformao as partculas deste corpo so aceleradas at atingirem
uma nova situao de equilbrio. O equilbrio de um corpo pode ser de tipos
diferentes. Inicialmente, um corpo s estar em equilbrio quando a resultante
das foras agindo sobre ele for nula. O equilbrio dito estvel quando uma
pequena perturbao tira o sistema de equilbrio, mas a vizinhana do corpo
age de forma a restaurar o equilbrio. O equilbrio dito instvel quando uma
pequena perturbao tira o sistema do equilbrio e a vizinhana age no sentido
de amplificar este efeito.
Vamos considerar que a quantidade de matria num determinado
corpo no se modifica. Neste caso, a ao de uma ou mais foras leva a uma
acelerao:
a m dt / v d m F
r r
r
= =


e a constante de proporcionalidade entre fora e acelerao denominada
massa do corpo. A unidade de massa Kg (SI) ou g (CGS) enquanto que a da
acelerao m/s
2
(MKS) ou cm/s
2
(CGS). Portanto, a unidade de fora
definida como: [F] = 1 N = 1 Kg.m/s
2
no Sistema Internacional (SI) ou [F] = 1
dyn = 1 g.cm/s
2
no sistema CGS, sendo portanto, 1 dyn = 10
-5
N.







As leis de Newwton



S. C. Zilio e V. S. Bagnato Mecnica, calor e ondas


51
Quando a massa de um corpo varia, como por exemplo, durante a
exausto de combustvel num foguete, a forma mais geral da segunda lei de
Newton fica:
( )
dt
dm
v
dt
v d
m v m
dt
d
dt
p d
F
r
r
r
r
r
+ = = =
A expresso v m p
r r
= para o momentum de um corpo vlida quando
este tem velocidade bem menor que a velocidade da luz, c, que de
aproximadamente 300.000 km/s. Para velocidades altas (v c),
v ) v ( m v
c / v 1
m
p
2 2
0
r r r
=

=
onde m
0
chamado de massa de repouso e m(v) varia de uma maneira que
corpo torna-se cada vez mais pesado quanto mais se aumenta sua velocidade.
Porm, se v/c << 1, a aproximao m m
0
bastante boa.
Quando um corpo encontra-se prximo superfcie da Terra, esta
exerce sobre ele uma fora que denominada peso, dada por: w = mg e que
est dirigida para o centro da Terra.
A massa de um corpo, como vimos, quantificada atravs da razo
entre a fora e a acelerao, Associado massa, h uma propriedade
importante que denominada inrcia. Imagine uma locomotiva e um carrinho
de beb sobre o cho sem atrito, completamente livres para se moverem. Ao
exercermos uma ao sobre cada um deles (por exemplo, um empurro), o
carrinho comea a andar enquanto que o trem oferecer forte resistncia
mudana de movimento por possuir uma inrcia maior. Corpos com maiores
massas apresentam maior inrcia e, conseqentemente, maior resistncia a
mudanas no seu estado de movimento.
Todos os corpos apresentam a tendncia de permanecer no seu estado
original de movimento quando acionados subitamente por um agente externo.
Uma ilustrao deste fato o que ocorre com os passageiros no interior de um
automvel em movimento retilneo uniforme que freado ou faz uma curva
acentuada. No primeiro caso, a tendncia do passageiro chocar-se contra o







As leis de Newwton


S. C. Zilio e V. S. Bagnato Mecnica, calor e ondas


52
pra-brisa enquanto que no segundo, a tendncia sair pela tangente curva.
Este tipo de comportamento est relacionado com a inrcia do passageiro.
Das trs leis de Newton, a 3
a
aquela que sem dvida exige um maior
esclarecimento. Ela descreve uma propriedade importante das foras: sua
ocorrncia em pares, isto , toda ao corresponde uma reao de mesma
intensidade, porm de sentido oposto. Um fato importante a ser observado
que ao e reao no se cancelam (ou se equilibram) porque agem em corpos
diferentes. Um exemplo disto o de um corpo sobre uma mesa como ilustrado
na Fig. 4.1. O corpo exerce uma fora ' N
r
sobre a mesa e esta responde
exercendo sobre o corpo uma fora '. N N
r r
= N
r
e ' N
r
constituem um par
ao-reao. A Terra exerce sobre o corpo a fora peso w
r
para a qual existe
uma reao ' w
r
exercida do corpo sobre a Terra. w
r
e ' w
r
' constituem outro
par ao-reao porm w
r
e N
r
no constituem par ao-reao. Devido ao
fato do corpo estar em equilbrio, pela 2
a
Lei de Newton, 0 a =
r
e portanto

= 0 F
r r
. Logo:
N w 0 N w
r
r
r
r
= = +
Quando dois corpos isolados constituem um sistema, as nicas foras
existentes so as que constituem o par ao-reao. Neste caso, olhando para o
sistema como um todo, vemos que:









Fig. 4.1 - Foras agindo num corpo sobre uma mesa.
N
r
N
r
' w
r
w
r
corpo
mesa







As leis de Newwton



S. C. Zilio e V. S. Bagnato Mecnica, calor e ondas


53
( ) 0
dt
p d
p p
dt
d
0
dt
p d
dt
p d
0 F F
2 1
2 1
2 1
r
r
r r
r
r r
r r r
= = +
= + = +

e assim conclumos que o momentum total se conserva na ausncia de foras
externas j que
1
F
r
e
2
F
r
constituem foras externas ao sistema. Esta lei de
conservao do momentum de grande importncia no estudo de colises
entre corpos, onde as foras envolvidas so internas ao sistema.
4.2 Referenciais
As grandezas cinemticas s tm sentido fsico quando medidas com
relao a um ponto de referncia. Assim, se considerarmos por exemplo, um
trem movendo-se com velocidade v
0
na direo x > 0 e um homem dentro do
trem movendo-se com velocidade -v
0
(na direo x < 0), observamos que para
uma pessoa parada fora do trem, a velocidade do homem ser nula. Com este
exemplo vemos claramente que o conceito de movimento est intrinsecamente
ligado ao de referencial.
Consideremos um sistema de coordenadas O (x, y, z) fixo no espao,
no qual a posio de um corpo especificada pelo vetor posio:
k

z j

y i

x r + + =
r

a partir do qual podemos encontrar a velocidade e a acelerao da maneira
tradicional:
k

z j

y i

x a
k

z j

y i

x v
& & & & & &
r
& & &
r
+ + =
+ + =

Consideremos a seguir um segundo sistema de coordenadas O (x, y, z)
movendo-se com velocidade k

' v j

' v i

' v ' v
oz oy ox 0
+ + =
r
com relao ao
referencial fixo, conforme mostra a Fig. 4.2. O vetor R
r
descreve a posio do
ponto O com relao ao ponto O. Se este referencial estiver uniformemente
acelerado, R
r
(t) ser dado por:







As leis de Newwton


S. C. Zilio e V. S. Bagnato Mecnica, calor e ondas


54









Fig. 4.2 - Referenciais em movimento relativo.
2
R 0 0
t a t v R ) t ( R
2
1
r r
r r
+ + =
Por outro lado, olhando para a figura vemos que a adio geomtrica dos
vetores nos fornece: R r r ou r R r
r
r r r
r
r
= + = , onde r
r
descreve a posio
do corpo visto por um observador solidrio ao referencial mvel. Este
observador ver a velocidade do corpo dada por:
t a v v R r r v
R 0
r r r
&
r
&
r
&
r r
= = =
que a velocidade que o corpo possui no sistema de coordenadas O menos a
velocidade de O com relao a O. A acelerao por sua vez :
R
a a a
r r r
=
que a acelerao no sistema fixo menos a acelerao relativa entro os dois
referenciais. No caso particular em que o sistema mvel O' no est acelerado
( 0 a
R
r
r
= ) temos a a
r r
= , isto , a acelerao a mesma nos dois referenciais.
Referenciais deste. tipo, onde a lei de Newton tem a mesma forma
( a m a m F
r r
r
= = ) so chamados de referenciais inerciais.
4.3 Aplicaes das leis de Newton
Como vimos, as leis de Newton so as leis bsicas da Mecnica
Clssica. Em principio, qualquer problema de dinmica pode ser resolvido
x
y
z
x
y
z
R
r
r
r
r
r
' v
r
O
O







As leis de Newwton



S. C. Zilio e V. S. Bagnato Mecnica, calor e ondas


55
atravs de sua aplicao. Passaremos agora a analisar uma srie de exemplos
que ilustram tais leis.
De modo geral, os problemas envolvendo foras podem ser
classificados em duas categorias. Na primeira, conhecemos as foras que
agem sobre o corpo e queremos encontrar seu efeito, expresso atravs de
mudanas na velocidade e posio. Na segunda categoria, conhecemos o
movimento do corpo e a partir disto queremos determinar o conjunto de foras
agindo sobre ele. A soluo de um problema pode ser encontrada atravs de
urna sequncia natural de anlises. Primeiramente, o problema deve estar
claramente colocado e se ele apresentar vrias partes, cada uma delas deve ser
analisada antes de se considerar o sistema como um todo. Sempre que houver
contato entre corpos, lembre-se que ao e reao agem em corpos diferentes.
a) Plano inclinado sem atrito
Queremos encontrar o movimento de um corpo colocado sobre um
plano com ngulo de inclinao como mostrado na Fig. 4.3. As foras
agindo sobre ele so: o peso w
r
, que dirigido para baixo e a fora de reao
N
r
, que normal superfcie.







Fig. 4.3 - Plano inclinado sem atrito.
Como o corpo no pode penetrar no plano inclinado, conclumos que
o movimento s deve ocorrer na direo paralela a ele. Isto implica em que a
fora resultante na direo perpendicular ao plano nula e assim:


x
y
N
r
W
r







As leis de Newwton


S. C. Zilio e V. S. Bagnato Mecnica, calor e ondas


56

=
=
x x
y
Ma F
0 F

de onde obtemos:
= = + cos Mg N 0 N cos Mg
= = sen g a Ma sen Mg
x x

e como a
x
constante, o movimento paralelo ao plano do tipo
uniformemente acelerado j visto anteriormente.
b) Corpo suspenso por cordas
Imagine um corpo suspenso por duas cordas conforme mostra a Fig.
4.4. As cordas ficaro sujeitas s tenses
2 1
T e T
r r

dirigidas ao longo de seu
comprimento e, portanto, agindo sobre o corpo. Como este est em equilbrio,
a soma total das foras agindo sobre ele nula, de forma que:
= =

0 F F
y x

0 cos T cos T
2 2 1 1
=
0 Mg sen T sen T
2 2 1 1
= +









Fig. 4.4 - Corpo suspenso por cordas.
Destas duas equaes tiramos
2 1
T e T
r r
:

2
M
T
1 T
2
Mg
x
y







As leis de Newwton



S. C. Zilio e V. S. Bagnato Mecnica, calor e ondas


57
( )
2 1
1
2 1 2
1
1
sen
cos Mg
sen sen cos
cos Mg
T
+

=
+

=
( )
2 1
2
2 1 2
2
21
sen
cos Mg
sen sen cos
cos Mg
T
+

=
+

=
No caso da corda resistir somente a uma tenso mxima T
max
, podemos
analisar se T
1
ou T
2
ultrapassa tal limite.
Em dinmica, os problemas envolvendo cordas e fios so bastante
freqentes e, portanto, vamos tecer algumas consideraes a este respeito.
Vamos considerar uma corda de massa M
c
e comprimento L que sustenta um
corpo de massa M ao longo da vertical (ver Fig.4.5). Queremos calcular a
tenso na corda em toda a extenso de seu comprimento.






Fig. 4.5 - Corpo suspenso por uma corda com massa.
Se isolarmos o ponto de contato entre o corpo e a corda temos T =
Mg. Por outro lado, se tomarmos um ponto a uma altura x sobre o corpo, a
massa total abaixo dele M+(M
C
/L) x e para que a corda esteja em equilbrio,
a tenso dever ser:
x g
L
M
Mg ) x ( T
c
+ =
Isto mostra que medida que subimos pela corda seu nvel de tenso
aumenta e no ponto de contato com o teto T = (M + M
c
) g, como esperado. No
entanto, se a massa da corda for desprezvel, a tenso a mesma em cada
ponto ao longo de seu comprimento e ela funciona apenas como transmissora
de esforos.
M
Mg
T
M
x







As leis de Newwton


S. C. Zilio e V. S. Bagnato Mecnica, calor e ondas


58
c) Dois corpos ligados por uma corda
Considere dois corpos com massas M
1
e M
2
ligados por uma corda
sem massa e podendo deslizar sobre uma mesa sem atrito. Existe ainda uma
fora F
r
agindo sobre M
2
,

como indicado na Fig. 4.6. Queremos encontrar a
tenso na corda e a acelerao do sistema. Como a corda tem massa
desprezvel, ela simplesmente transmite a fora. Isolando os corpos, temos:
T = M
1
a
1

F T = M
2
a
2






Fig. 4.6 - Corpos ligados p uma corda.
O sistema est vinculado de forma tal que os corpos so obrigados a
andar juntos e assim a
1
= a
2
= a. Logo:
F M
1
a = M
2
a a =
2 1
M M
F
+

2 1
1
1
M M
F M
a M T
+
= =
d) Corpos em contato
Uma fora F
r
aplicada sobre um corpo de massa M
1
que est em
contato com outro corpo de massa M
2
, como mostra a Fig. 4.7. Ambos esto
colocados sobre uma mesa sem atrito e a questo que se pretende responder
sobre a fora que transmitida ao corpo 2. Como os corpos se movem juntos,
a acelerao ser a mesma para os dois e ento podemos escrever:
( )
2 1
2 1
M M
F
a a M M F
+
= + =
T

T

M
1
M
2
F
r







As leis de Newwton



S. C. Zilio e V. S. Bagnato Mecnica, calor e ondas


59
Voltamos agora a analisar o corpo 2. Chamando T a fora que 1 faz
sobre 2, temos:
2 1
2
2
M M
F M
a M T
+
= =
e assim vemos que este resultado similar ao do caso em que os dois corpos
esto ligados pela corda.




Fig. 4.7 - Corpos em contato.
e) n corpos conectados por cordas
Temos n corpos conectados por cordas conforme mostra a Fig. 4.8 e
queremos calcular a tenso na corda que conecta um par qualquer destes
corpos. Como os corpos possuem mesma massa M e se deslocam juntos
quando submetidos ao da fora F, podemos escrever que a acelerao do
sistema a fora dividida pela massa total, isto , a = F/(nM). A fora T
i
por
sua vez movimenta todos os corpos a sua esquerda, desde i at n. O nmero
destes corpos n - i +1 e portanto:
( ) ( )
( )
F
n
1 i n
n
MF
1 i n Ma 1 i n T
i
+
= |

\
|

+ = + =

Fig. 4.8 - Corpos conectados por cordas.
f) Sistema com polias: mquina de Atwood
Vamos considerar inicialmente uma corda ao redor de uma polia sem
atrito e sem massa como indica a Fig. 4.9(a). Como a corda possui massa
desprezvel, ela simplesmente transmite a tenso e portanto, F
1
= F
2
= F.
M
1
M
2
F
r
F
r
n
n-1 n-2 3
2 1







As leis de Newwton


S. C. Zilio e V. S. Bagnato Mecnica, calor e ondas


60








Fig. 4.9 - Corda ao redor de uma polia (a) e pequena poro da corda (b).
Desta forma, como se a polia simplesmente mudasse a direo da fora.
Podemos calcular a fora normal polia da seguinte maneira. Tomemos uma
pequena poro de corda definida pelo ngulo , como mostra a Fig. 4.9(b).
Projetando as foras F na direo radial temos:
( ) 2 sen F 2 dN = F
enquanto que a componente tangencial se anula. Para encontrarmos a fora
normal total (somada em mdulo) devemos integrar no ngulo:

= =
0
F d F N (em mdulo)
A mquina de Atwood um dos exemplos mais simples envolvendo
polias, onde duas massas, M
1
e M
2
so interligadas atravs de uma corda sem
massa, como mostrado na Fig. 4.10. Chamando a tenso na corda de T, temos:
T - M
1
g = M
2
a
-T+M
2
g = M
2
a
de onde tiramos
( )
( )
g
M M
M M
a
1 2
1 2
+

=
A tenso T dada por:
( )
( )
g
M M
M M
M g M a M g M T
1 2
1 2
1 1 1 1
+

+ = + =
2

F
r
F
r
N
r
1
F
r
2

F
r
F
r
2

2
F
r
(a) (b)







As leis de Newwton



S. C. Zilio e V. S. Bagnato Mecnica, calor e ondas


61
e a fora exercida sobre o suporte da polia :
( )
2 1
2 1
M M
g M M 4
T 2
+
+
=










Fig. 4.10 - Mquina de Atwood.
g) Bloco sobre a mesa puxado por corpo na vertical
A Fig. 4.11 mostra um bloco de massa M
1
sobre uma mesa sem atrito,
puxado por outro bloco de massa M
2
sob a ao da gravidade. Isolando o
bloco 1 temos:
a M T
1
=
enquanto que ao isolar o bloco 2 obtemos:
a M T g M
2 2
=
Combinando estas duas equaes obtemos a acelerao do sistema como:
2 1
2
M M
g M
a
+
=




M
2
g M
1
g
M
1
M
2

2T

a
r
T
T T
T







As leis de Newwton


S. C. Zilio e V. S. Bagnato Mecnica, calor e ondas


62








Fig. 4.11 - Bloco sobre a mesa e corpo na vertical.
h) Peso aparente de um objeto num elevador acelerado
Vamos imaginar um objeto no interior de um elevador acelerado como
indica a Fig. 4.12. Qual seria seu peso aparente se ele estivesse sendo medido
por urna balana? O objeto pressiona a balana com urna fora N, que o
prprio peso aparente medido por ela. Pela 3
a
lei de Newton, a balana produz
uma fora N, s que dirigida para cima. O objeto anda junto com o elevador
de forma que a 2
a
lei de Newton fica:
N Mg = Ma N = M (g + a)
Se o elevador estiver acelerado para cima, o peso aparente maior que
Mg,enquanto que se a acelerao for para baixo, o peso aparente ser menor
que Mg.








Fig. 4.12 - Objeto num elevador acelerado.
M
1
M
2
g
T

T

a
r
M
2
a
r
M
Mg
N







As leis de Newwton



S. C. Zilio e V. S. Bagnato Mecnica, calor e ondas


63
4.4 Movimento circular
Como vimos anteriormente, quando um corpo encontra-se em
movimento circular, existe uma acelerao radial, denominada centrpeta, que
dada por r / v a
2
c
= , onde r o raio do movimento circular e v a
velocidade tangencial. claro que a velocidade tangencial pode variar e,
portanto, existir uma acelerao tangencial. Vamos a seguir estudar vrios
casos deste tipo de movimento.
a) Pndulo cnico
Considere um pndulo de comprimento L, formando um ngulo com
a vertical e descrevendo um crculo de raio R no plano horizontal, como indica
a Fig. 4.13. Qual a velocidade tangencial da massa M? Para responder esta
pergunta, vamos analisar as foras agindo sobre ela.







Fig. 4.13 - Pndulo cnico.
Na direo radial temos T sen = Mv
2
/R, enquanto que na direo
vertical, T cos = Mg. Dividindo uma equao pela outra obtemos:
Rg / v tg
2
=
ou ento:
2 2
2
2 2
2
R L
g R
R L
R
Rg v

=
Suponha que o fio se rompa com uma tenso T
0
. Para que velocidade
isto ocorrer?
R
L
M

Mg
Tcos
Tsen







As leis de Newwton


S. C. Zilio e V. S. Bagnato Mecnica, calor e ondas


64
b) Movimento circular vertical
Considere um corpo de massa M preso a uma corda de comprimento
R sem massa, posto para rodar em movimento circular no plano vertical, como
mostrado na Fig. 4.14. A posio do corpo especificada pelo ngulo e tal
que no ponto mximo (1) = 0 e no ponto mnimo (2) = . Inicialmente
estamos interessados em determinar a tenso na corda quando o corpo se
movimenta com velocidade constante. Na direo radial temos:
T + Mg cos = Mv
2
/R T = cos Mg
R
Mv
2








Fig. 4.14 - Movimento circular vertical.
Deste resultado vemos que T = Mv
2
/R - Mg a tenso mnima para
= 0
o
e T = Mv
2
/R + Mg a tenso mxima para = . A Fig. 4.15 mostra um
grfico completo de T contra . A velocidade mnima capaz de manter o
movimento circular ocorre quando T = 0 e vale gR v
min
= . Para
velocidades inferiores a esta, no possvel haver movimento circular na
vertical.






Fig. 4.15 - Tenso na corda em funo do ngulo .

T()

2
Mg
R
Mv
2
+
Mg
R
Mv
2

M

Mg
T
R
1
2







As leis de Newwton



S. C. Zilio e V. S. Bagnato Mecnica, calor e ondas


65
c) Pndulo simples
O movimento pendular um dos movimentos mais estudados em
Mecnica Clssica, ao lado do movimento harmnico do sistema massa-mola.
Considere o pndulo da Fig. 4.16 deslocado de um certo ngulo . Usando a 2
a

lei de Newton nas direes radial e tangencial temos respectivamente:
t
2
Ma sen Mg
L / Mv cos Mg T
=
=









Fig. 4.16 - Pndulo simples.
Vamos supor que a condio inicial do movimento seja =
0
e v = 0, de
forma que T
0
= Mg cos
0
. Como ( )( ) dt / d d / dv dt / dv a
t
= = ( ) L / v d / dv =
temos para a direo tangencial:
dv v d sen gL
L
v
d
dv
M sen Mg
=

=

que pode ser integrado, resultando em:
2
v
0
v dv v d sen gL
0 2
1

= =
A realizao desta integral simples e leva a:
( )
2
0
v cos cos gL
2
1
=
L
Mg

T








As leis de Newwton


S. C. Zilio e V. S. Bagnato Mecnica, calor e ondas


66
Logo:
v
2
/L = - 2g(cos
0
- cos)
e assim, a tenso no fio varia com de acordo com:
T = Mg(3cos - 2cos
0
)
d) Corda girante
Imagine uma corda de massa M e comprimento L colocada para girar
num plano horizontal (sobre uma mesa sem atrito) com velocidade angular ,
conforme mostra a Fig. 4.17. Queremos encontrar a tenso na corda a uma
distncia r do ponto de fixao. Para isto vamos considerar um elemento de
comprimento r, como mostrado na figura, cuja massa ( ) r L / M m = .
Este elemento est sujeito s tenses T(r) e T(r +r). Pela 2
a
lei de Newton
temos:
( ) ( ) r r r m r r T r T
2 2
L
M
= = +





Fig. 4.17 - Corda girando sobre uma mesa sem atrito.
Podemos re-escrever esta expresso como:
( ) ( )
L
r M
r
r T r r T
2

+

No limite em que r tende a zero ficamos com:
( ) ( )
L
r M
dr
dT
r
r T r r T
lim
2
0 r

= =
(

+


A seguir, vamos integrar entre os pontos 0 e r:
r

r
T(r)
T(r+r)
r
m







As leis de Newwton



S. C. Zilio e V. S. Bagnato Mecnica, calor e ondas


67
( )

=
R T
T 0
2
0
r
dr r
L
M
dT
( ) ( )
L 2
r M
T r T
L 2
r M
T r T
2 2
0
2 2
0

=

=
Para encontrar o valor de T
0
, notamos que T = 0 para r = L (a corda
acaba neste ponto). Logo,
2
L M
T
2
L M
T 0
2
0
2
0

=

=
e conseqentemente:
( )
2 2
2
r L
L 2
M
) r ( T

=
A Fig. 4.18 mostra o grfico de T(r).








Fig. 4.18 Tenso na corda como funo da posio radial.
4.5 Fora retardadora proporcional velocidade
Quando um corpo move-se no interior de um fluido (gs ou liquido),
age sobre ele uma fora proporcional velocidade, porm na direo oposta
ao movimento. Esta forca denominada viscosa. Assim, vamos imaginar um
corpo com velocidade inicial v
0
, movendo-se num meio viscoso. Pela 2
a
lei de
Newton temos:
bv
dt
dv
m ma = =
2
L M
2

r
T(r)
0 L







As leis de Newwton


S. C. Zilio e V. S. Bagnato Mecnica, calor e ondas


68
Esta equao, chamada de equao diferencial, pode ser resolvida se
isolarmos v e t e a seguir integrarmos:

= =
t
0
v
v
dt
m
b
v
dv
dt
m
b
v
dv
0

Logo:
m
bt
v
v
n v n v n
0
0
=
|
|

\
|
= l l l
( )
)
`

=
m
bt
exp v t v
0

de modo que a velocidade do corpo decresce exponencialmente como mostra a
Fig. 4.19.






Fig. 4.19 - Velocidade de um corpo jogado com velocidade v
0
num meio viscoso.
Vamos imaginar agora um corpo num meio viscoso caindo sob a ao
da gravidade. O balano das foras leva seguinte equao de movimento:
dt
dv
m bv mg =
A velocidade vai aumentando at que a fora gravitacional equilibrada pela
fora viscosa. A partir deste ponto teremos dv/dt = 0 e conseqentemente no
haver mais mudanas de velocidade. Dizemos ento que o corpo atingiu sua
velocidade terminal v
T
que dada por:
b
mg
v bv mg
T T
= =
t
v(t)
v
0







As leis de Newwton



S. C. Zilio e V. S. Bagnato Mecnica, calor e ondas


69
Para resolvermos a equao de movimento vamos supor que o corpo partiu do
repouso. Isolando v e t temos:

=

t
0
v
0
dt
m
bv
g
dv
dt
m
bv
g
dv

Fazendo a substituio: du
b
m
dv u
m
bv
g = =

=
t
0
m / bv g
g
dt
u
du
b
m

{ } m / bt exp
g
m / bv g
t
m
b
g
m / bv g
n =

= |

\
|
l
A velocidade do corpo cresce como mostra a Fig. 4.20.






Fig. 4.20 - Velocidade de um corpo acelerado num meio viscoso.
4.6 Foras observadas na natureza
As foras existentes entre as partes de um sistema so oriundas de
interaes fundamentais tais como: foras gravitacionais, foras
eletromagnticas e foras nucleares (fortes e fracas). Estas foras,
responsveis pela existncia da matria, sero vistas em vrios cursos futuros.
Ns vamos aqui abordar apenas os efeitos macroscpicos destas foras.
a) Forcas elsticas: lei de Hooke
Denominamos de elsticos aqueles corpos que ao sofrerem
deformaes quando sujeitos a esforos, tm a propriedade de recuperarem
t
v(t)
v
t







As leis de Newwton


S. C. Zilio e V. S. Bagnato Mecnica, calor e ondas


70
sua forma original quando tais esforos so removidos. Vamos imaginar a
seguinte experincia: consideremos uma mola com uma das extremidades fixa
na parede e com uma fora F aplicada na outra, como ilustra a Fig. 4.21.




Fig. 4.21 - Mola tracionada.
Antes da aplicao da fora F, a mola tem um comprimento livre x
0
.
Aps a aplicao desta, ela distende-se para um novo comprimento x, tal que a
deformao dada por
0
x x x = . Se formos aumentando gradativamente a
fora F e medindo a deformao x associada, verificaremos a existncia de
dois tipos de comportamento. Inicialmente, a fora e a deformao so
diretamente proporcionais, mas conforme F aumenta isto deixa de ser verdade.
Num grfico de F contra x, mostrado na Fig. 4.22, a regio de linearidade vai
do ponto 0 at o ponto 1. Neste regime, denominado de elstico, vale a
relao:
F = k x
onde k (inclinao da reta) chamada de constante de mola e a expresso
acima, conhecida como lei de Hooke. Se olharmos microscopicamente para o
material, neste regime os vrios planos de tomos sofrem deslocamento
relativo entre si, mas um determinado tomo permanece sempre ligado sua
posio original.







Fig. 4.22 - Deformao de uma mola real sujeita a uma fora F.
F
k
x
F
1
2
0







As leis de Newwton



S. C. Zilio e V. S. Bagnato Mecnica, calor e ondas


71
O regime que vai de 1 a 2 denominado plstico e a deformao
causada nesta regio permanente. Microscopicamente, os planos atmicos
pulam de uma posio para a seguinte, gerando deformaes permanentes no
material. Ao atingir o ponto 2, o material no resiste mais ao esforo e rompe-
se.








Fig. 4.23 - Descrio microscpica dos regimes elstico e plstico.
b) Forcas de contato e atrito
Quando duas superfcies slidas so colocadas em contato, existe uma
resistncia ao deslocamento relativo destas superfcies que denominada de
atrito. O atrito tem sua origem no fato de que as superfcies no so
microscopicamente perfeitas, de maneira a se estabelecerem vrios pontos de
contato que dificultam o movimento relativo entre as superfcies, como mostra
a Fig. 4.24.
Fig. 4.24 - Superfcies reais em contato.
Devido a esta natureza da fora de atrito, esperamos que quanto mais
forte uma superfcie for pressionada contra a outra, maior deve ser a
resistncia ao deslizamento, ou seja, maior o atrito. Logo, a fora de atrito
elstico
plstico







As leis de Newwton


S. C. Zilio e V. S. Bagnato Mecnica, calor e ondas


72
proporcional fora normal entre as duas superfcies: F
at
N. Outro fator que
influencia a intensidade da fora de atrito a qualidade da superfcie: se esta
for bem polida, o atrito ser menor. Finalmente, o tipo de material usado na
confeco de corpo tambm importante na determinao de F
at
: se o material
for macio, a tendncia que ele se amolde outra superfcie e isto dificulta
o deslizamento. A qualidade da superfcie e a dureza do material especificam
o coeficiente de atrito que definiremos a seguir.
Vamos imaginar um experimento onde uma fora F varivel
aplicada sobre um corpo de massa M, inicialmente em repouso sobre uma
superfcie spera, como esquematizado na Fig. 4.25. Se F relativamente
pequena, o corpo continua em repouso e neste caso, F = F
at
. Note que se F = 0,
F
at
= 0, indicando que a fora de atrito s existe se houver tendncia ao
deslizamento. Se continuarmos a aumentar F, esta atinge um valor mximo
para o qual o corpo se encontra iminncia de deslizar. Neste ponto define-se o
coeficiente de atrito esttico como F
max
=
e
N. A partir da, o corpo entra em
movimento e qualquer incremento em F contribui exclusivamente para
acelerar o corpo, como mostra a Fig. 4.26. Na situao de movimento, a fora
de atrito F
at
=
d
N, onde
d
chamado de coeficiente de atrito dinmico.
Assim, no regime esttico F
at

e
N e no regime dinmico F
at
=
d
N, sendo
d

<
e
(verificado experimentalmente).





Fig. 4.25 - Corpo puxado sobre uma superfcie com atrito.
Como exemplo do clculo de fora de atrito, tomemos um corpo de
massa M sobre um plano inclinado, como mostra a Fig. 4.27. Da. 2
a
lei de
Newton temos:
N Mg cos = 0
e
Mg sen - F
at
= Ma
M
F
r
at
F
r







As leis de Newwton



S. C. Zilio e V. S. Bagnato Mecnica, calor e ondas


73









Fig. 4.26 - Variao da fora de atrito com a fora externa aplicada.
No caso do corpo estar na iminncia de deslizamento, a = 0 e F
at
=

e
N. Desta forma,
e
= tg.







Fig. 4.27 - Corpo sobre um plano inclinado com atrito.
Como segundo exemplo, vamos analisar um rotor no parque de
diverses, mostrado na Fig. 4.28. Este rotor constitudo de um cilindro de
raio R, com fundo, colocado para rodar com velocidade angular , tendo
vrias pessoas no seu interior. Assim que o cilindro atinge a rotao mxima,
o fundo retirado e as pessoas so mantidas no seu interior somente pelo
atrito do contato com a parede. Sendo o coeficiente de atrito esttico, g a
acelerao da gravidade local, queremos encontrar a mnima velocidade
angular capaz de manter a pessoa equilibrada. Neste caso, a fora normal
dada pela fora centrpeta e ento,
45
o
F
F
at

e
N

d
N
iminncia de deslizamento
deslizamento

N
r
at
F
r
Mg
a
r








As leis de Newwton


S. C. Zilio e V. S. Bagnato Mecnica, calor e ondas


74

2 F
2

2

T
T+T
N
N
R
g
R M N Mg
e
min
2
e e

= = =









Fig. 4.28 - Rotor com atrito num parque de diverses.
Como exemplo final desta seo, vamos tratar o caso de uma polia
com atrito. Como j discutimos anteriormente, uma polia ideal (sem atrito)
apenas modifica a direo de uma fora sem modificar seu valor. Queremos
agora analisar como a presena do atrito modifica F
1
comparada com F
2
. Para
isto, vamos tomar um elemento da polia mostrada na Fig. 4.29 e verificar as
foras sobre ele.







Fig. 4.29 - Corda em polia com atrito.
Na direo x:
( )
|

\
|
+
|

\
|
+ =

2 2
sen T sen T T N
N F
e at
=
N = M
2
R
Mg

1 F







As leis de Newwton



S. C. Zilio e V. S. Bagnato Mecnica, calor e ondas


75
Como pequeno, 1 cos e sen
2 2 2


e assim,
( )

+
|

\
|
=
|

\
|
+
|

\
|
+ =

T
2
T
T 2 T T T N
2 2 2

Na direo y:
( ) N cos T cos T T
2 2
+
|

\
|
=
|

\
|
+


T
T
T N T =

= =
no limite em que 0, temos lim
0
( ) T d / dT / T = = e
portanto:


= =
0
F
F
d
T
dT
d
T
dT
1
2

{ } = = |

\
|
exp F F
F
F
n
2 1
2
1
l
4.7 Foras inerciais
Quando a observao de um movimento feita de um referencial no
inercial (acelerado), as leis de Newton deixam de ser vlidas, isto , a fora
sobre o corpo no obedece a relao dt / v md F
r
r
= . Como a lei de fora neste
caso fica bastante difcil de ser escrita, principalmente porque ela depende da
posio momentnea do corpo, ns introduziremos uma fora extra no
problema, que equivalente ao efeito produzido pelo fato do referencial ser
no inercial. Com a adio destas foras fictcias, chamadas de foras
inerciais, a lei de Newton passa a ser novamente vlida. Note que as foras
inerciais simulam o efeito de uma fora real, porm elas no so exercidas por
nenhum elemento do sistema. Vamos ilustrar o uso das foras inerciais atravs
dos vrios exemplos que seguem.
a) Vago acelerado







As leis de Newwton


S. C. Zilio e V. S. Bagnato Mecnica, calor e ondas


76
Vamos considerar um vago acelerado como mostrado na Fig. 4.30
dentro do qual encontra-se um observador. Se deixarmos um corpo cair a
partir do repouso, para um observador externo, a trajetria tal que a nica
fora agindo sobre o corpo g M
r
.
Para um observador no interior do vago acelerado, a trajetria do
corpo tal que indica a existncia de uma fora a M
r
, de forma que a fora
total vista por ele :
a M g M F
r r
r
=
onde o termo entre a M
r
a fora inercial.








Fig. 4.30 - Corpo em queda livre visto por um observador acelerado.
Por outro lado, se o corpo estiver preso por uma corda no teto do
vago, um observador externo ver o corpo acelerado tal que:
a M g M T
r r
r
= + (observador em repouso)
Para um observador no interior do vago, o corpo no est acelerado
e, portanto, para ele, a equao de foras :
0 a M g M T
r
r r
r
= + (observador acelerado)
b) Fora centrfuga
Consideremos uma plataforma girando com velocidade angular e
sobre ela um corpo preso ao centro por uma haste sem massa, como mostrado
na Fig. 4.31. Para um observador externo plataforma, a nica fora agindo
a M
r

g M
r
a
r







As leis de Newwton



S. C. Zilio e V. S. Bagnato Mecnica, calor e ondas


77
sobre o corpo a fora centrpeta r M F
2
= , que mantm o corpo na sua
trajetria circular. Para este observador, a 2
a
lei de Newton vale na sua forma
usual:
a M r M F
2
r r
r
= =
Para um observador sobre a plataforma, o corpo est em repouso
( 0 a
r
r
= ), porm a haste continua tensionada por um valor que pode ser medido
com um dinammetro. Para ele, deve ento existir uma fora contrria da
haste que mantenha o equilbrio do corpo. Esta fora tambm vale r M
2
,
porm dirigida para fora do crculo. Ela chamada de fora centrfuga e s
existe no referencial no inercial.






Fig. 4.31 - Corpo solidrio a uma plataforma rodando com velocidade .
c) Fora de Coriolis
Um segundo tipo de fora inicial existente em referencial girante a
fora de Coriolis, que depende da velocidade e perpendicular a ela quando
medida no referencial girante. Consideremos dois observadores, um no centro
e o outro na borda de uma plataforma girante, como na Fig. 4.32. Num
determinado instante, o observador do centro (A) arremessa um corpo com
velocidade v
r
para o observador da borda (B).
Quando o corpo chega na borda, o observador B j deslocou-se de um
ngulo e para ele, o corpo foi submetido a uma fora que se desviou para a
esquerda. O segmento de arco descrito pelo observador B, localizado a uma
distncia r do centro s = r = rt. Por outro lado, o corpo anda uma
distncia r com velocidade constante v e portanto r = vt. Conseqentemente, s

M
r
r







As leis de Newwton


S. C. Zilio e V. S. Bagnato Mecnica, calor e ondas


78
= v t
2
. Para o observador B, este segmento de arco consequncia da
acelerao provocada pela fora de Coriolis:
( )
2
c
2
t a t v 2 s
2
1
2
1
= =
ou ento: F
c
= 2mv, perpendicular velocidade. Esta fora tem direo
tangencial e o sentido oposto ao da rotao do referencial.







Fig. 4.32 - Observadores numa plataforma girante.
As foras inerciais em referenciais girantes so de extrema
importncia devido ao fato que a Terra um referencial deste tipo. Estas
foras podem ser escritas em termos de produtos vetoriais se considerarmos o
vetor
r
como sendo perpendicular plataforma girante.
( ) r m F
centrfuga
r r r
r
= ( ) ( ) r m . r m r . m
2
r r r r r r r
= + =
v m 2 F
Coriolis
r r
r
=
onde v
r
a velocidade no referencial girante.
Como exemplo do efeito da fora de Coriolis, vamos analisar o caso
de um corpo que cai de uma altura h sobre a superfcie da Terra, na linha do
Equador. Na ausncia de rotao, o corpo cairia exatamente na direo radial.
Devido rotao da Terra, a fora de Coriolis produzir uma pequena
deflexo que queremos calcular. Vamos desprezar a fora centrfuga supondo
que ela j est includa em g
r
. Vamos fazer um clculo simplificado para
determinar a deflexo x. Suporemos v = gt radial muito maior que a
velocidade produzida pela fora de Coriolis.
v
r
A
B
A
B

v
r
r
r
s







As leis de Newwton



S. C. Zilio e V. S. Bagnato Mecnica, calor e ondas


79
= = = =
2
c
c
c
t g
dt
dx
v t g 2
dt
dv
a
2
gt x
3
1
=
Como o tempo de queda
g
h 2
t = temos
2
3
g
h 2
3
g
x
|

\
|
= . Usando

s
rad
10 3 . 7
3600 24
2 5
=


=
e h = 100 m obtemos x 2 cm.
Exerccios
1 - Encontre o ngulo da Fig. 4.33 tal que o sistema permanea em repouso.
Despreze o atrito.
2 - Encontre a razo entre as massas M
1
e M
2
tal que o sistema permanea em
repouso na Fig. 4.34. Despreze o atrito.






Fig. 4.33 Fig. 4.34
3 - Encontre a acelerao do corpo de 2 Kg da Fig. 4.35.
4 - Encontre a massa do corpo A tal que a acelerao do corpo B da Fig. 4.36
nula.






Fig. 4.35 Fig. 4.36

2 Kg
1 Kg
30
o
M
1
M
2
1 Kg
2 Kg
5 Kg
A 2 Kg
B
16 Kg
fixo
60
o







As leis de Newwton


S. C. Zilio e V. S. Bagnato Mecnica, calor e ondas


80
5 - No sistema da Fig. 4.37 o corpo A desliza sobre uma superfcie com
coeficiente de atrito . As cordas e polias no tm massa.
a) encontre as aceleraes dos blocos A e B;
b) encontre a tenso na corda ligada ao corpo A.












Fig. 4.37
6 - Dado o ngulo de um plano inclinado sem atrito, qual deve ser a
acelerao a
R
tal que o bloco de massa m mostrado na Fig. 4.38 no
deslize?





Fig. 4.38
7 - Se o plano inclinado do problema anterior tiver um coeficiente de atrito ,
qual so as aceleraes mxima e mnima tal que o bloco no deslize?
8 - Uma corda de comprimento L e densidade linear de massa passa por uma
polia sem atrito. Ela solta do repouso, estando um comprimento x
pendente de um lado e L-x do outro.
A
B
M
1
M
2


m
R
a
r







As leis de Newwton



S. C. Zilio e V. S. Bagnato Mecnica, calor e ondas


81
a) determine a acelerao como funo de x;
b) para que situao a acelerao nula?
9 - a) O sistema da Fig. 4.39 livre de atrito. Determine o valor da fora F tal
que o corpo A no desa nem suba.
b) Se houver um atrito esttico entre as superfcies dos blocos, quais os
valores de foras mxima e mnima tal que o corpo A no desa nem
suba?






Fig. 4.39
10 - Um corpo com velocidade inicial v
0
penetra num meio que produz uma
fora viscosa v b F = . Determine a mxima distncia que o corpo
penetra neste meio.
11 - No sistema mostrado na Fig. 4.40 encontre: a) a acelerao do conjunto e
b) a fora na corda, no ponto A.
12 - O sistema mostrado na Fig. 4.41 usa polias sem massa. Encontre as
aceleraes de cada bloco e a tenso na corda.







Fig. 4.40 Fig. 4.41

M

M

M

A
F
r
1
1 Kg
polia sem atrito
A
3 Kg
M
1
M
M
2







As leis de Newwton


S. C. Zilio e V. S. Bagnato Mecnica, calor e ondas


82
13 - No sistema mostrado na Fig. 4.42, o bloco em contato com a superfcie
horizontal sem atrito est sujeito a uma fora F. Existe um atrito esttico
entre este bloco e o bloco A de tal maneira que no existe movimento
relativo entre os trs blocos que compem o sistema. Calcule: a) o ngulo
, b) a tenso na corda e c) mnimo.
14 - N corpos ligados entre si atravs de cordas sem massa so puxados em
uma rampa por meio de uma fora F. Calcule a tenso na corda ligada ao
i-simo corpo.
15 - Considere o pndulo cnico mostrado na Fig. 4.43, onde a corda que liga
a massa M ao ponto O no tem massa.
a) encontre o ngulo como funo da velocidade da massa M
b) encontre a tenso da corda no ponto O








Fig. 4.42 Fig. 4.43
16 - Um corpo de massa M encontra-se pendurado atravs de uma corda ideal
sobre um bloco triangular de ngulo , conforme mostra a Fig. 4.44. No
existindo atrito entre os blocos, pergunta-se qual a acelerao mxima
que pode ser dada ao sistema tal que o corpo M permanea em contato
com o bloco triangular. Neste caso, qual a tenso na corda? Se o
sistema estiver se deslocando com velocidade constante, qual o valor da
tenso na corda e da normal?
17 Um bloco de massa M repousa sobre uma mesa com coeficiente de
atrito esttico
e
. Uma fora F aplicada ao bloco de maneira a
formar um ngulo com a horizontal, como mostra a Fig. 4.45.
L
M

0

= 0
F
r
M
M
M
A
0







As leis de Newwton



S. C. Zilio e V. S. Bagnato Mecnica, calor e ondas


83
Supondo que o bloco esteja sempre na iminncia de deslizar, a)
qual o ngulo
0
que permite que a fora aplicada seja mnima? e
b) neste caso, qual ser o valor da fora F
min
?






Fig. 4.44 Fig. 4.45
18 Um bloco de massa M
1
encontra-se sobre outro bloco de massa
M
2
, que desliza sobre o cho, conforme mostra a Fig. 4.46. O
atrito esttico entre os dois blocos
e
e o atrito cintico entre o
bloco 2 e o cho
c
. a) Determine a mxima fora F que pode
ser aplicada ao bloco 2 sem que o bloco 1 deslize sobre ele. b) se a
fora for aumentada tal que M
1
comea a deslizar, e o atrito
cintico entre os blocos tambm
c
, qual ser a acelerao de
cada massa?
19 - Um bloco de massa M encontra-se sobre outro bloco de mesma
massa, num plano inclinado liso, de ngulo , conforme mostra a
Fig. 4.47. O atrito esttico entre os dois blocos , e entre o bloco
inferior e o plano zero. a) Determine a mxima fora F que pode
ser aplicada ao bloco superior sem que este deslize sobre o bloco
inferior. b) Neste caso, qual ser a acelerao do sistema?







Fig. 4.46 Fig. 4.47

M
R
a
r
M
F


F
M
M
M
2

M
1

F







As leis de Newwton


S. C. Zilio e V. S. Bagnato Mecnica, calor e ondas


84
20 - Um corpo de massa m encontra-se sobre um bloco triangular de
ngulo e massa M, conforme mostra a Fig. 4.48. No existe
atrito entre o bloco triangular e o cho, e o atrito esttico entre os
dois blocos . Pergunta-se: a) qual a fora horizontal mxima F
que pode ser aplicada ao bloco m tal que ele no deslize sobre a
cunha? b) qual o valor da normal nesta situao?






Fig. 4.48






M
F

m






Trabalho e energia

S. C. Zilio e V. S. Bagnato Mecnica, calor e ondas


85






5.1 Trabalho e energia cintica
O conceito de energia um dos mais importantes em Fsica. De uma
forma geral, dizemos que um corpo contm uma determinada quantidade de
energia quando ele tem capacidade de exercer fora e realizar trabalho sobre
um segundo corpo. Para estabelecermos o conceito de energia, vamos
inicialmente definir trabalho em uma dimenso como:
( ) dx x F W
2
1
x
x
x

=
que nada mais do que a rea da curva F
x
(x) entre os pontos x
1
e x
2
. Esta
fora a fora total agindo sobre o corpo, isto ,
( ) ( )

=
=
N
1 i
x x
x F x F
i

Vemos que s h realizao de trabalho quando a fora e o
deslocamento forem no nulos. Podemos ainda definir um trabalho
infinitesimal como sendo:
( ) dx x F dW
x
=
onde dx um deslocamento infinitesimal no qual F
x
(x) pode ser considerada
constante. A unidade de trabalho N.m J ou dyn.cm erg (1J = 10
7
erg).
A partir da definio de trabalho dada acima, podemos usar a 2
a
Lei de
Newton para definir o que energia cintica.
dx
dt
dx
dx
dv
m dx
dt
dv
m dx F W
2
1
2
1
2
1
x
x
x
x
x
x

= = =
5
TRABALHO E
ENERGIA

S. C. Zilio e V. S. Bagnato Mecnica, calor e ondas








86 Trabalho e energia


( )
( ) ( )
1
2
2
2
x
x
2
x
x
x v
m
x v
m
dx
dx
v d
m
dx
dx
dv
v m W
2 2 2
2
1
2
1
= = =


A quantidade K = m / p mv
2
2
1
2
2
1
= denominada de energia cintica.
O resultado mostrado acima, chamado de teorema do trabalho-energia,
estabelece que o trabalho realizado por um sistema de foras igual variao
da energia cintica do corpo no intervalo considerado. Matematicamente,
W = K(x
2
) - K(x
1
)
Exemplo: Vamos considerar um corpo movendo-se sobre um plano
com coeficiente atrito dinmico . Queremos determinar, usando trabalho e
energia, qual a variao da velocidade do corpo com a distncia e qual a
distncia percorrida at ele parar. A condio inicial para este exemplo que
na origem (x = 0) a velocidade v
0
. A fora agindo sobre o corpo F
at
= - N
= - Mg de forma que o trabalho W = -Mgx. Quando o trabalho negativo
significa que estamos retirando energia cintica do corpo. Pelo teorema
trabalho-energia, temos:
2
0
2
Mv Mv Mgx W
2
1
2
1
= =
de onde encontramos:
( ) gx 2 v x v
2
0
=
A posio para a qual o corpo pra dada pela condio v(x) = 0, isto
, ( ) g 2 / v x
2
0
= .
O enfoque que demos ao trabalho at este ponto foi baseado no caso
unidimensional. Podemos generalizar a definio de trabalho para o caso
tridimensional esquematizado na Fig. 5.1 como:
( ) ( )
1 2
S
S
S K S K s d . F W
2
1
= =

r
r









Trabalho e energia

S. C. Zilio e V. S. Bagnato Mecnica, calor e ondas


87






Fig. 5.1 - Realizao de trabalho para o caso tri-dimensional.
e desta forma, apenas a componente da fora paralela ao deslocamento realiza
trabalho. Lembrando da definio de produto escalar, podemos escrever:

+ + =
2
1
2
1
2
1
y
y
z
z
z y
x
x
x
dz F dy F dx F W
de forma que o trabalho em trs dimenses pode ser entendido como a sorna
dos trabalhos em cada dimenso. Este fato est de acordo com o princpio de
Galileu da independncia os movimentos que vimos no Cap. 3. Como o
produto s d . F
r
r
pode variar ao longo do caminho de integrao, o clculo de W
pode muitas vezes ser complicado. Como exemplo deste tipo de clculo,
vamos tomar o caso de um corpo descendo um plano inclinado sem atrito e
partindo do repouso, como mostrado na Fig. 5.2.







Fig. 5.2 - Corpo descendo um plano inclinado sem atrito.
Caso 1: Como primeira maneira de resolver este problema vamos
considerar um eixo s parale1o ao plano e, conseqentemente, tambm paralelo
componente de fora mg sen que realiza trabalho. As componentes N e mg
S
1
S
2
s d
r
r
r

m
Mg sen
s
x
y
Mg
N

S. C. Zilio e V. S. Bagnato Mecnica, calor e ondas








88 Trabalho e energia


cos so perpendiculares ao deslocamento e, portanto, no realizam trabalho.
Temos:
=
= = =

sen gs 2 v
mv mv s sen mg ds sen mg W
2
0
2
S
0
2
1
2
1

Caso 2: Podemos ainda escolher o sistema de coordenadas cartesianas
xy tambm mostrado na Fig. 5.2. Usando N = mg cos , as equaes neste
caso ficam:
F
x
= N sen = mg sen cos
F
y
= N cos - mg = mg (cos
2
- 1) = - mg sen
2

e assim podemos calcular o trabalho nas duas direes:
2
y
2
y
0
y y y
2
x
x
0
x x
mv y sen mg d F W
mv x cos sen mg dx F W
2
1
2
1
= = =
= = =


de onde tiramos: ( ) + = + = sen y cos x sen g 2 v v v
2
y
2
x
2
. Como x = s
cos e y = -s sen, temos: v
2
= 2gs sen (cos
2
+ sen
2
) e portanto,
= sen gs 2 v , que concorda com o resultado obtido anteriormente.
Caso 3: Uma terceira maneira de se calcular o trabalho realizado sobre um
corpo atravs da parametrizao da trajetria, que se traduz no conhecimento
de y(x). Neste caso, de acordo com a Fig. 5.3,

+ = + =
2
2 2
dx
dy
1 dx dy dx ds
( )

+ = =
2
1
2
1
S
S
S
S
2
S S
dx
dx
dy
1 x F ds F W






Trabalho e energia

S. C. Zilio e V. S. Bagnato Mecnica, calor e ondas


89





Fig. 5.3 - Parametrizao de uma trajetria S.
No exemplo do plano inclinado que estamos tratando,
= = tg
dx
dy
x tg y y
0

= = + =

+
cos
1
sec tg 1
dx
dy
1
2
2

( )dx
cos
sen
mg W sen mg F
z
0
s

= =
2
mv
2
1
x
cos
sen
mg W =

=
Como x = s cos v = sen gs 2 , como j havamos encontrado.
Resumindo, vimos trs maneiras de se calcular W. No caso 1),
escolhemos uma coordenada natural para o problema e a soluo foi simples.
No caso 2), escolhemos coordenadas cartesianas e a soluo j foi mais
complicada. No caso 3), a trajetria foi parametrizada por y = y(x), mas este
mtodo s conveniente quando a trajetria for complicada, como por
exemplo, y = x
3
/3, etc.
Uma outra situao que consideraremos a seguir a de um corpo
vinculado a mover-se sobre um cilindro sem atrito e que solto de um ngulo

0
com velocidade nula, como indica a Fig. 5.4. Uma anlise rpida das foras
agindo sobre o corpo indica que apenas a componente tangencial mg cos
capaz de realizar trabalho. N e mg sen so perpendiculares trajetria. Neste
S
1
S
2
s d
r
dx
dy
trajetria

S. C. Zilio e V. S. Bagnato Mecnica, calor e ondas








90 Trabalho e energia


problema, a coordenada natural o ngulo . Vemos que: F
S
= mg cos e ds
= - Rd, j que s e aumentam em sentidos opostos. Assim,







Fig. 5.4 - Corpo vinculado a mover-se sobre um cilindro sem atrito.
( )
( ) ( ) =
= =
= =

sen sen gR 2 v
mv sen sen mgR K W
) sen (sen mgR Rd cos mg W
0
2
0
0
2
1
0

5.2 Potncia
Quando um agente externo realiza trabalho sobre um corpo, podemos
definir potncia como sendo a taxa temporal de energia que ele capaz de
fornecer ao corpo. Assim, no caso de uma fora constante, s d . F dW
r
r
= e
v . F
dt
s d
. F dt / dW P
r
r
r
r
= = = . A unidade de potncia energia/tempo:
[ ] ( ) W Watt s / J P = .
5.3 Energia potencial
Nem sempre o trabalho realizado sobre um corpo por um agente
externo convertido totalmente em energia cintica. Muitas vezes o trabalho
d origem a um outro tipo de energia, chamada energia potencial.
Analogamente energia cintica, um corpo com energia potencial tem a

S
R
x
y
M
Mg cos






Trabalho e energia

S. C. Zilio e V. S. Bagnato Mecnica, calor e ondas


91
capacidade de realizar trabalho. Em geral, nesta situao existe um agente
externo realizando trabalho sobre o sistema de interesse. Atravs da realizao
deste trabalho, o agente externo transfere energia para o sistema, que a
armazena de alguma forma. Quando o agente externo retirado, o sistema
libera a energia armazenada (energia potencial) atravs da realizao de
trabalho e converte esta energia em energia cintica.
Dentre os vrios tipos de energia potencial, os mais comuns so a
gravitacional, elstica (mola) e eltrica (Coulombiano). Como exemplo de
energia potencial gravitacional, vamos considerar um corpo que se desloca
uma altura h = h
2
h
1
. Para isto necessrio um agente externo trabalhando
contra a fora peso, como indicado na Fig. 5.5. Neste caso, F
ext
= mg e o
trabalho realizado :
1 2 ext ext
U U h mg h F W = = =
onde U = mgh definido como energia potencial gravitacional. O trabalho
feito pela fora peso
( )
1 2 p
U U h mg W = =
Se soltarmos o corpo, a energia potencial U = mgh se transformar
em energia cintica. Na verdade, o que fazemos dar condies para a fora
peso realizar trabalho:
h g 2 v mv
2
1
h mg W
2
= = =







Fig. 5.5 - Corpo sob a ao da fora gravitacional.
mg

F
ext
h
h
2
h
1

S. C. Zilio e V. S. Bagnato Mecnica, calor e ondas








92 Trabalho e energia


5.4 Foras conservativas
Como vimos na seco anterior, a energia potencial est associada
existncia de uma fora qual demos condies de realizar trabalho. Como
exemplo, temos a fora gravitacional, fora eletrosttica e fora elstica (de
mola). Estas foras so denominadas conservativas. Quando as foras
conservativas so as nicas existentes no sistema, a soma das energias cintica
e potencial (chamada de energia mecnica total) permanece constante.
Se uma fora conservativa, o trabalho total realizado sobre um corpo
nulo do ele efetua uma trajetria fechada e retorna posio inicial. Isto quer
dizer na trajetria fechada a fora conservativa no retira e nem cede energia
ao sistema. Matematicamente,
0 s d . F =

r
r

Imaginemos que um corpo est indo do ponto S
1
ao ponto S
2
pela
trajetria C
1
, sob ao de uma fora conservativa, como mostra a Fig. 5.6. Ao
atingir S
2
, o corpo retorna ao ponto inicial S
1
pelo caminho C
2
.







Fig. 5.6 - Trajetria fechada seguida pelo corpo sob a ao de fora conservativa.
Nestas condies temos:
0 s d . F s d . F s d . F
2 1
C C
= + =

r
r
r
r
r
r


=
2 1
C C
s d . F s d . F
r
r
r
r

S
1

S
2

C
1

C
2







Trabalho e energia

S. C. Zilio e V. S. Bagnato Mecnica, calor e ondas


93
Por outro lado, se invertermos o sentido de percurso do caminho C
2
, a integral
muda de sinal e assim,


=
2 1
C C
s d . F s d . F
r
r
r
r

Conseqentemente, conclumos que o trabalho realizado por uma fora
conservativa independe do caminho que conecta os pontos 1 e 2. Ele s
depende da posio dos pontos inicial e final do movimento, o que torna
lgico associar-se uma energia potencial a cada ponto do percurso.
Lembrando-me do caso da fora peso,
( ) U U U s d . F W
1 2
= = =

r
r

e para um deslocamento infinitesimal, s d . F dW
r
r
= . As energias potenciais
mais comuns so:
a) gravitacional (prximo superfcie da Terra): y d s d , y mg F
r r
r
= =
( )
1
y
y
2 1 2
y mg y mg dy mg U U
2
1
= =


Portanto, U(y) = mgy + C
b) Elstica (de uma mola): x d s d , x kx F
r r
r
= =
( ) ( ) ( )
2
1
2
2
x
x
1 2
x x dx x k x U x U
2
1 2
1
= =

( ) C x x U
2
k
2
1
+ =
c) Eletrosttica: r
r
F
2

=
r
e dr s d . r =
r

( ) ( ) ( ) C
r
r U
r
1
r
1
dr
r
r U r U
1 2
r
r
2
1 2
2
1
+

=



S. C. Zilio e V. S. Bagnato Mecnica, calor e ondas








94 Trabalho e energia


5.5 Determinao da fora a partir da energia
potencial
Corno vimos anteriormente, toda fora conservativa est associada
uma energia potencial. Muitas vezes conhecemos a energia do sistema e a
partir dela queremos encontrar a fora e o movimento do corpo. Considerando
apenas o caso unidimensional,
dx
dU
F dx F dU
x x
= =
ou seja, o conhecimento da energia potencial permite o clculo da fora que
age sobre o corpo. No caso de foras radiais, fcil verificar que:
dr
dU
F
r
=
O uso desta expresso importante quando queremos determinar as
posies de equilbrio de um corpo. Consideremos, por exemplo, a energia
potencial de uma mola, que como vimos, dada por: U(x) = kx
2
. Neste
caso, F(x) = -kx. Vemos que no ponto x = 0, dU/dx = 0 e assim F(0) = 0 e
assim esta uma posio de equilbrio. Neste caso, o equilbrio estvel, pois
quando o corpo se afasta da origem a mola exerce uma fora restauradora no
mentido de traz-lo de volta. Por outro lado, se considerarmos um potencial do
tipo:
2 2
a x
C
) x ( U
+
=
como mostra a Fig. 5.7, a fora ser dada por:
( )
2
2 2
x
x
a x
C 2
F
+
=
Neste caso tambm temos 0 dx / dU = em x = 0, porm agora o equilbrio
instvel pois quando o corpo se afastar da origem, teremos uma fora positiva
que o obriga a se afastar ainda mais. Note que:






Trabalho e energia

S. C. Zilio e V. S. Bagnato Mecnica, calor e ondas


95
x
U(x)
0
a
C 2
dx
U d
4
0 x
2
2
< =
=

De um modo geral, dado U(x), teremos equilbrio se U(x
2
)= 0. O equilbrio
ser estvel se U(x
0
) > 0 e instvel se U(x
0
) < 0, onde x
0
o ponto de
equilbrio.








Fig. 5.7 Potencial com ponto de equilbrio instvel.
5.6 Foras dissipativas
Alm das foras conservativas temos ainda as chamadas foras
dissipativas, que ao contrrio das primeiras, removem energia do sistema,
transformando-as em outras formas de energia, como por exemplo, calor. Na
presena de foras dissipativas, o trabalho realizado por estas foras igual
variao da energia mecnica total do sistema. Tomemos por exemplo, o caso
do atrito. Lanando-se um corpo de massa m com velocidade v
0
sobre a mesa
com atrito , o trabalho realizado pela fora de atrito :
2
0
2
mv mv x mg W
2
1
2
1
= =
onde x a distncia percorrida pelo corpo e a diferena de energia
dissipada na forma de calor.

S. C. Zilio e V. S. Bagnato Mecnica, calor e ondas








96 Trabalho e energia


5.7 Conservao de energia
At agora vimos que um sistema mecnico pode apresentar dois tipos
de energia potencial, do tipo:
C r d ). r ( F ) r ( U
2
1
+ =

r r
r
r

e a cintica:
2
mv K
2
1
=
A soma dessas energias denominada energia mecnica total do
sistema num determinado ponto
K U E
mec
+ =
Na ausncia de foras dissipativas esta quantidade uma constante de
movimento, isto ,
0
dt
dE
mec
=
Como exemplo, vamos considerar o sistema massa-mola na ausncia
de foras dissipativas. A energia mecnica dada por:
2 2
mec
kx mv E
2
1
2
1
+ =
Como kx dt / dv m = e v dt / dx = , temos:
0 vkx vkx
dt
dE
mec
= + =
No caso de haver foras dissipativas,
P
dt
dE
W E
m
fd mec
= =
onde P a energia dissipada. No caso do problema com atrito que resolvemos
na seco anterior temos:






Trabalho e energia

S. C. Zilio e V. S. Bagnato Mecnica, calor e ondas


97
( ) gx 2 v x v g a
2
0
= =
( ) gx 2 v mg x mgv
dt
dE
mgx mv
2
1
mv
2
1
E
2
0
mec
2
0
2
mec
= =
= =

Um grfico desta potncia como funo de x est mostrado na Fig.
5.8.








Fig. 5.8 - Potncia como funo da posio.
O uso da lei de conservao de energia muito importante porque
quase sempre permite a resoluo de problemas sem a necessidade de se
resolver a equao de movimento. Vamos a seguir apresentar alguns exemplos
que utilizam o princpio da conservao de energia.
a) Pndulo simples
Este problema j foi resolvido atravs da 2
a
Lei de Newton, de onde
obtivemos ( ) ( )
0
2
cos cos Lg 2 v = . Vamos obter este mesmo resultado
usando conservao de energia. O pndulo solto com v
0
= 0 na posio
0
,
como indica a Fig. 5.9. Escolhendo a posio do teto como U = 0, temos
( )
( )
2
0 0
mv cos mgL E
cos mgL E
2
1
+ =
=

mgv
g 2
v
2
0


x
P
0

S. C. Zilio e V. S. Bagnato Mecnica, calor e ondas








98 Trabalho e energia


( )
2
0 0
mv cos mgL cos mgL ) ( E E
2
1
+ = =
de onde tiramos que ( ) ( )
0
2
cos cos Lg 2 v = .







Fig. 5.9 - Pndulo simples.
b) Mquina de Atwood
Este dispositivo, tambm j discutido com a 2
a
Lei de Newton est
esquematizado na Fig. 5.10. Vamos supor que os corpos so soltos em y
10
e
y
20
. A conservao da energia mecnica fornece:
2
2 2
2
1 1 2 2 1 1 20 2 10 1
v m v m gy m gy m gy m gy m
2
1
2
1
+ + + = +
Derivando em relao ao tempo temos: 0 = m
1
gv
1
+ m
2
gv
2
+ m
1
v
1
dv
1
/dt +
m
2
v
2
dv
2
/dt, onde dt / dv a
1 1
= e dt / dv a
2 2
= . Como a corda inextensvel,









Fig. 5.10 - Mquina de Atwood.
L
M

U=0
U=0

m
1
m
2
a
r a
r






Trabalho e energia

S. C. Zilio e V. S. Bagnato Mecnica, calor e ondas


99
1 2 1 2
a a e v v = = , e portanto: ( ) ( )
1 2 1 1 2 1
av m m gv m m 0 + + = , de
onde sai que:
( )
( )
g
m m
m m
a
1 2
1 2
1
+

=
para cima, pois positiva e
1 2
a a = , para baixo pois negativa.
c) Corpo preso num aro por meio de uma mola
Vamos considerar um corpo de massa m preso a um aro sem atrito
atravs de uma mola constante k e comprimento livre nulo. O corpo solto do
ponto 0 mostrado na Fig. 5.11, com velocidade inicial nula. Queremos
encontrar as velocidades nos pontos 1 e 2. Usando conservao da energia
mecnica temos:
( ) ( ) ( )
( ) ( )
( )
2
2
2
1
2
2
mv 2 E
mv mgR R 2 k 1 E
R 2 mg R 2 k 0 E
2
1
2
1
2
1
2
1
=
+ + =
+ =

Fazendo E(0) = E(1) e E(0) = E(2) obtemos respectivamente:
m
mgR kR
2 v
2
2
1
+
= e
m
mgR kR
4 v
2
2
2
+
= , e portanto,
1 2
v 2 v = .







Fig. 5.11 - Corpo preso num aro por meio de uma mola.
R

k

1

2

U=0

0

m

S. C. Zilio e V. S. Bagnato Mecnica, calor e ondas








100 Trabalho e
energia


d) Fora viscosa
Vamos ver um exemplo onde a energia no se conserva.
Consideremos um corpo lanado com velocidade
0
v num meio viscoso cuja
fora de atrito F = -bv. Neste caso no temos energia potencial, s energia
cintica. No incio do movimento,
2
0
mv K
2
1
= . Para um deslocamento
infinitesimal x:
( ) ( )
2
bv
dt
dK
bv
v
1
dt
dK
dx
dt
dt
dK
bv
dx
dK
x bv v K v v K K
=
= = =
= + =

Como
2
2
1
mv K = temos
m
K 2
v
2
= . Logo,
dt
m
b 2
K
dK
K
m
b 2
dt
dK
= =
Integrando entre 0 e t temos: t
m
b 2
K
K
n
0

l , de onde sai que: K


= K
0
exp{-2bt/m} e portanto,
2
0
2
mv mv
2
1
2
1
= exp{-2bt/m}. Tirando a raiz
obtemos: v = v
0
exp{-bt/m}, que coincide com o resultado obtido com a 2
a
lei
de Newton.
5.8 Corpo sob a ao de um potencial arbitrrio
Quando um corpo move-se num potencial arbitrrio conservativo,
como aquele mostrado na Fig. 5.12, a energia total ( ) ) x ( U x mv E
2
2
1
+ =
uma constante de movimento. Nos pontos x
1
e x
2
, K = 0, E = U(x
1,2
) e,
portanto, v = 0. Estes pontos so chamados pontos de retorno. O movimento






Trabalho e energia

S. C. Zilio e V. S. Bagnato Mecnica, calor e ondas


101
U
K
s ocorre entre x
1
e x
2
, pois fora desta regio U(x) > E e a energia cintica
teria que ser negativa, o que implicaria numa velocidade imaginria. Para
encontrarmos a equao de movimento, fazemos:
( ) ) x ( U E
m
2
dt
dx
x v = =
dt
m
2
) x ( U E
dx
=


Integrando esta igualdade teremos x = x(t), que representa a equao
de movimento.








Fig. 5.12 - Corpo movendo-se num potencial arbitrrio.
Exerccios
1 - Um corpo acelerado uniformemente a partir do repouso at atingir a
velocidade v
f
no tempo t
f
. Mostre que a potncia instantnea fornecida ao
corpo :
( )
2
f
2
f
t
t
mv t P =
2 - Considere o sistema da Fig. 5.13, onde a fora F constante e os planos
tm coeficiente de atrito dinmico . Calcule o trabalho total realizado
pelas foras agindo no sistema quando o mesmo desloca-se uma distncia
infinitesimal x.

S. C. Zilio e V. S. Bagnato Mecnica, calor e ondas








102 Trabalho e
energia






Fig. 5.13
3 - Considere o potencial de Lennard-Jones comumente utilizado como sendo
a energia de interao entre dois tomos constituindo uma molcula:
( ) ( ) ( ) [ ]
6
0
12
0
r r 2 r r C r U =
a) Faa um grfico de U(r) contra r, b) Mostre que o mnimo de energia
(posio de equilbrio) ocorre em r
0
, c) Ache a fora entre os tomos como
funo de r e d) Qual a energia necessria para separar os tomos que
constituem a molcula?
4 - Um pndulo de massa m e comprimento l solto do ponto = 60
o
a partir
do repouso, como indicado na Fig. 5.14. Ao atingir a posio vertical =
0
o
, o cordo do pndulo encontra um prego colado a uma distncia d do
teto. Encontre a distncia d mnima que a massa m execute rotao ao
redor do prego.
5 - Um corpo de massa m move-se no interior de um trilho circular vertical de
raio R (Fig. 5.15). Quando m est na posio mais baixa sua velocidade
v
0
. a) Qual o mnimo valor de v
0
tal que o corpo percorra todo o trilho? b)
Se v
0
for 78% do valor determinado em a), o corpo sobe pelo trilho at o
ponto P, perder contato com o trilho. Determine a coordenada deste
ponto.
6 - Um corpo de massa M, sujeito a um potencial U(x) = - cosx, solto na
origem (x = 0) com velocidade v
0
. a) Faa um esboo do potencial na
regio - 1 x 1; b) Encontre a fora F(x) agindo no corpo e c) Qual a
mxima velocidade v
0
que pode ser dada ao corpo de tal maneira que ele
fique confinado na regio - 1 x 1?

F
M
M






Trabalho e energia

S. C. Zilio e V. S. Bagnato Mecnica, calor e ondas


103









Fig. 5.14 Fig. 5.15
7 - Uma massa m escorrega sem atrito ao longo da montanha russa mostrada
na Fig. 5.16. A parte circular tem raio R e a massa parte do repouso no
ponto B, altura h medida em relao base dos trilhos. a) Qual a
energia cintica de m no ponto P? b) Qual a acelerao de m no ponto P,
admitindo que a massa permanea no trilho? c) Qual o menor valor de h
para que m execute o movimento circular? d) Para um valor h maior do
que este mnimo escreva a expresso da fora normal exercida pelo trilho
sobre a massa.
8 - Um corpo de 2 Kg solto num plano inclinado de um ponto que dista 4 m
de uma mola de constante de fora k = 100 N/m. A mola est fixa
paralelamente ao plano, inclinada de = 30
0
(Fig. 5.17). a) Calcular a
compresso mxima da mola, admitindo que a sua massa seja desprezvel;
b) Calcular a compresso mxima da mola, quando o plano inclinado tem
atrito (coeficiente de atrito entre ele e o corpo igual a 0,2); c) No caso do
plano com atrito, qual a altura atingida pelo corpo no seu retorno para
cima?
9 - Dois corpos andando com mesma velocidade v sobre um plano horizontal
esto distanciados de d. Aps subirem uma ladeira de altura h, qual ser a
distncia entre eles? (Fig. 5.18).

l
m

Prego
R

m
v
0

P

S. C. Zilio e V. S. Bagnato Mecnica, calor e ondas








104 Trabalho e
energia


h
d
d
M
L
O
v
r





Fig. 5.16 Fig. 5.17
10 - Um bloco desliza com velocidade v
0
sobre um plano horizontal sem atrito.
Subitamente ele encontra uma rampa com ngulo de inclinao e
coeficiente de atrito dinmico . Qual altura mxima H o bloco sobe na
rampa?
11 - Um corpo de massa M preso por uma corda de comprimento L e pode
rodar em torno do ponto O, como indicado na Fig. 5.19. Qual a mnima
velocidade que o corpo pode ter ao passar pelo plano horizontal de forma
que ele fique em movimento circular?




Fig. 5.18 Fig. 5.19
12 - Um corpo colocado exatamente na vertical de uma superfcie cilndrica
sem atrito, comea a deslizar com velocidade v
0
, conforme mostra a Fig.
5.20. (a) Encontre sua velocidade em funo do ngulo . (b) Encontre a
fora normal como funo do ngulo . (c) Determine o ngulo para o
qual corpo se desprende do cilindro.
13 - Um corpo de massa m preso a uma mola vertical, de constante de mola
k, como mostra a Fig. 5.21. O corpo solto a partir do repouso, da posio
y=0, sendo que nesta situao a mola no est distendida. a) Escreva a
energia potencial como funo de y (tome o zero de energia potencial
B
m
h
R
P
30
o
4m
k
m






Trabalho e energia

S. C. Zilio e V. S. Bagnato Mecnica, calor e ondas


105
gravitacional na posio onde a mola no est distendida). b) Complete
quadrados e faa um grfico de U(y) contra y, c) usando este grfico,
encontre a posio de equilbrio do corpo, d) qual o deslocamento
mximo realizado pelo corpo? e) qual a velocidade mxima atingida
pelo corpo?




Fig. 5.20 Fig. 5.21
14 - Um pndulo simples de massa m e comprimento L encontra-se em
repouso na vertical. Subitamente a massa recebe um impulso
instantneo que lhe confere uma velocidade v
0
, como mostra a Fig.
5.22. Encontre: a) a velocidade tangencial como funo do ngulo
, b) a tenso na corda como funo do ngulo , c) a menor
velocidade (v
0
)
min
que permite ao pndulo realizar uma volta
completa em torno do pino O.
15 - Um bloco de massa M desliza sobre uma mesa com coeficiente de
atrito cintico =3/4
.
Ele colide com uma mola de massa
desprezvel, de constante de mola k, inicialmente na posio
relaxada, como mostra a Fig. 5.23. Na hora que o bloco atinge a
mola ele possui velocidade v
0
= /k Mg
2
. a) encontre a energia
cintica como funo da posio x, b) complete quadrados e faa
um grfico de E
k
(x), c) qual a deformao mxima da mola? d) que
frao da energia inicial dissipada pelo atrito neste processo?


M
y
k


m
R
v
0


S. C. Zilio e V. S. Bagnato Mecnica, calor e ondas








106 Trabalho e
energia








Fig. 5.22 Fig. 5.23
16 - Considere um corpo de massa m preso a um aro de raio R, sem
atrito, atravs de uma mola de constante k e comprimento livre
nulo, como mostra a Fig. 5.24. O corpo solto do ponto O com
velocidade inicial nula. Tomando o zero da energia potencial
gravitacional como mostrado na figura, encontre: a) a energia
mecnica do sistema no ponto O, b) uma expresso para a energia
mecnica no ponto P descrito pelo ngulo , c) a velocidade da
massa no ponto P, d) a fora de reao do trilho no ponto P, e e) o
menor valor de k para que a massa permanea em contato com o
trilho.






Fig. 5.24



m

k
U=0
R
O
P

L
m
v
0
v

O

k
M

v
0
x






Sistemas de partculas Conservao de momentum

S. C. Zilio e V. S. Bagnato Mecnica, calor e ondas


107






6.1 Centro de massa
Quando foras externas agem sobre um sistema composto de vrios
corpos, cada um deles movimenta-se, em princpio, de uma forma diferente. O
movimento total do sistema bastante complicado, porm existe um ponto
particular, chamado centro de massa, cujo movimento pode ser encontrado
com facilidade. Sua introduo visa facilitar a soluo de problemas
envolvendo muitos corpos e seu comportamento como se toda massa do
sistema estivesse concentrada sobre ele. Para um sistema composto de N
massas, o centro de massa definido como:



= =
= =
= =
= |

\
|
= |

\
|
= |

\
|
N
1 i
i i CM
N
1 i
i
N
1 i
i i CM
N
1 i
i
N
1 i
i i CM
N
1 i
i
z m Z m
y m Y m
x m X m

onde (X
CM
, Y
CM
, Z
CM
) so as coordenadas do centro de massa e (x
i
, y
i
, z
i
) so
as coordenadas do i-simo corpo. Para um sistema de duas partculas em uma
dimenso, por exemplo,
MX
CM
= m
1
x
1
+ m
2
x
2

Tomemos alguns casos particulares:
a) m
1
= m
2
, x
1
= 0, x
2
= d X
CM
= d/2
6
SISTEMA DE PARTCULAS
CONSERVAO DE MOMENTUM






Sistemas de partculas Conservao de momentum


S. C. Zilio e V. S. Bagnato Mecnica, calor e ondas


108
R

x
y
d

= = Rd
R
M
ds dm
b) m
1
= 2m
2
, x
1
= 0, x
2
= d X
CM
= d/3
Para um sistema com distribuio contnua de massa, fazemos m
i

dm,

V
e a definio de X
CM
generalizada como:

=
V
CM
dm x
M
1
X
Como exemplo, vamos calcular a posio do centro de massa de uma
barra rgida de comprimento L e a massa M, mostrada na Fig. 6.1. Neste caso:





Fig. 6.1 - Barra rgida de comprimento L e massa M.

= =
L
0
L
0
CM
dx x
M
1
dm x
M
1
X
2
L
2
L
L
M
M
1
X
2
CM
=
|

\
|
=
Como segundo exemplo, vamos encontrar o centro de massa de um
semicrculo de raio R e massa M mostrado na Fig. 6.2, onde R o
comprimento do semicrculo.





Fig. 6.2 - Semicrculo de raio R e massa M.
( ) 0 sen
R
d cos
R
Rd
R
M
cos R
M
1
X
0 0 0
CM
=

=



L
dx
x
dm = dx





Sistemas de partculas Conservao de momentum

S. C. Zilio e V. S. Bagnato Mecnica, calor e ondas


109

( ) R 6 . 0
R 2
cos
R
d sen
R
Rd
R
M
sen R
M
1
Y
0 0 0
CM

=



6.2 Movimento do centro de massa
Tomando a derivada temporal na equao que define a coordenada
X
CM
do centro de massa em uma dimenso, temos:
...
dt
dx
m
dt
dx
m
dt
dx
M
2
2
1
1
CM
+ + =
onde p
i
= m
i
v
i
o momentum do i-simo corpo. Tomando-se novamente a
derivada temporal e usando-se a 2
a
Lei de Newton:
F F
dt
dp
Ma
dt
dV
M
N
1 i
i
N
1 i
i
CM
CM
= = = =

= =

Assim, a resultante de todas as foras atuantes sobre o sistema
obedece a 2
a
Lei de Newton, desde que seu efeito seja considerado sobre o
centro de massa. interessante notar-se que quando somamos todas as foras
existentes no sistema, estamos considerando, alm das foras externas, as
foras internas exercidas por um corpo sobre o outro. Desta forma

= = =
+ = =
N
1 i
ext
i
N
1 i
int
i
N
1 i
i
F F F F
Como sabemos, as foras internas sempre ocorrem aos pares (ao -
reao) e cancelam-se mutuamente quando efetuamos a soma sobre todos os
constituintes do sistema |

\
|
=

=
0 F
N
1 i
int
i
. Assim, costumamos dizer que as
foras internas no modificam o estado de movimento do sistema como um
todo e portanto,






Sistemas de partculas Conservao de momentum


S. C. Zilio e V. S. Bagnato Mecnica, calor e ondas


110

=
= =
N
1 i
ext ext
i CM
F F Ma
Somente foras externas so capazes de modificar o estado de movimento do
centro de massa do sistema, que se move como se fosse urna nica partcula
de massa M =

=
N
1 i
i
m , sob a ao da fora externa resultante que atua no
sistema.
Exemplo: Imaginemos um projtil lanado obliquamente que explode no
ponto alto da trajetria de modo que a primeira metade cai verticalmente,
conforme mostra a Fig. 6.3. Queremos calcular a trajetria da segunda metade.
Existe apenas a fora externa peso agindo sobre o sistema. Desta forma, de
acordo com o que vimos no Cap. 3 sobre lanamento de projteis,






Fig. 6.3 - Projtil lanado obliquamente que explode no ponto alto da trajetria.
t cos v X
0 CM
=
2
0 CM
gt t sen v Y
2
1
=
Aps a exploso, que ocorre em
g
sen v
t
0
max

= , tempo que a massa 2m
demora para atingir o topo da trajetria, temos:
te tan cons
g 2
2 sen v
x
2
0
1
=

=
( )
CM
2
0
2
max
2 2
0
1
Y
2
t
g t sen v
2
t t
g
g 2
sen v
y = =

=
v
0
m

m

1

2

2m







Sistemas de partculas Conservao de momentum

S. C. Zilio e V. S. Bagnato Mecnica, calor e ondas


111
Vemos que a altura da massa 1 a mesma que a do centro de massa, ou em
outras palavras, eles caem juntos. Podemos calcular a posio da massa 2 de
acordo com:
(


+ = =
g 2
2 sen mv
mx
m 2
1
t cos v X
2
0
2 0 CM

( )
g 2
2 sen v
t cos v 2 t x
2
0
0 2

=
e,
(

\
|
+ = =
2
gt
t sen v m my
m 2
1
2
gt
t sen v Y
2
0 2
2
0 CM

( )
CM
2
0 2
Y
2
gt
t sen v t y = =
que assim como m
1
cai junto com o centro de massa. Da equao para x
2
(t)
eliminamos o tempo:

+
=
cos v 2
g 2 / 2 sen v x
t
0
2
0 2

e pela substituio deste em y
2
(t) encontramos a equao da trajetria y
2
(x
2
)
aps a exploso:
( )
( )

+
|

\
|
+

=
2 2
0
2
2
0 2
2
0
2 2 2
cos v 4
g 2 / 2 sen v x
2
g
g 2
2 sen v
x
2
tg
x y
Daqui obtemos que y
2
= 0 ocorre quando
g
2 sen v
2
3
x
2
0
max

= , como
esperado. Na ausncia de foras externas, a velocidade do centro de massa
constante, de onde segue que P =

i
i i
v m tambm uma constante de
movimento. Isto quer dizer que durante colises ou movimentos relativos
entre as vrias partes do sistema, o momentum total conservado. Quando






Sistemas de partculas Conservao de momentum


S. C. Zilio e V. S. Bagnato Mecnica, calor e ondas


112
estudamos colises, as leis de conservao de energia e momentum sero de
extrema importncia.
6.3 Sistemas onde a massa varia
Sistemas onde a massa varia so difceis de serem resolvidos pela
aplicao direta da 2
a
Lei de Newton porque m e v variam simultaneamente.
Vamos considerar a situao mostrada na Fig. 6.4. Uma quantidade de massa
m, com velocidade ' v
r
, deposita-se sobre um corpo de massa m deslocando-
se com velocidade v
r
. Vamos supor a existncia de uma fora F
r
que pode
alterar o momentum do sistema. No instante imediatamente anterior coliso:
' v m v m ) t ( P
r r
+ =
e no instante imediatamente posterior coliso:
( ) ( ) v m v v m v v ) m m ( ) t t ( P
r r r r r
r
+ + + + = +
onde o termo de ordem superior v m
r
foi desprezado por ser muito
pequeno.




Fig. 6.4 - Sistema com massa varivel.
A variao do momentum dada por:
( ) ( ) v ' v m v m ) t ( P t t P P
r r r
r r r
= + =
Podemos reconhecer v ' v u
r r r
= como sendo a velocidade de m
relativa massa m, de forma que ficamos com: u m v m P
r r
r
= .
Dividindo-se esta expresso por t e tomando o limite para t 0, obtemos:
u
dt
dm
dt
v d
m
dt
P d
F
r
r
r
r
= =
' v
r

F
r

v
r

m
m





Sistemas de partculas Conservao de momentum

S. C. Zilio e V. S. Bagnato Mecnica, calor e ondas


113
ou alternativamente,
u
dt
dm
F a m
r
r
r
+ =
que a 2
a
lei de Newton, porm modificada pelo termo u
dt
dm
r
, conhecido por
fora de empuxo (no confundir com a que aparece no princpio de
Arquimedes, que veremos posteriormente ao tratar a mecnica dos fluidos).
Na ausncia de um agente externo, esta a fora que exercida sobre o
sistema pela poro de massa que foi adicionada ou que deixou o sistema.
Passemos agora a analisar alguns exemplos em que a equao acima se aplica.
a) Correia de carga - Cai areia a uma taxa dm/dt sobre uma correia
deslocando-se com velocidade constante v, como mostra a Fig. 6.5. Neste
caso, a acelerao nula e a velocidade da areia sendo adicionada relativa
correia u, pois tomamos o sentido positivo das velocidades para a direita. A
fora necessria para manter a correia com velocidade v :
u
dt
dm
F =







Fig. 6.5 - Correia de carga.
b) Foguete no espao sem gravidade - Em vrias situaes fsicas o sistema
consegue um grande impulso atravs da ejeo de massa. Consideremos um
foguete num instante de tempo t, como esquematizado na Fig. 6.6. Vamos
supor que o foguete esteja no espao sem gravidade. Entre t e t + t, uma
quantidade m de massa ser expelida do foguete com uma velocidade u
r

v
areia







Sistemas de partculas Conservao de momentum


S. C. Zilio e V. S. Bagnato Mecnica, calor e ondas


114
M

m

v
r
t

M

m

u v
r r
+
v v
r r
+
t +t
(relativa ao foguete). Aplicando a 2
a
lei de Newton modificada pela fora de
empuxo temos:






Fig. 6.6 - Foguete no espao sem gravidade.
u
dt
dm
0
dt
v d
m
r
r
r
+ =
onde 0 F
r r
= , dt / v d
r
a acelerao do foguete com relao a um referencial
em repouso e u
r
a velocidade da massa expelida com relao ao foguete.
Tomando o sentido positivo das velocidades para a direita vemos que u
r

negativo, mas como o foguete perde massa, dm/dt tambm negativo. Assim,
a acelerao dirigida para a direita. A equao de movimento pode ser
integrada resultando em:
|

\
|
+ =
= =

0
0
m
m
v
v
m
m
n u v v
m
dm
u v d
m
dm
u v d
0 0
l
r r r
r r r r

sendo m
0
a massa inicial do foguete. Se o foguete tem uma carcaa que pesa
M, carrega uma quantidade de combustvel M e parte do repouso, a
velocidade final ser:
|
|

\
|
=
|
|

\
|
+ =
f
i
i
f
M
M
n u
M
M
n u 0 v l
r
l
r r

onde M
i
= M + M e M
f
= M. Como o sentido de u
r
oposto ao de v
r
temos:
( )
M
M
1 n u v

+ = l





Sistemas de partculas Conservao de momentum

S. C. Zilio e V. S. Bagnato Mecnica, calor e ondas


115
v
r
y
g M F
r
r
=
c) Foguete no campo gravitacional - Esta situao est mostrada na Fig. 6.7.
Neste caso, g M F
r
r
= e portanto,
( )
dt
dm
m
u
g
dt
dv
dt
dm
u
dt
dv
m mg
y u u e y v v , y g g
dt
dm
u g m
dt
v d
m
+ = + =
= = =
+ =
r r r
r r
r









Fig. 6.7 - Foguete no campo gravitacional.
Tomando v
0
= 0 e massa inicial m
0
,
gt
m
m
n u v
m
dm
u dt g dv
0
t
0
m
m
v
0
0

\
|
=
=

l







Sistemas de partculas Conservao de momentum


S. C. Zilio e V. S. Bagnato Mecnica, calor e ondas


116
Exerccios
1 - Um bloco de massa m repousa sobre uma cunha de massa M e ngulo ,
que por vez est colocada sobre uma superfcie horizontal, como
mostrado na Fig. 6.8. Soltando sistema a partir do repouso, com o corpo a
uma altura h, determine a velocidade da cunha quando o bloco tocar o
solo. Todas as superfcies so isentas de atrito.






Fig. 6.8
2 - Um corpo de massa M = 400 g solto do repouso de uma altura h = 10 m
com relao superfcie da Terra. Simultaneamente, uma bala de massa
m = 100 g disparada verticalmente da superfcie com velocidade v
0
= 10
m/s. Sabendo-se que em algum lugar da trajetria as massas colidem e se
unem, pergunta-se qual o tempo que as massas demoram para cair desde
o instante em que M solta.
3 - Um vago de carga aberto pesa 10 ton. e est deslizando num trilho sem
atrito com velocidade de 60 cm/s. Inicia-se repentinamente uma forte
chuva e as gotas caem verticalmente com v
0
em relao ao cho. Qual a
velocidade do vago aps coletar 500 Kg de gua?
4 - Sobre o prato de uma balana colocada uma ampulheta. No instante
inicial (t = 0) toda a areia est em repouso na parte superior da ampulheta.
Sendo que a areia cai para o recipiente inferior a uma taxa = dm/dt, faa
um grfico mostrando a leitura na balana para t 0.

m
h
M





Sistemas de partculas Conservao de momentum

S. C. Zilio e V. S. Bagnato Mecnica, calor e ondas


117
5 - Um jato de gua com velocidade v
0
e fluxo = dm/dt dirigido para cima
conforme mostra a Fig. 6.9. Uma lata com a boca para baixo sustentada
pelo jato. Se a massa da lata M, a que altura ela ficar?
6 - Uma gota de chuva de massa inicial M
0
comea a cair a partir do repouso.
Supondo que a gota ganhe massa ao passar pelas nuvens a uma taxa
proporcional ao produto da massa pela velocidade (dm/dt = KMv),
calcule a velocidade v(t). Despreze a resistncia do ar.
Nota:


2
ax 1
dx
=
a
) x a ( arctgh





Fig. 6.9
7 - Um foguete de brinquedo tem uma carcaa que pesa 100 g e uma
quantidade inicial de combustvel de 400 g. A velocidade relativa ao
foguete com que o combustvel sai 100 m/s e a taxa com que ele
queimado de 100 g/s. Supondo que ele decole da superfcie da Terra (g
= 10 m/s
2
constante), com velocidade inicial nula, que velocidade
mxima ele atingir?
8 - Calcule a potncia necessria para levantar verticalmente uma corda
inicialmente enrolada no solo, com velocidade constante v
0
. A densidade
linear de massa da corda e no incio ela est completamente enrolada.
9 - Calcule as coordenadas (X
CM
,Y
CM
) do centro de massa de um pedao de
anel delgado (arco) de raio R e ngulo 2, como mostra a Fig. 6.10.
10 - Um carrinho motorizado, de massa m, encontra-se sobre uma prancha de
madeira de massa M, que por sua vez encontra-se sobre o cho
extremamente liso (sem atrito), conforme mostra a Fig. 6.11. O carrinho
M
v
0






Sistemas de partculas Conservao de momentum


S. C. Zilio e V. S. Bagnato Mecnica, calor e ondas


118
comea a andar com acelerao a relativa prancha. a) Encontre a
distncia que a prancha percorre aps decorrido um intervalo de tempo .
b) Qual a energia cintica relativa ao centro de massa depois de
decorrido este tempo?










Fig. 6.10 Fig. 6.11








y

x
R

m

M
M





Colises

S. C. Zilio e V. S. Bagnato Mecnica, calor e ondas


119






7.1 Impulso
Coliso pode ser definida como sendo a interao momentnea entre
dois corpos. Durante o intervalo de tempo em que ocorre a coliso, as foras
entre os corpos alteram-se sensivelmente. As leis de conservao em Fsica
auxiliam consideravelmente o estudo das colises.
Vamos comear considerando a lei trabalho-energia que vimos no
Cap. 5. Uma fora F agindo sobre um corpo de massa m ao longo de um
pequeno deslocamento x realiza um trabalho:
( )( )
1 2 1 2
2
1
2
2
v v v v
2
m
mv mv x F
2
1
2
1
+ = =
No caso do deslocamento ser infinitesimal, v
1
v
2
= v e v
2
- v
1
= v e assim,
Fx = mv v
dividindo por t e tomando o limite t 0
dt
dv
mv Fv =
de onde obtemos a 2
a
lei de Newton para massa constante,
dt
dp
dt
dv
m F = =
Se a fora age sobre o corpo durante um intervalo de tempo t = t
2
t
1
,
podemos integrar a expresso acima e obter:

7
Colises






Colises


S. C. Zilio e V. S. Bagnato Mecnica, calor e ondas


120
1 2
t
t
p p dt F I
2
1
r r
r r
= =


onde j fizemos uma generalizao para o caso tridimensional. A grandeza I
r

o impulso causado pela fora F
r
sobre o corpo durante o intervalo de tempo
t. Dizemos ento que o impulso causado pela fora F igual variao de
momentum do corpo.
Durante as colises, as foras existentes em geral agem durante
intervalos de tempo bem curtos e a representao grfica para este tipo de
fora est mostrada na Fig. 7.1. Do que discutimos anteriormente, a variao
da quantidade de movimento exatamente a rea sob a curva. Muitas vezes
interessante definir a fora mdia que age sobre o corpo:
t / I F
m
=
r r







Fig. 7.1 - Exemplo de variao da fora com o tempo.

Como exemplo, podemos calcular a fora mdia exercida pelas bolas
disparadas por uma metralhadora contra um alvo. Se a metralhadora dispara R
balas por segundo, a fora real exercida sobre o alvo algo do tipo mostrado
na Fig. 7.2.





Fig. 7.2 - Sequncia de impactos produzida pelas balas de uma metralhadora.
t
F(t)
t
F(t)
F
m
1/R





Colises

S. C. Zilio e V. S. Bagnato Mecnica, calor e ondas


121

Desta forma, duraste o tempo 1/R, um momentum mv transferido ao alvo, ou
seja:
mv
R
1
. F
m
=
Rmv F
m
=
Para R 4/s, m = 0,05 Kg e v = 100 m/s, temos F
m
= 20 N.
7.2 Transporte de momentum para uma superfcie.
Presso de um gs
Queremos encontrar a fora exercida por um feixe de partculas de
velocidade v e espaamento l sobre uma superfcie. Aps a coliso as
partculas deixam a superfcie com velocidade v, como indica a Fig. 7.3.






Fig. 7.3 - Colises de partculas com uma parede.

Durante um tempo = l/v (tempo de chegada), o momentum
transferido para a superfcie m(v + v) e conseqentemente a fora mdia
dada por:
( ) ( )
l l
' v v mv
v /
' v v m
F
m
+
=
+
=
No caso em que v = v temos um choque perfeitamente elstico e definindo
uma densidade linear de massa como = m/l, a fora mdia se torna:
l
v
r
v
r






Colises


S. C. Zilio e V. S. Bagnato Mecnica, calor e ondas


122
2
2
m
v 2
mv 2
F = =
l

Uma das grandes aplicaes da transferncia de momentum para
superfcies o clculo da presso que um gs contido numa caixa exerce sobre
as paredes da mesma. Vamos imaginar as molculas do gs como sendo
esferas rgidas de massa m. Consideremos um recipiente de volume V com N
molculas dentro, como mostra a Fig. 7.4. Supondo que o movimento das
molculas perfeitamente isotrpico, podemos dizer que a velocidade mdia
a mesma nas direes x, y e z, isto , v
x
= v
y
= v
z
. Assim, num dado intervalo
de tempo t podemos imaginar que n molculas caminham para a superfcie
com velocidade v
z
.







Fig. 7.4 - Presso que um gs contido numa caixa.
O nmero de molculas que atingiro a rea A num intervalo de
tempo t :
A t v
V
N
2
1
A t v
2
1
n
x x
= =
onde o fator
2
1
surge pelo fato de termos metade das molculas caminhado
para a esquerda. Supondo que a coliso com a superfcie completamente
elstica, cada molcula transfere 2mv
x
de momentum. Isto quer dizer que no
tempo t, o impulso dado por:
t A mv
V
N
mv 2 . n I
2
x x
= =
e assim a fora mdia agindo sobre a parede :
x
v
rea A





Colises

S. C. Zilio e V. S. Bagnato Mecnica, calor e ondas


123
A m v
v
N
t
I
F
2
0 m
=

=
A grandeza presso definida como fora por unidade rea e portanto:
m v
V
N
A
F
P
2
0
m
= =
Existe um teorema, que veremos no futuro, chamado teorema da
equiparao de energia que diz o seguinte: energia contida em cada grau de
liberdade do sistema est associada uma quantidade
2
1
K
B
T, onde K
B

chamada constante de Boltzmann e T a temperatura em graus Kelvin. Por
grau de liberdade queremos nos referir . translao, rotao ou vibrao de
molcula. Como a molcula que estamos considerando ideal, isto , uma
massa pontual sem estrutura interna, o nico tipo de energia que ela pode ter
a translacional (cintica). Para o grau de liberdade x, a energia :
T K
2
1
mv
2
1
K
B
2
x x
= =
Portanto,
PV = NKT
que conhecida como equao dos gases ideais.
7.3 Coliso e conservao de momentum
Vamos considerar um sistema de partculas na ausncia de foras
externas ( ) 0 F
ext
i
=
r
. Neste caso, existem apenas foras internas, mas j
vimos no captulo anterior que ( ) 0 F
int
i
=
r
, j que os pares ao-reao
cancelam-se mutuamente. Portanto, como a fora total nula, o impulso
tambm o e, conseqentemente, o momentum total do sistema conservado.

= =
=
N
1 i
i 2
N
i i
i 1
p p
r r







Colises


S. C. Zilio e V. S. Bagnato Mecnica, calor e ondas


124
Por outro lado, se olharmos para o centro de massa, veremos que sua
velocidade constante, pois a fora externa nula. Com relao energia
cintica do sistema, podemos separ-la em duas partes distintas:
r
2
CM
K MV
2
1
K + =
onde M a massa total do sistema. O 1
o
termo, que permanece constante para
qualquer tipo de coliso a energia cintica do centro de massa. O 2
o
termo K
r

corresponde energia dos componentes do sistema com relao ao centro de
massa. De acordo com a variao de K
r
a coliso pode ser classificada como:
coliso perfeitamente elstica - K
r
no se altera; e coliso perfeitamente
inelstica (plstica) - K
r
completamente dissipada. A maioria das colises
est entre estes dois extremos.
a) Coliso perfeitamente elstica
Neste tipo de coliso, tanto o momentum como a energia cintica so
conservados. Vamos considerar um sistema de dois corpos antes e depois da
coliso como mostra a Fig. 7.5







Fig. 7.5 - Coliso perfeitamente elstica.
Pelas leis de conservao de momentum e energia as equaes para o
problema so:

2
f 2 2
2
f 1 1
2
i 2 2
2
i 1 1
v m v m v m v m
2
1
2
1
2
1
2
1
+ = +
f 2 2 f 1 1 i 2 2 i 1 1
v m v m v m v m
r r r r
+ = +
m
1
m
2
v
1i
v
2i
antes

m
1
m
2
v
1f
v
2f
depois






Colises

S. C. Zilio e V. S. Bagnato Mecnica, calor e ondas


125
que podem ser resolvidas fornecendo os valores de
f 1
v
r
e
f 2
v
r
. Como exemplo,
vamos considerar o caso da coliso de dois corpos em uma dimenso, tendo
um deles velocidade inicial nula, conforme indicado na Fig. 7.6.




Fig. 7.6 - Coliso unidimensional elstica com corpo inicialmente em repouso.
Queremos encontrar
f 1
v

e
f 2
v

aps a coliso. Usando as equaes
vistas anteriormente,
f 2 f 1
mv Mv Mv + =
Eliminando
f 2
v

da equao de conservao de energia e substituindo
na de conservao de momentum, obtemos uma equao de 2
o
grau cuja
soluo :
m
M
m
M
f 1
1
v v
v
+

=
( )
m
M
f 2
1
v v
m
M
v
+
=
m

O sinal + em
f 1
v

e - em
f 2
v

fornece
f 1
v = v e
f 2
v = 0 que a
condio inicial do problema, sempre contida na soluo pois satisfaz
conservao de momentum e energia. A soluo que nos interessa aquela
que ocorre aps a coliso, que dada por:
( )
v
1
1
v
m
M
m
M
f 1
+

=
v
1
2
v
m
M
m
M
f 2
+
=
A primeira observao que podemos fazer que:
( )
i 1 i 2 f 1 f 2
v v v v =
m

M

v
1i
= v
v
2i
= 0






Colises


S. C. Zilio e V. S. Bagnato Mecnica, calor e ondas


126
isto , a velocidade relativa de um corpo em relao ao outro apenas
invertida na coliso. Este um resultado sempre vlido em colises elsticas
unidimensionais.
A seguir vamos analisar alguns casos particulares de colises elsticas
em uma dimenso:
M = m) Neste caso,
f 2
v = v e
f 1
v = 0 de forma que existe uma troca de
velocidades.
M/m >>1)
f 1
v v e
f 2
v 2v, existindo um grande impulso na massa menor.
M/m <<1)
f 1
v -v e
f 2
v 0, ocorrendo somente uma reflexo do corpo mais
leve.
b) Colises perfeitamente inelsticas
Neste caso a energia cintica no conservada embora o momentum o
seja. Como exemplo, vamos considerar o tipo de coliso os corpos ficam
unidos aps o choque, como mostrado na Fig. 7.7.




Fig. 7.7 - Coliso perfeitamente inelstica.
Usando a conservao de momentum,
( )
2 1
i 2 2 i 1 1
f f 2 1 i 2 2 1 1
M M
v M v M
v v M M v M v M
+
+
= + = +
que justamente a velocidade do centro de massa do sistema. Vamos a seguir
calcular a quantidade de energia dissipada na coliso. Calculando as energias
antes e depois da coliso temos:
M
1
v
1i
v
2i

M
2 M
1
v
f

M
2





Colises

S. C. Zilio e V. S. Bagnato Mecnica, calor e ondas


127
( )
( )
2
i 2 i 1
2 1
2 1
i f
2 1
2
i 2 2 i 1 1
f
2
i 2 2
2
i 1 1 i
v v
M M
M M
K K K
M M
v M v M
K
v M v M K
2
1
2
1
2
1
2
1

+
= =
+
+
=
+ =

que sempre negativo, mostrando haver perda de energia. Para o caso em que
v
2i
= 0 temos
2 1
2
i
M M
M
K
K
+
=


que um resultado importante para estimarmos perda de energia em
experimentos com pndulo balstico.
Exerccios
1 - Dois carrinhos com massas m
1
e m
2
e velocidades v
1
e v
2
chocam-se
elasticamente (a energia se conserva). Sabendo-se que o momentum do
sistema se conserva durante a coliso, calcule as velocidades dos carros
aps o choque.
2 - Duas bolas A e B de massas diferentes colidem. A est inicialmente em
repouso e B tem velocidade v. Depois do choque B tem velocidade v/2 e
se move perpendicular direo do movimento inicial. Determine a
direo do movimento de A aps a coliso. Qual a variao da energia
devido coliso?
3 - Considere o pndulo balstico mostrado na Fig. 7.8. A massa m tem
velocidade inicial v e se une massa M aps a coliso. Determine o
ngulo mximo atingido pelo pndulo.







Colises


S. C. Zilio e V. S. Bagnato Mecnica, calor e ondas


128










Fig. 7.8 - Pndulo balstico.
4 - Uma bala de massa m disparada com velocidade v contra um pndulo
balstico de massa M. A bala atravessa o pndulo e emerge com
velocidade ()v. (a) calcular a altura mxima de elevao do pndulo, (b)
calcular a energia dissipada quando a bala atravessa o pndulo.
5 - Duas partculas, de massas m e M, deslizam sem atrito ao longo do eixo x
com velocidades iniciais v
0
e V, e colidem. (a) qual a velocidade do
centro de massa? (b) qual o momentum final de cada partcula no
referencial do centro de massa? (c) qual a velocidade de cada partcula
no referencial do laboratrio?
6 - Lana-se um corpo de massa m=0,2 kg com velocidade v
a
=12 m/s sobre
um carrinho de massa M=1,8 kg, que tem velocidade V=2 m/s. Sabendo-
se que existe atrito entre o corpo e o carro, mas no entre o carro e a pista
pergunta-se: a) qual a velocidade final do corpo e do carro? b) qual a
energia dissipada pelo atrito?
7 - Um bloco de massa 3m repousa sobre uma mesa sem atrito, preso parede
por uma mola de constante k. Uma bala de massa m disparada
horizontalmente contra o bloco, como mostra a Fig. 7.9 e engasta nele.
Observa-se que a mxima deformao da mola x. Encontre: (a) a
L
M+m

L
M
m
v
r





Colises

S. C. Zilio e V. S. Bagnato Mecnica, calor e ondas


129
velocidade inicial da bala, (b) o impulso transferido pela bala ao bloco, (c)
a frao de energia E/E
i
perdida na coliso.
8 - Uma partcula com velocidade inicial v
0
colide com uma outra em repouso
e desviada de um ngulo . A sua velocidade, depois da coliso v. A
segunda partcula recua e a direo de seu movimento faz um ngulo
com a direo inicial do movimento da primeira., como mostra a Fig.
7.10. Mostrar que:


=
cos v v
vsen
tan
0
. Para obter este resultado,
necessrio admitir que a coliso elstica ou inelstica?





Fig. 7.9 Fig. 7.10
9 - Um homem de massa m est sobre um carrinho de massa M, que rola num
terreno plano sem atrito, com velocidade v
0
. Num certo instante ele pula
para o cho com velocidade v
0
/2 em relao ao solo e na direo oposta ao
movimento do carro. (a) qual a velocidade do centro de massa do
sistema antes e depois do pulo? (b) qual a velocidade do carrinho depois
do pulo? (c) transforme todas as velocidades para o referencial do centro
de massa e indique num diagrama as velocidades iniciais e finais do
homem e do carro neste referencial. (d) que energia o homem dissipou no
pulo? (e) qual a velocidade do centro de massa depois que o homem
atinge o cho e fica parado?
10 - Uma mola de massa desprezvel e constante k est comprimida de uma
quantia x entre dois corpos de massa m
1
e m
2
. A mola no est presa aos
corpos, mas sua compresso mantida inicialmente por um barbante sem
massa, conforme mostra a Fig. 7.11. O sistema todo est se movendo
v
0
k
3m

m

m
1
m
2
m
1
m
2
v
0


v
1
v
2






Colises


S. C. Zilio e V. S. Bagnato Mecnica, calor e ondas


130
sobre uma mesa sem atrito, com velocidade V
CM
. Subitamente o barbante
se rompe. Calcule as velocidades finais v
1
e v
2
das massas.


Fig. 7.11
11 - Dois corpos de massas m
1
e m
2
caminham para a direita com velocidades
v
1
e v
2
, tal que v
1
>v
2
, conforme mostra a Fig. 7.12. O corpo 1 possui uma
mola de constante de mola k, que comprimida durante a coliso. Qual
ser a mxima deformao da mola?



Fig. 7.12



m
1
m
2
v
1
v
2

V
cm
k
m
1
m
2
v
2
k
m
1
m
2
v
1





Dinmica do corpo rgido

S. C. Zilio e V. S. Bagnato Mecnica, calor e ondas


131






8.1 Introduo
Um corpo rgido constitui-se de um conjunto de partculas (massas
pontuais) dispostas de tal forma que as distncias relativas entre elas so fixas.
As leis da mecnica do ponto continuam vlidas se considerarmos somente o
movimento do centro de massa do corpo rgido. Alm deste movimento
translacional descrito pelas leis de Newton, o corpo tambm pode sofrer uma
rotao ao redor de um eixo, que pode eventualmente passar pelo seu centro
de massa. Assim, para especificarmos com exatido a posio de um corpo
rgido, necessrio conhecermos o movimento de seu centro de massa e o
ngulo de rotao , como mostra a Fig. 8.1.







Fig. 8.1 Movimento de um corpo rgido combinando translao e rotao.
8.2 Rotao em torno de um eixo fixo
Vamos rever algumas grandezas fsicas necessrias para descrever a
rotao de um corpo rgido ao redor de um eixo fixo. Consideremos um ponto
localizado a uma distncia r do eixo de rotao de tal maneira que seu vetor
8
DINMICA DO
CORPO RGIDO

CM






Dinmica do corpo rgido


S. C. Zilio e V. S. Bagnato Mecnica, calor e ondas


132
posio forma um ngulo com a linha tracejada horizontal, conforme mostra
a Fig. 8.2.







Fig. 8.2 Rotao de um corpo rgido em torno de um eixo fixo.
A velocidade angular do corpo definida como sendo a variao
temporal do ngulo :
( )
dt
d
t

= [rad/s]
Durante um intervalo de tempo dt, o ponto descreve um arco ds = rd
= rdt, onde na ltima igualdade usamos a definio de dada acima. A
velocidade tangencial corresponde variao de ds com o tempo e assim,
( ) ( ) t r
dt
ds
t v = = [ m/s]
Como deixamos explcito acima, (t) pode depender do tempo e sua
variao define a acelerao angular :

dt
d
dt
d
2
2

= [rad/s
2
]
Evidentemente, neste caso temos tambm acelerao tangencial e
como r constante durante a rotao (corpo rgido), ela definida como:
r
dt
d
r
dt
s d
a
2
2
=

= = [m/s
2
]
Devemos nos lembrar que como este ponto descreve um crculo,
tambm sofre a acelerao centrpeta dada por:
r

d
ds





Dinmica do corpo rgido

S. C. Zilio e V. S. Bagnato Mecnica, calor e ondas


133
r
r
v
a
2
2
c
= =
O conjunto de equaes acima tem uma forma similar ao encontrado
no caso do movimento retilneo se substituirmos , e por x, v e a. No caso
em que constante (rotao uniformemente acelerada) obtemos por
integrao direta que:
=
0
+ t
=
0
+
0
t +
2
1
t
2

( )
0
2
0
2
2 + =
As grandezas , e que caracterizam o movimento rotacional
tambm podem ser representadas vetorialmente. A direo neste caso a do
eixo em torno do qual o corpo roda. O sentido definido pela regra da mo
direita, colocando-se os dedos na direo em que aumenta. O polegar
coincide ento com o eixo de rotao e indica o sentido do vetor
r
.
Estritamente falando,
r
s pode ser considerado vetor quando o eixo de
rotao no muda ou quando |
r
| 0. Caso contrrio, ele no obedece a regra
de comutao dada por:
1 2 2 1
+ = +
r r r r

Como exemplo, consideremos duas rotaes consecutivas de um
paraleleppedo, ambas de 90
0
, conforme mostra a Fig. 8.3. Se a 1
a
rotao se
der em torno de x e a 2
a
em torno de y, a posio final do corpo ser diferente
daquela obtida se a ordem das rotaes forem invertidas. Isto demonstra a no
comutatividade de
r
. Por outro lado,
r r
e comutam porque envolvem
vetores infinitesimais ( ) dt / d e dt / d = =
r r
r
r
.








Dinmica do corpo rgido


S. C. Zilio e V. S. Bagnato Mecnica, calor e ondas


134





















Fig. 8.3 No comutatividade da rotao de um corpo rgido para eixo no fixo.
8.3 Energia rotacional e momento de inrcia
Se imaginarmos um corpo rgido rodando em torno de um eixo fixo,
dividido num nmero muito grande de partes, cada uma com massa m
i
,
veremos que a energia cintica de cada uma destas partes :
2 2
i i
2
i i i
r m v m K
2
1
2
1
= =
Posio final
y
90
x
90
y
90
x
90





Dinmica do corpo rgido

S. C. Zilio e V. S. Bagnato Mecnica, calor e ondas


135
pois sabemos que a velocidade tangencial =
i i
r v e que a velocidade
angular a mesma para todos os elementos de massa
i
m . A energia
cintica total do corpo rgido pode ser encontrada somando-se as energias
individuais de cada componente do sistema:
2
i
2
i i
i
i
r m K K
2
1
|

\
|
= =


O termo entre parnteses conhecido como momento de inrcia,
denotado por I. A energia cintica de rotao de um corpo rgido pode ento
ser escrita como:
2
I K
2
1
=
com
2
i
i
i
r m I

= . A definio do momento de inrcia dada acima vlida


no caso em que o corpo composto por partculas discretas. Se tivermos uma
distribuio contnua de massa devemos fazer m
i
dm e transformar a soma
em integral:
dm r I
V
2

=
Comparando a energia cintica de rotao com a de translao, vemos
que o momento de inrcia faz o papel da massa inercial e a velocidade angular
faz o papel da velocidade tangencial.
O momento de inrcia depende fortemente da distribuio de massa
com relao a um eixo especfico, em torno do qual o corpo roda. Desta
forma, um mesmo corpo rgido pode ter vrios momentos de inrcia que
dependem dos eixos de rotao escolhidos. Para uma dada velocidade angular
, como o momento de inrcia depende do eixo de rotao, a energia cintica
tambm depender. Como exemplo, vamos considerar trs massas pontuais
(portanto, discretas), ligadas por hastes rgidas sem massa, de maneira a
formar o tringulo equiltero mostrado na Fig. 8.4. O momento de inrcia em
relao ao eixo (1) mostrado na Fig. 8.4 dado por:






Dinmica do corpo rgido


S. C. Zilio e V. S. Bagnato Mecnica, calor e ondas


136
( )
2
2
2
i i 1
mL
2
L
m 2 r m I
2
1
= = =








Fig. 8.4 Corpo rgido formado por um tringulo equiltero com massas no vrtice.
Queremos, a seguir, calcular o momento de inrcia com relao a um
eixo perpendicular ao plano da figura e passando pelo centro de massa do
tringulo. A distncia de cada massa a este eixo de 3 / L d = , de forma que
teremos:
2 2
CM
mL md 3 I = =
Por sua vez, o momento de inrcia em relao a um eixo
perpendicular ao plano da figura e passando por uma das massas :
2
2
mL 2 I =
O momento de inrcia apresenta uma srie de propriedades
interessantes que muitas vezes simplificam a realizao dos clculos. Veremos
estas propriedades a seguir.
Teorema dos eixos paralelos (Teorema de Steiner)
Se conhecermos o momento de inrcia de um corpo em relao a um
eixo passando pelo centro de massa, podemos facilmente encontrar o
momento de inrcia em relao a um eixo paralelo a ele, como mostra a Fig.
8.5. O momento de inrcia em relao ao eixo passando pelo centro de massa
:
dm r I
2
CM

=
m
m
m
L
L
L





Dinmica do corpo rgido

S. C. Zilio e V. S. Bagnato Mecnica, calor e ondas


137
enquanto que em relao ao eixo paralelo,
dm ' r I
2

=










Fig. 8.5 Figura usada para a demonstrao do teorema dos eixos paralelos.
Entretanto, como r = h r podemos escrever:
( )

+ = = rdm h 2 dm h dm r dm r h I
2 2
2

O ltimo termo nos d a distncia do C.M. ao eixo passando pelo C.M. que,
obviamente, nula. Logo,
2
CM
Mh I I + =
No exemplo das trs massas formando o tringulo equiltero visto
anteriormente na Fig. 8.4, encontramos: I
CM
= mL
2
. Tomando um eixo
paralelo que passa por uma das massas temos 3 / L h = e M = 3m. Pelo
teorema de Steiner encontramos:
2
2
2 2
CM 2
mL 2
3
L
m 3 mL Mh I I = + = + =
que coincide com o resultado obtido anteriormente.
I
CM
C.M.
r r
I
h
dm






Dinmica do corpo rgido


S. C. Zilio e V. S. Bagnato Mecnica, calor e ondas


138
Teorema dos eixos perpendiculares
Este teorema vlido para corpos planos, do tipo placa, mostrado na
Fig. 8.6.









Fig. 8.6 Figura usada para a demonstrao do teorema dos eixos perpendiculares.
Os momentos de inrcia com relao aos eixos x, y e z so dados por:
dm y I
2
x

=
dm x I
2
y

=
( )

+ = = dm y x dm I
2 2 2
z

Portanto,
y x z
I I I + = , isto , a soma dos momentos de inrcia de dois eixos
perpendiculares contidos no plano do corpo igual ao momento de inrcia em
relao a um 3
o
eixo perpendicular ao plano do corpo e passando pela
interseo dos dois primeiros. Quando este 3
o
eixo passa pelo centro de massa
denominado de eixo polar.
Vamos a seguir considerar alguns exemplos de clculo de momento
de inrcia.
a) Massa pontual Este caso, mostrado na Fig. 8.7, o mais simples e leva a
um momento de inrcia dado por: I = md
2
,. Onde d a distncia da massa ao
eixo.
x
x
y
y

z





Dinmica do corpo rgido

S. C. Zilio e V. S. Bagnato Mecnica, calor e ondas


139




Fig. 8.7 Massa pontual rodando em torno de um eixo.
b) Arco de raio R - O clculo do momento de inrcia de um anel de raio R
em relao a um eixo passando pelo centro de massa e perpendicular ao plano
do anel (eixo polar) tambm bastante simples porque cada elemento de
massa dm est mesma distncia r = R = constante do eixo de rotao,
conforme indicado na Fig. 8.8. Portanto,

= = =
2 2 2
MR dm R dm r I






Fig. 8.8 Anel rodando em torno do eixo polar.
c) Barra delgada de comprimento L - A barra mostrada na Fig. 8.9 roda em
torno de um eixo perpendicular passando pelo centro de massa. A densidade
linear de massa = M/L, de forma que o elemento infinitesimal possui dm =
dx = (M/L) dx. Como L/2 x L/2, temos:
12
ML
3
x
dx x I
2 2 / L
2 / L
3
2 / L
2 / L
2
L
M
L
M
=
|

\
|
=
|

\
|
=






m
d
d
ds
R
M






Dinmica do corpo rgido


S. C. Zilio e V. S. Bagnato Mecnica, calor e ondas


140






Fig. 8.9 Barra delgada rodando em torno do centro de massa.
d) Barra delgada com eixo passando pela extremidade Conforme
indicado na Fig. 8.10, a nica diferena do caso anterior o intervalo de
variao de x, que agora : 0 x L. Desta forma, a integral fica:
( ) ( ) ( )
2
CM
2
L
0
3 L
0
2
2
L
M I
3
ML
3
x
dx x I
L
M
L
M
+ = = = =


onde nesta ltima passagem utilizamos o teorema dos eixos paralelos.





Fig. 8.10 - Barra delgada rodando em torno da ponta.
e) Anel de raio r com eixo no plano do anel - A densidade linear de massa
para o anel = M/2R, de forma que o elemento de massa mostrado na Fig.
8.11 possui dm = Rd = (M/2)d. Sua distncia ao eixo y dada por:
= sen R x e, portanto,

=


d
2
) 2 cos 1 (
2
MR
d sen R
2
M
I
2
0
2
2 2
2
0
y

x
2
y
I
2
MR
I = =
pela prpria simetria do problema. Por outro lado, vemos que I
x
+ I
y
= MR
2
=
dx
L
M
L
M





Dinmica do corpo rgido

S. C. Zilio e V. S. Bagnato Mecnica, calor e ondas


141
I
z
como era de se esperar pelo teorema dos eixos perpendiculares.




Fig. 8.11 Anel rodando em torno de um eixo contido no seu plano.
f) Disco de raio R - A densidade superficial de massa de um disco de raio R
= M/R
2
. Conforme vemos na Fig. 8.12, um disco pode ser considerado
como composto de um grande nmero de anis concntricos, de raio r e
espessura dr. A massa elementar de cada um destes anis dada por: dm =
dA, onde dA = 2rdr.







Fig. 8.12 Disco rodando em torno do eixo polar.
O momento de inrcia polar do anel dI
z
= dm r
2
. Para calcularmos o
momento de inrcia do disco devemos somar as contribuies de todos os
anis concntricos compreendidos entre 0 e r. Assim,
( )
2
MR
dr r
R
M 2
rdr 2
R
M
r I
2 R
0
3
2 2
R
0
2
z
= =

=


Pelo teorema dos eixos perpendiculares,
4
MR
I I I
2
x y z
2
1
= = =
M
dm
y
x
x

R
r






Dinmica do corpo rgido


S. C. Zilio e V. S. Bagnato Mecnica, calor e ondas


142
g) Cilindro macio de raio R - A densidade volumtrica de massa para um
cilindro de raio R e altura H dada por: = M/V = M/R
2
H. Por outro lado,
um cilindro pode ser encarado como se fosse uma pilha de inmeros discos
finos de raio R e altura dz, como visto na Fig. 8.13. A massa de cada disco
dm = R
2
dz = Mdz/H. O momento de inrcia polar de cada disco elementar
dI
z
= dm r
2
, como calculado no tem anterior. Ao se somar as
contribuies de todos os discos fazemos uma integral onde z varia de 0 a H.
Assim,
2
H
0
2
MR dz dm R I
2
1
H
2
MR
2
1
2
1
= = =


Neste caso no podemos usar o teorema dos eixos perpendiculares
para encontrar I
z
ou I
y
, pois o corpo no plano.








Fig. 8.13 Cilindro macio rodando em torno do eixo de simetria.
8.4 Dinmica da rotao em torno de um eixo fixo
Da mesma forma que a translao, a rotao causada por um agente
externo produzindo uma fora. Consideremos um corpo rgido rodando com
velocidade angular em torno de um eixo fixo O, conforme mostra a Fig.
8.14. Existe uma fora F
r
aplicada a uma distncia r
r
do eixo. Queremos
encontrar o trabalho realizado por esta fora.



H
z
dz





Dinmica do corpo rgido

S. C. Zilio e V. S. Bagnato Mecnica, calor e ondas


143








Fig. 8.14 - Trabalho realizado por uma fora durante a rotao de um corpo rgido
Durante um intervalo de tempo dt, o corpo roda a um ngulo
dt d =
r
r
, de modo que o ponto especificado pelo vetor ( ) t r
r
percorrer uma
distncia s d
r
e o trabalho realizado pela fora F
r
dado por:
s d . F dW
r
r
=
Usando a definio vetorial de
r
d e
r
d , podemos escrever s d
r

como sendo
dt r r d s d
|

\
|
= =
r r r
r
r

e assim,
( )dt r . F dW
r r
r
=
Entretanto, como ( ) ( ) F r . r . F
r
r r r r
r
= podemos escrever a potncia
fornecida pela fora F
r
como sendo:
( ) F r .
dt
dW
P
r
r r
= =
Para que o trabalho (ou potncia) no seja nulo, F
r
deve ter uma
componente paralela a s d
r
e, consequentemente, perpendicular a r
r
.
Comparando com o movimento translacional vemos que
r
faz o papel de v
r
e
F r
r
r r
= faz o papel da fora.
r
denominado de torque da fora F
r
em
relao ao ponto O.
dt) (t r +
r
(t) r
r
s d
r
F
r
O






Dinmica do corpo rgido


S. C. Zilio e V. S. Bagnato Mecnica, calor e ondas


144
Vamos a seguir encontrar uma relao entre
r
e
r
que seja
correspondente 2
a
lei de Newton. J vimos que
2
2
1
I K = , portanto:
=

=
|

\
|
= =
r r
r
r
.
dt
d
. I
2
I
dt
d
dt
dK
P
2

Logo,
=

=
r
r
r
I
dt
d
I
No caso de termos vrias foras produzindo vrios torques,
escrevemos:
=

=
r r
I
N
1 i
i

Como aplicao direta desta lei de rotao, vamos considerar o
seguinte exemplo: uma corda de densidade linear de massa est enrolada em
uma roldana de massa M e raio R, que pode rodar livremente em torno de um
eixo, como mostra a Fig. 8.15. O momento de inrcia da roldana
2
2
1
0
MR I = .








Fig. 8.15 Corda desenrolando de uma roldana.
Se um comprimento l
0
est inicialmente desenrolado e a velocidade
angular inicial
0
nula, qual ser a acelerao e a velocidade da corda como
funo de l? Nesta situao, a massa pendente m(l) = l e a 2
a
lei de
Newton para a corda :
ma = mg T la = lg - T
O
M
R
l





Dinmica do corpo rgido

S. C. Zilio e V. S. Bagnato Mecnica, calor e ondas


145
O torque realizado pela corda sobre a polia dada por: = RT de
modo que,
dt
d
I I RT
0 0

= =
Por outro lado, como a corda no desliza sobre a roldana podemos escrever v
= R e a = R. Assim,
2
Ma
R
a I
T
R
a
I RT
2
o
0
= = =
Substituindo este resultado na equao para a corda obtemos:
2
M
g
a
+

=
l
l

Para o clculo da velocidade fazemos:
dt
d
d
dv
dt
dv
a
l
l
= =
Como dl/dt = v, temos:
2
M 2
1
g
d
dv
d
dv
v
2
+

= =
l
l
l l

( )
|

\
|
+
+

=
+

0
0
2
2 / M
2 / M
2 / M 2
1
n
2
gM
g
d g
v
0 l
l
l l l
l
l l
l
l

Este resultado mostra que se M = 0 a corda estar em queda livre. Porm, se
M 0 parte da energia gasta para aumentar a velocidade angular da polia.
8.5 Equilbrio esttico de um corpo rgido
Como vimos at agora, um corpo rgido pode possuir movimentos
rotacional e translacional, aos quais estaro associados respectivamente um
torque e uma fora externa. Dizemos que um corpo est em equilbrio quando
seu movimento rotacional-translacional no se alterar no tempo (a = = 0).
Em particular, quando h ausncia de movimento (v = = 0) dizemos que o






Dinmica do corpo rgido


S. C. Zilio e V. S. Bagnato Mecnica, calor e ondas


146
corpo est em equilbrio esttico (repouso). A condio necessria para que
ocorra esta situao :

= 0 F
r
(translacional)

= 0
r
(rotacional)
Vamos analisar alguns exemplos onde o equilbrio esttico se
manifesta. Inicialmente, vamos considerar uma escada encostada numa parede
sem atrito, como mostra a Fig. 8.16. Sabendo-se que a massa da escada M, o
comprimento L e o atrito com o cho , queremos calcular o menor ngulo
para o qual a escada no escorrega.








Fig. 8.16 Escada apoiada numa parede sem atrito.
Como as foras e torques totais so nulos:
Mg N
2
= 0
N
1
F
at
= N
1
- N
2
= 0
0 sen L N cos Mg
1
2
L
= (torque em torno de O)
Deste conjunto de equao encontramos uma expresso para o ngulo
mnimo :
|

\
|

=
2
1
tg ou
2
1
tg
1 -

Um segundo exemplo o de uma caixa de altura h e base L, colocada

F
at
Mg
N
1
N
2
L






Dinmica do corpo rgido

S. C. Zilio e V. S. Bagnato Mecnica, calor e ondas


147
sobre uma mesa inclinada com atrito, conforme mostra a Fig. 8.17. Se a caixa
no desliza, qual a mxima inclinao antes dela rolar?
Enquanto o torque em torno de O devido fora peso estiver apontado
no sentido indo para o interior da pgina (horrio) no haver rotao. Quando
ele inverte de sentido (anti-horrio), haver rotao. Existe uma situao em
que o torque nulo, o que define o ngulo crtico
c
. Se este ngulo for
diminudo, o torque horrio (no roda) e se for aumentado anti-horrio
(roda). Esta situao crtica ocorre quando r
r
paralelo a ) 0 F r ( F = =
r
r r
r
,
isto , quando a fora peso passar pelo ponto de apoio O. Nesta condio,
h
L
2 h
2 L
tg
c
= =





Fig. 8.17 Caixa num plano inclinado com atrito.
8.6 Acelerao constante
Vamos nesta seo considerar o caso em que existe uma fora externa
agindo sobre o corpo rgido, capaz de produzir uma acelerao constante. Em
geral, existe nesta situao um movimento combinado de rotao e translao.
Estes movimentos podem ser analisados independentemente e posteriormente
relacionados por uma equao do tipo = a/R ou = v/R.
a) I-i Considere um i-i de massa M e raio R (na forma de um disco
como o mostrado na Fig. 8.18) que solto a partir do repouso. Queremos
encontrar (t) e v(t).





h
L
O






Dinmica do corpo rgido


S. C. Zilio e V. S. Bagnato Mecnica, calor e ondas


148







Fig. 8.18 I-i solto a partir do repouso.
As equaes para foras e torque so dadas respectivamente por:
Mg T = Ma
R
I
T I TR

=
onde I = MR
2
/2, a a acelerao do centro de massa do disco e o torque
calculado em relao a este ponto. Nas duas equaes acima temos 3
incgnitas (T, a e ), mas uma nova equao envolvendo a e pode ser
encontrada. Se estivermos no centro de massa veremos o ponto P subindo com
acelerao a e o disco rodando com acelerao angular . Assim, fcil
notarmos que: dt Rd R a = = , onde ento,

dt
d
MR
dt
d
R
I
Mg

=


Mg
dt
d
2
1
1 MR
dt
d
R
I
MR =

\
|
+ =

\
|
+
Logo:
R
g
3
2
dt
d
=

de onde se obtm
R
gt
3
2
) t ( = e consequentemente
( ) ( ) gt
3
2
t R t v = =
b) Carretel Um carretel de raio interno r e raio externo R encontra-se sobre
uma mesa com atrito como indicado na Fig. 8.19. Ele puxado por um fio que
produz uma fora F
r
fazendo um ngulo com a horizontal. Observa-se que
T
Mg
R
P





Dinmica do corpo rgido

S. C. Zilio e V. S. Bagnato Mecnica, calor e ondas


149
para <
c
(ngulo crtico) o carretel rola sem deslizar na direo da fora F
r
e
para >
c
ele roda no sentido oposto. Queremos encontrar o valor de
c
.
Vamos supor que o carretel ande na direo de F
r
. As equaes para a
translao do centro de massa so:
+ = sen F N Mg
+ = cos F F Ma
at

e para o torque em torno do centro de massa:









Fig. 8.19 - Carretel puxado sobre uma mesa com atrito.
=
0 at
I Fr R F
onde supusemos que o carretel acelerado para a direita. Usando
2
2
1
0
MR I e R a = = , temos para a equao da translao:
= = cos F MR cos F Ma F
at

Substituindo F
at
na equao da acelerao angular obtemos:
= +
0
2
I Fr cos FR MR
( ) ( ) r cos R F MR I
2
0
= +
( )
2
2
3
MR
r cos R
F

=

r
R
Mg
F
at
N
F






Dinmica do corpo rgido


S. C. Zilio e V. S. Bagnato Mecnica, calor e ondas


150
Quando > 0 o carretel roda para a direita (sentido horrio) e quando
< 0 ele roda para a esquerda (sentido anti-horrio). O ngulo crtico ocorre
para = 0 isto :
|

\
|
= =

R
r
cos 0 r cos R
1
c c

c) Disco sobre uma mesa sem atrito A Fig. 8.20 mostra um disco de massa
M e raio R, que est deitado sobre uma massa sem atrito. Ele puxado por um
corpo de massa m atravs de um fio enrolado ao seu redor. Se o sistema
solto a partir do repouso, qual ser a velocidade do centro do disco e a tenso
na corda?







Fig. 8.20 - Disco puxado sobre uma mesa sem atrito.
A nica fora agindo sobre o disco a tenso na corda. As equaes
para a translao e rotao do corpo so dadas respectivamente por:
dt
dv
M Ma T = =

dt
d
I I TR
0 0

= =
com
2
0
MR I
2
1
= . Para o corpo de massa m temos apenas a translao,
descrita pela equao:
( ) R a m ' ma T mg + = =
R
M
m





Dinmica do corpo rgido

S. C. Zilio e V. S. Bagnato Mecnica, calor e ondas


151
onde levamos em conta o vnculo a= a + R. Isto significa que a acelerao
do corpo m se deve tanto acelerao do centro de massa disco, como do fato
da corda estar sendo desenrolada com acelerao angular . Substituindo os
valores de a e obtidos anteriormente ficamos com:
M
mT 3
I
TR
M
T
m T mg
0
2
= |

\
|
+ =
Desta equao podemos tirar o valor de T como:
M / m 3 1
mg
T
M
m 3
1 T mg
+
= |

\
|
+ =
e voltando para a expresso da acelerao obtemos:
( )
m
M
m
M
3
gt
t v
3
g
M
T
dt
dv
a
+
=
+
= = =
enquanto que para a acelerao angular,
( )
) m / M 3 ( R
gt 2
t
3
1
R
g 2
M / m 3 1
1
MR
mg 2
MR
TR
m
M
2
2
1
+
=
+
=
+
|

\
|
= =

d) Disco puxado pelo centro de massa O disco visto na Fig. 8.21 no
patina devido existncia da fora de atrito. Neste caso, a = R e as equaes
de movimento so:
F F
at
= Ma
2
Ma
R
Ia
F R / Ia I R F
2
at at
= = = =
Logo, F - Ma/2 = Ma, que nos leva acelerao:
M
F
3
2
dt
dv
a = = e
velocidade: ( ) t t v
M 3
F 2
= .







Dinmica do corpo rgido


S. C. Zilio e V. S. Bagnato Mecnica, calor e ondas


152





Fig. 8.21 Disco puxado pelo centro de massa.
8.7 Momentum angular
Consideremos uma partcula descrita pelo vetor posio ) t ( r
r
,
movendo-se no espao com velocidade ( ) t v
r
, como mostra a Fig. 8.22. A 2
a
lei
de Newton para esta partcula dada por:
dt
p d
F
r
r
=






Fig. 8.22 Movimento tridimensional de uma partcula.
Vamos multiplicar vetorialmente os dois lados desta igualdade por ) t ( r
r
.
Desta forma teremos:
dt
p d
r F r
r
r
r
r r
= =
onde
r
o torque da fora F
r
em relao ao ponto O. Por outro lado, se
tomarmos a derivada do produto p r
r r
temos:
( ) v m v
dt
p d
r p
dt
r d
dt
p d
r p r
dt
d
r r
r
r r
r
r
r r r
+ = + =
O produto vetorial v v
r r
nulo (vetores paralelos) e assim:
R
F
F
at
v
r
r
r
O





Dinmica do corpo rgido

S. C. Zilio e V. S. Bagnato Mecnica, calor e ondas


153
( )
dt
p d
r p r
dt
d
r
r r r
=
Logo:
( )
dt
L d
p r
dt
d
r
r r r
= =
onde p r L
r r
r
= definido como momentum angular da partcula em relao
ao ponto O. Note que L
r
depende do ponto O considerado. Como vimos
anteriormente, ( ) dt / I d dt / Id I = = =
r r r r
. Esta ltima passagem s
vlida quando temos rotao em torno de um eixo fixo e neste caso I
constante. Ento,
p r L ou I L
r r
r
r
r
= =
Como dissemos, a primeira igualdade vlida quando I constante
(movimento em torno de um eixo) e muito empregada em clculos onde
corpos extensos so envolvidos. A segunda igualdade bastante importante
quando massas pontuais so envolvidas. Um exemplo interessante o de uma
partcula movendo-se em linha reta, como mostra a Fig. 8.23. Ela apresenta
momentum angular se sua trajetria no passar pela origem de r
r
. Vemos que:
k

sen rmv v m r p r L = = =
r r r r
r

Como rsen = b, temos k

mvb L =
r
.








Fig. 8.23 Movimento retilneo de uma partcula.
x
y
O

r
r
b
v m
r
trajetria






Dinmica do corpo rgido


S. C. Zilio e V. S. Bagnato Mecnica, calor e ondas


154
8.8 Torque e momentum angular de um sistema de
partculas
Consideramos um sistema composto de N partculas, cada uma
movendo-se com velocidade
i
v
r
medidas em relao a um ponto fixo O. O
momentum angular total do sistema dado por:

=
=
N
1 i
i
L L
r r

que por sua vez satisfaz a igualdade:
dt
L d
N
1 i
i
r
r r
= =

=

Ao calcularmos o torque total sobre o sistema de partculas, notamos
que o torque devido s foras internas nulo. Para provarmos este fato,
consideremos duas partculas constituintes do sistema mostradas na Fig. 8.24.






Fig. 8.24 Torque devido s foras internas..
As foras
2 1
F e F
r r
constituem um par ao-reao e pela 3
a
lei de
Newton sabemos que
2 1
F - F
r r
= . Assim, o torque devido a estas foras dado
por:
( )
1 2 1 2 2 1 1 2 1
F r r F r F r
r
r r
r
r
r
r r r
= + = +
Como ( )
2 1
r r
r r
est na direo da linha pontilhada que une 1 a 2, seu produto
vetorial com
1
F
r
nulo e assim conclumos que os torques devidos s foras
internas se cancelam aos pares. Portanto, a equao correta :
1
2
O
1 F
r
2 F
r
1 r
r
2 r
r





Dinmica do corpo rgido

S. C. Zilio e V. S. Bagnato Mecnica, calor e ondas


155
dt / L d
ext
r
r
=
Como vimos anteriormente, para o caso de rotao ao redor de um eixo fixo
temos =
r
r
I L e, portanto:
dt / d I
ext
=
r r

Como um primeiro exemplo, vamos calcular o momentum angular
para um disco girando ao redor de seu eixo de simetria como mostrado na Fig.
8.25. Tomando a i-sima massa m
i
do disco, temos:
k

v r m p r L
i i i i i i
= =
r r
r

Como v
i
= r
i
, segue que:
( ) ( ) k

r m k

r r m L
2
i i i i i i
= =
r









Fig. 8.25 Rotao de um disco em torno de seu eixo polar.
e assim, o momentum angular total dado por:
( ) = = = =

r
r r
I k

I k

r m L L
i
2
i i
i
i

onde, de acordo com a definio anterior

=
i
2
i i
r m I .
No caso que acabamos de ver, o eixo de rotao passa pelo centro de
massa. Caso ele esteja deslocado de uma distncia R do centro de massa,

r
i
v
r

i
r
r







Dinmica do corpo rgido


S. C. Zilio e V. S. Bagnato Mecnica, calor e ondas


156
como mostra a Fig. 8.26, o momento de inrcia ser dado por (teorema dos
eixos paralelos):
2
CM
MR I ' I + =
e o momento angular fica sendo:
( ) + = =
r r r
r
2
CM
MR I ' I ' L
A velocidade do centro de massa pode ser escrita vetorialmente como:
R V
CM
r
r
r
=










Fig. 8.26 Rotao de um disco em torno de um eixo paralelo ao eixo polar.
Assim, se tomarmos o duplo produto vetorial
( ) ( ) ( ) = =
2
R . R R R . R R R
r r r r
r
r
r
r

pois como
r
r
e R so perpendiculares temos 0 . R =
r
r
. Desta forma,
CM CM
P R L ' L
r r r r
+ =
de onde vemos que o momentum angular de um corpo rgido em relao
rotao em torno de um eixo O a soma do momentum angular em relao

r
r
r
C.M.
O





Dinmica do corpo rgido

S. C. Zilio e V. S. Bagnato Mecnica, calor e ondas


157
ao centro de massa com o momentum angular associado rotao do centro de
massa em torno do eixo O.
Vimos h pouco que a variao do momentum angular de um sistema
depende apenas do torque externo aplicado sobre ele. Vamos utilizar este
conceito para analisar a mquina de Atwood mostrada na Fig. 8.27, onde a
roldana tem massa M, raio R e momento de inrcia 2 / MR I
2
0
= . O
momentum angular total do sistema em relao ao eixo O a soma do
momentum da roldana (I
0
) com o das duas massas ( ). vR m vR m
2 1
+ Como
a corda no desliza sobre a roldana, R v = e, portanto:
( )vR m m
R
v
I L
2 1 0
+ + =












Fig. 8.27 Mquina de Atwood.
As foras externas agindo sobre o sistema so m
1
g
r
, m
2
g
r
e R
r
, mas
esta ltima no realiza torque pois est aplicada sobre o eixo de rotao
( 0 r =
r
). Tomando como positivo o sentido entrando no papel, encontramos o
torque como sendo:
( ) a
dv
dL
dt
dv
dv
dL
dt
dL
gR m m
1 2 ext
= = = =
O
g m
1
r
m
1
m
2
v
r
v
r
T
r
T
r
R
r
R
M
g m
2
r






Dinmica do corpo rgido


S. C. Zilio e V. S. Bagnato Mecnica, calor e ondas


158
Substituindo a expresso de L como funo de v obtemos:
( ) ( ) a R m m
R
I
gR m m
2 1
0
1 2
(

+ + =
( ) ( )
2
M
R
I
2 1
1 2
2
0
2 1
1 2
m m
g m m
m m
g m m
a
+ +

=
+ +

=
Para o caso M = 0, recuperamos o resultado obtido no Cap. 4. Podemos ainda
calcular as tenses T e T existentes na corda. Isolando m
1
e m
2
, obtemos:
( ) a g m T a m T g m
2 2 2
= =
( ) g a m ' T a m g m ' T
1 1 1
+ = =
Como o valor de a j foi determinado, encontramos:
( )
2 / M m m
g 2 / M m 2
m T
2 1
1
2
+ +
+
=
( )
2 / M m m
g 2 / M m 2
m ' T
2 1
2
1
+ +
+
=
No caso M = 0, T = T =
2 1
2 1
m m
g m m 2
+
, como j obtido anteriormente.
Isolando a corda, como sua massa nula, a fora total sobre ela
tambm o . Podemos ento calcular a fora tangencial exercida pela polia
sobre ela pois F + T- T = 0. Como m
2
> m
1
, temos T > T e
( )
2 / M m m
m m
g
2
M
' T T F
2 1
1 2
+ +

= =
8.9 Relao trabalho-energia rotacional
Da mesma forma que na translao, a variao da energia rotacional
de um corpo se deve realizao de trabalho por um agente externo, no caso,





Dinmica do corpo rgido

S. C. Zilio e V. S. Bagnato Mecnica, calor e ondas


159
o torque. J sabemos que
2
I K
2
1
= e a diferencial desta energia ser:
( )
dt
d
dt I d I dK

= =
Sabemos ainda que = = = I dt / d I e dt d . Logo:
dW d dK = =
Assim, torque que produz uma rotao d realiza uma quantidade de trabalho
dW = d. A variao da energia rotacional dada por:

= d K
8.10 Conservao do momentum angular
Quando o torque nulo, 0 dt L d
ext
= =
r
r
, o momentum angular
constante. Alm da conservao do momentum e da energia, vemos agora a
grandeza momentum angular tambm pode se conservar. Como = I L ,
conclumos que diminuindo I, aumenta ou vice-versa. Como casos prticos
podemos citar a patinao no gelo e saltos ornamentais. Vamos a seguir
analisar alguns exemplos tpicos de conservao de momentum angular.
a) Choque entre dois discos Considere um disco de momento de inrcia I
1

e velocidade angular
0
, rodando num eixo sem atrito passando por seu centro
de massa. Ele cai sobre outro disco de momento de inrcia I
2
, inicialmente
parado. Vamos calcular a velocidade angular final do conjunto usando
conservao do momentum angular.
( )
0
2 1
1
f f 2 1 0 1 f i
I I
I
I I I L L
|

\
|
+
= + = =
Este um choque do tipo inelstico. Vamos calcular a variao da
energia do sistema, dissipada em calor. As energias cintica inicial e final so
dadas respectivamente por:







Dinmica do corpo rgido


S. C. Zilio e V. S. Bagnato Mecnica, calor e ondas


160










Fig. 8.28 - Choque inelstico entre dois discos.
( )
1
2
i
1
2
0 1
2
0 1 i
I 2
L
I
I
I E
2
1
2
1
=

= =
( )
( ) | |
( ) ( )
2 1
2
f
2 1
2
f 2 1
2
f 2 1 f
I I
L
I I
I I
I I E
2
1
2
1
2
1
+
=
+
+
= + =
Como
f i
L L = vemos que E
f
< E
i
, isto e:
i
2 1
2
2
0
2 1
1 2
1 2 1
2
i
i f
E
I I
I
2 I I
I I
I
1
I I
1
2
L
E E E
+
=

+
=
|

\
|

+
= =
Este resultado anlogo ao do choque perfeitamente inelstico entre
duas massas, discutido no Cap.4.
b) Choque inelstico entre bala e barra delgada Uma barra, de massa M,
comprimento L e momento de inrcia I = ML
2
/12, tem seu centro fixo. Uma
bala de massa m e velocidade v disparada perpendicularmente barra e a
atinge a uma distncia l, ficando engastada. Se a haste est inicialmente em
repouso com que velocidade angular ela rodar?


0

r

I
1
I
2





Dinmica do corpo rgido

S. C. Zilio e V. S. Bagnato Mecnica, calor e ondas


161






Fig 8.29 - Choque inelstico entre bala e barra delgada.
O momentum angular inicial L
i
= mvl e o final :
( ) |

\
|
+ = + =
2
2
2
f
m
12
ML
m I L l l
Usando a conservao do momentum angular obtemos:
2
2
m
mv
12
ML
l
l
+
=
c) Velocidade dos planetas Os planetas girando em torno do sol conservam
momentum angular, pois a fora gravitacional central ( ) F // r
r
r
. Baseados
neste fato, queremos mostrar que o vetor descrevendo a posio do planeta
varre reas iguais em tempos iguais. Vamos considerar um planeta girando em
torno do Sol, como mostra a Fig. 8.30.






Fig. 8.30 Rotao de um planeta em torno do Sol.
L
M
m
v
r
l
Sol
dA
r
r






Dinmica do corpo rgido


S. C. Zilio e V. S. Bagnato Mecnica, calor e ondas


162
Durante um intervalo de tempo dt, o planeta desloca-se vdt e o vetor
posio r
r
varre uma rea dt v r dA
2
1
r r
= . Assim,
m 2
L
v m r
m 2
1
v r
dt
dA
2
1
r
r r r r
= = =
Como L
r
constante, dA/dt tambm o e os planetas varrem reas iguais em
tempos iguais. Esta lei foi deduzida por Keppler a partir de observaes
astronmicas.
8.11 Combinao de translao e rotao
Quando um corpo rgido executa simultaneamente os movimentos de
translao e rotao, torna-se em geral difcil descrever este movimento
combinado. O que se faz decompor o movimento em dois: o de translao
do centro de massa e o de rotao em torno dele. A anlise se simplifica
mediante o uso do seguinte teorema: O torque total em relao ao centro de
massa
CM

r
igual derivada temporal do momentum angular relativo a este
ponto. Matematicamente,
CM CM
L
dt
d
r
r
=
Para a demonstrao deste teorema vamos considerar o sistema de
partculas mostrado na Fig. 8.31.
CM
r
r
o vetor posio do centro de massa,
i
r
r

o da i-sima partcula relativo origem O e
iR
r
r
o vetor relativo ao centro
de massa. Estas coordenadas esto relacionadas atravs da expresso:
iR CM i
r r r
r r r
+ =
J vimos no Cap. 6 que o vetor
CM
r
r
dado por:
( )

= =
+ = =
N
1 i
iR CM i
N
1 i
i i CM
r r m r m r M
r r r r

= =
+ |

\
|
=
N
1 i
iR i CM
N
1 i
i
r m r m
r r








Dinmica do corpo rgido

S. C. Zilio e V. S. Bagnato Mecnica, calor e ondas


163










Fig. 8.31 Sistema de partculas sujeito a translao e rotao.
e, portanto,

= 0 r m
iR i
r
, como era esperado j que a massa est distribuda
compensadamente ao redor do centro de massa. A velocidade de i-sima
partcula dada por:
i CM
iR CM i
i
u v
dt
r d
dt
r d
dt
r d
v
r r
r r r
r
+ = + = =
onde
CM
v
r
a velocidade do centro de massa,
i
u
r
a velocidade relativa ao
centro de massa e 0 u m
i i
=

r
, que conseqncia direta da derivao de
. 0 r m
iR i
=

r
O momentum angular da partcula relativo ao ponto O :
( ) ( )
i CM iR CM i i i i i
u v r r m v m r L
r r r r r r
r
+ + = =
( ) ( ) ( ) ( )
i iR i CM iR i i CM i CM CM i i
u r m V r m u r m V r m L
r r
r
r r r
r
r
r
+ + + =
O momentum angular total do sistema

=
i
L L
r r
. Portanto,
( ) ( )
i
i
iR i CM iR i i i CM CM CM
u r m V r m u m r V M r L
r r
r
r r r
r
r
r
+ + + =


O primeiro termo o momentum angular do centro de massa relativo
ao ponto O, os dois termos entre parnteses so nulos e o ltimo o
momentum angular relativo ao centro de massa, isto ,
CM
x
y
z
cm r
r
ir r
r
i r
r






Dinmica do corpo rgido


S. C. Zilio e V. S. Bagnato Mecnica, calor e ondas


164
CM CM CM
P r L L
r
r
r r
+ =
O torque total externo agindo no sistema :
( )
i
N
1 i
iR
N
1 i
i CM i
N
1 i
iR CM i
N
1 i
i ext
F r F r F r r F r
r
r
r
r
r
r r
r
r r
+ = + = =

= = = =

O segundo termo o torque relativo ao centro de massa e o primeiro o
torque no centro de massa relativo ao ponto O.
CM ext CM ext
F r + =
r
r
r r

Tomando a derivada temporal da expresso L
r
temos:
dt
V d
M r V M
dt
r d
dt
L d
dt
L d
CM
CM CM
CM CM
r
r
r
r
r
r
+ + =
ext CM
CM
CM CM
CM
F r
dt
L d
a M r
dt
L d
= + = + =
r
r
r
r
r r
r

Comparando com a expresso para
ext

r
obtemos:
dt
L d
CM
CM
r
r
=
Esta equao muito importante, pois permite separar o movimento
translacional do centro de massa do movimento rotacional ao seu redor.
Vamos, em seguida, analisar alguns exemplos em que o movimento de rotao
aparece combinado com o de translao.
a) Cilindro descendo um plano inclinado Consideremos um cilindro
rolando sobre um plano inclinado sem deslizar, como mostra a Fig. 8.32.
Como o ponto de contato o centro instantneo de rotao sabemos que V
CM

= R e a
CM
= R.








Dinmica do corpo rgido

S. C. Zilio e V. S. Bagnato Mecnica, calor e ondas


165






Fig. 8.32 - Cilindro descendo um plano inclinado com atrito.
O nico torque ao centro de massa dado pela fora de atrito, no
sentido que sai do plano da figura. Desta forma,
CM
2
CM
at
CM
CM CM CM
CM
at CM
a
R
I
F
R
a
I I
dt
d
I
dt
dL
R F = = =

= = =
O movimento de translao descrito pela equao:
CM
2
CM
CM CM at
Ma
R
I
M a sen Mg Ma F sen Mg
2
3
=
|

\
|
+ = =
Logo: = sen g a
2
3
CM
e = sen Mg F
3
1
at

b) Movimento de um giroscpio O giroscpio, mostrado na Fig. 8.33, roda
de tal forma que seu momentum angular L
r
e seu eixo mantm-se no plano
horizontal, isto , g M F
r
r
= . O torque relativo ao centro de massa
F r
r
r r
= , com mdulo MgD =
r
e sentido de .







Fig. 8.33 - Movimento de um giroscpio.
M
R
g M
r
at F
r
N
r

D
F
r

g M
r
L
r

o






Dinmica do corpo rgido


S. C. Zilio e V. S. Bagnato Mecnica, calor e ondas


166
Como
dt
L d
r
r
= , vemos que dt L d =
r
r
. O vetor L d
r
perpendicular a
L
r
, como indica a Fig. 8.34. Em outras palavras, variao de L
r
se d apenas
na direo e no no mdulo (no existe componente de L d
r
paralela a L
r
). O
efeito um giro do sistema em torno do ponto de apoio. O acrscimo de
ngulo produzido pelo torque dado por:
L
dL
d =
, e durante o intervalo de
tempo dt temos:
dt
L
MgD
L
dt
L
dL
d =

= =
A taxa de variao do ngulo , chamada de velocidade de
precesso do giroscpio, dada por:

= =

=
I
MgD
L
MgD
dt
d




Fig. 8.34 Variao de momentum angular produzida pelo torque.
Se L
r
fosse nulo Mg seria maior que F e o giroscpio cairia.
De um modo geral, quando o giroscpio solto na horizontal, h um
pequeno movimento na vertical denominado de nutao. Esta contribuio
aparece quando consideramos a contribuio do movimento do centro de
massa ao momentum angular do sistema. O centro de massa tem uma
velocidade V
CM
= D e assim . MD D DM D MV L
2
CM CM
= = =
r
A
direo e sentido de
CM
L
r
a mesma que F
r
. Quando o giroscpio solto, =
0 e
CM
L
r
= 0. O momentum angular nesta direo deve se conservar, pois no
h nenhum torque externo nela. Assim, quando
CM
L
r
deixa de ser zero, o
giroscpio abaixa um pouco tal que L
r
passa a ter uma componente contrria a
CM
L
r
no sentido de anul-lo. A inrcia associada a este movimento faz com
d
L
r
L
r
L d
r





Dinmica do corpo rgido

S. C. Zilio e V. S. Bagnato Mecnica, calor e ondas


167
que haja uma oscilao peridica (nutao) na vertical, que tende a se
amortecer se houver atrito no sistema. Extinto este movimento, o giroscpio
se movimenta um pouco abaixo do plano horizontal.
Caso o giroscpio esteja fazendo um ngulo com a vertical, o torque
dado por sen MgD = , de onde se obtm dt d
L
sen MgD
sen L
dL

=

=
e, portanto,

=
I
MgD
independe do ngulo em que o giroscpio
posicionado.






Fig. 8.35 Giroscpio com eixo inclinado.
L
r
sen L
d

L d
r






Dinmica do corpo rgido


S. C. Zilio e V. S. Bagnato Mecnica, calor e ondas


168
Exerccios
1 - Calcule o momento de inrcia de um quadriltero de massas pontuais em
relao aos eixos mostrados na Fig. 8.36.
2 - Um disco de raio R e densidade superficial de massa tem um buraco
circular de raio r, distando a do centro do disco. Calcule os momentos de
inrcia em relao aos eixos 1, 2 e 3, mostrados na Fig. 8.37.










Fig. 8.36 Fig. 8.37
3 - Calcule o momento de inrcia de uma esfera de massa M e raio R em
relao a um eixo passando pelo centro de massa.
4 - Uma barra delgada de massa M e comprimento L faz um ngulo com
eixo y, conforme mostra a Fig. 8.38.
a) Calcule o momento de inrcia para rotao em torno do eixo;
b) Calcule o momento de inrcia para rotao em torno de um eixo
paralelo a y e passando pelo centro de massa.
5 - Uma escada de M e comprimento L est apoiada numa parede sem atrito e
no cho, com atrito (Fig. 8.39). Sabendo que o ngulo entre a escada e a
parede 45
o
, qual deve ser a tenso numa corda amarrada no meio da
escada para que ela no caia?

I
2

I
1

I
3

I
4

m
m
2m
2m
a
I
2

I
1

I
3

R
r
a





Dinmica do corpo rgido

S. C. Zilio e V. S. Bagnato Mecnica, calor e ondas


169







Fig. 8.38 Fig. 8.39
6 - Uma escada de massa M e comprimento L est apoiada numa parede e no
cho (ambos sem atrito) de maneira a formar um ngulo com a parede,
conforme mostra a Fig. 8.40. Uma corda amarrada a uma altura H
(paralela ao cho) mantm a escada em repouso. Calcule:
a) a tenso na corda;
b) a mxima altura H
max
em que possvel haver equilbrio;
c) a acelerao angular no instante em que esta corda for cortada.
7 - Uma escada de pintor de massa total 2M est aberta de maneira a formar
um ngulo . Qual deve ser o coeficiente de atrito esttico com o cho
para que ela no caia? (Fig. 8.41).








Fig. 8.40 Fig. 8.41
8 - Um pintor de massa M est no topo de uma escada de peso desprezvel
(comprimento de cada lateral: L) que se apia sobre um assoalho
M
L
y
L

T
45
o

M
L

M

H
L

2M

L







Dinmica do corpo rgido


S. C. Zilio e V. S. Bagnato Mecnica, calor e ondas


170
extremamente liso. H uma travessa a meia altura que impede a abertura
da escada. O ngulo do vrtice . Qual a fora na travessa?
9 - Uma barra de comprimento L e massa M colocada sobre um buraco,
como mostrado na Fig. 8.42. Qual deve ser o coeficiente de atrito para a
barra permanecer em repouso?
10 - Sobre uma superfcie lisa desliza um bloco cbico de lado L e massa M,
com velocidade v (Fig. 8.43). Num determinado ponto, o cubo bate em
um pequeno obstculo. Qual deve ser a velocidade v para que o bloco rode
em torno deste ponto?






Fig. 8.42 Fig. 8.43
11 - Na extremidade de uma haste de comprimento L a massa desprezvel
colocada uma massa M. O sistema solto de vertical sob a ao da
gravidade. Qual a equao que descreve o ngulo (t)? (Fig. 8.44).
12 - Um arco de raio R, que gira com velocidade angular
0
, colocado sobre
uma superfcie horizontal spera, como mostra a Fig. 8.45, sendo a
velocidade de seu centro de massa nula. Determinar a velocidade do
centro de massa depois de cessado o escorregamento.
13 - A integral do torque com relao ao tempo chamada impulso angular.
Partindo da relao dt / L d
r
r
= , mostre que o impulso a variao do
momentum angular.
14 - Uma bola de bilhar inicialmente em repouso recebe um impulso
instantneo de um taco. Este mantido horizontal a uma distncia h do
45
o

l
v
M
M





Dinmica do corpo rgido

S. C. Zilio e V. S. Bagnato Mecnica, calor e ondas


171
centro. A bola sai com velocidade v
0
e a velocidade final 9/7v
0
. Mostre
que h = 4/5R, onde R o raio da esfera.







Fig. 8.44 Fig. 8.45
15 - Niels Bohr postulou que um sistema mecnico em rotao s pode ter
momentum angular com valores mltiplos de uma constante h, chamada
constante de Planck h = h/2 = 1.054 x 10
-34
J.S), ou seja: L = I = nh,
sendo n um inteiro positivo ou zero.
a) Mostre que com este postulado, a energia de um rotor s pode adquirir
valores discretos, isto , quantizados.
b) Considere uma massa m obrigada a girar num crculo de raio R
(tomo de Bohr ou tomo de hidrognio). Quais so os possveis
valores para a velocidade angular levando-se em conta o postulado
acima?
c) Quais valores de energia cintica o tomo pode ter?
16 - Muitos dos grandes rios correm para a regio equatorial levando
sedimentos arenosos. Que efeito isto tem sobre a rotao da Terra?
17 - Um cilindro de massa M e raio R roda sem deslizar sobre um plano
horizontal. A velocidade do centro de massa v. Ele encontra um plano
com ngulo de inclinao sua frente, como mostra a Fig. 8.46.
a) Que altura o cilindro sobe no plano inclinado?
b) Nesta posio, qual foi a variao do momentum angular?
c) Quais foram os impulsos linear e angular?
d) Qual o tempo que o cilindro demora para atingir a altura mxima?
M

L
M
o
R






Dinmica do corpo rgido


S. C. Zilio e V. S. Bagnato Mecnica, calor e ondas


172
18 - Um disco de massa M e raio R pode mover-se em torno de um eixo
passando pelo seu centro de massa O, conforme mostra a Fig. 8.47. Uma
partcula de massa tambm M segue uma trajetria linear com velocidade
v e parmetro de impacto d = R/2 relativo ao ponto 0. Ao chocar-se com
o disco ela sofre uma deflexo de 90
o
e tem sua velocidade mudada para
( ) 3 2 v .
a) Qual a velocidade angular do disco aps a coliso?
b) Qual a energia dissipada na coliso?







Fig. 8.46 Fig. 8.47
19 - Um disco de massa 2m e raio R repousa sobre uma mesa horizontal
extremamente lisa. Uma bala de massa m, velocidade v
0
e parmetro de
impacto R atinge o disco e engasta nele (Fig. 8.48). Calcule:
a) A velocidade angular do sistema logo aps a coliso;
b) A velocidade do centro de massa aps a coliso;
c) A energia dissipada na coliso.
20 - Uma bola de bilhar inicialmente em repouso recebe um impulso
instantneo de um taco, que forma um ngulo com a horizontal, como
mostra a Fig. 8.49. A bola sai com velocidade inicial v
0
e ao final do
movimento ela encontra-se em repouso.
a) Determine o ngulo para que isto acontea;
b) Qual a velocidade angular inicial da bola?
c) Qual foi a energia dissipada durante o movimento?


O
M
R
V
r

H
V
r
d
M
R
M





Dinmica do corpo rgido

S. C. Zilio e V. S. Bagnato Mecnica, calor e ondas


173





Fig. 8.48 Fig. 8.49
21 - Uma partcula de massa m est presa ao extremo de um fio e percorre
uma trajetria circular de raio r sobre uma mesa horizontal sem atrito. O
fio passa por um orifcio de mesa e o outro extremo se encontra
inicialmente fixo. Neste caso, o raio inicial r
0
e a velocidade angular
inicial
0
. Comea-se ento a puxar lentamente o fio de maneira a
diminuir o raio da trajetria circular, como mostra a Fig. 8.50.
a) Como variar a velocidade angular em funo de r?
b) Qual o trabalho realizado para levar a partcula at o raio 2 / r
0
?






Fig. 8.50
22 - Considere um cilindro de massa M e raio R descendo um plano inclinado
de ngulo sem deslizar. Calcule a acelerao do centro de massa e a
fora de atrito agindo sobre o cilindro.
23 - Uma bola de bilhar de massa M e raio R ( )
2
5
2
MR I = desliza sem rodar
com velocidade v
0
sobre uma mesa sem atrito. Subitamente ela encontra
uma parte da mesa com atrito e depois de algum tempo est rodando sem
deslizar.
a) Calcule a velocidade final da bola;
b) Qual a energia dissipada no processo?
m
R
m 2
R
0 v
r

R
taco
M
m
F
r
v






Dinmica do corpo rgido


S. C. Zilio e V. S. Bagnato Mecnica, calor e ondas


174







Oscilaes

S. C. Zilio e V. S. Bagnato Mecnica, calor e ondas


175






9.1 O movimento harmnico simples
De um modo geral, chamamos de oscilaes aquela classe de
movimento que se repete no tempo, quer seja de uma maneira ordenada ou
no. O movimento que se repete regularmente com o passar do tempo
chamado de peridico e o intervalo decorrente entre duas situaes
equivalentes o perodo do movimento.
O estudo de oscilaes uma parte importante da mecnica devido
frequncia com que este tipo de evento ocorre. O simples balanar das folhas
de uma rvore, as ondas de rdio, o som e a luz so exemplos tpicos onde o
movimento oscilatrio acontece. Dentre estes movimentos, aquele chamado de
harmnico o mais simples, porm, um dos mais importantes devido sua
vasta aplicabilidade. No estudo do movimento harmnico simples (MHS) ns
vamos considerar apenas o caso unidimensional, onde a posio de um corpo
em relao posio de equilbrio dada por uma expresso do tipo:
( ) ( ) + = t cos A t x
0

onde A a amplitude do movimento, a fase e
0
a freqncia natural ou
freqncia de ressonncia do sistema. A e dependem das condies iniciais
do movimento enquanto que
0
uma grandeza intrnseca ao sistema, que est
relacionada com o perodo pela expresso:
f 2
T
2
0
=

=
onde f = 1/T a freqncia em Hertz (Hz) e
0
tem dimenses de rad/s.
9 OSCILAES






Oscilaes


S. C. Zilio e V. S. Bagnato Mecnica, calor e ondas


176
Um grfico da funo x(t) est mostrado na Fig. 9.1. Um exemplo
simples do MHS a projeo (ou a sombra) de um corpo em movimento
circular uniforme sobre o eixo x.








Fig. 9.1 - Movimento harmnico simples.
O MHS caracterizado por ter funes bem comportadas (analticas)
tanto em x(t) como em v(t) e a(t). De fato, estas grandezas so sempre
contnuas, com derivadas tambm contnuas. Isto j no ocorre, por exemplo,
para uma partcula oscilando no interior de uma caixa unidimensional de
comprimento L, mostrada na Fig. 9.2. Neste caso, x(t) uma funo peridica
e triangular, apresentando nos pontos x = 0 e x = L descontinuidade na
derivada primeira, j que a velocidade troca de sinal devido coliso com a
parede.










Fig. 9.2 Movimento peridico de uma partcula dentro de uma caixa.
A
A
-A
T
t
x(t)

0
x(t)
L/v
0


2L/v
0


t
v
0

v(t)
t
-v
0

v
0







Oscilaes

S. C. Zilio e V. S. Bagnato Mecnica, calor e ondas


177
Voltando ao caso do MHS, onde x(t) = Acos(
0
t + ), podemos
encontrar v(t) e a(t) atravs da operao de diferenciao:
( ) ( ) ( ) ( )
2
t cos A t sen A t x t v
0 0 0 0

+ + = + = = &
( ) ( ) ( ) ( ) t x t cos A t x t a
2
0 0
2
0
= + = = & &
de onde vemos que a velocidade est 90
o
fora de fase com a posio e que a
acelerao proporcional ao deslocamento, porm com a sentido oposto. Da
2
a
lei de Newton, temos:
kx x m ma F
2
0
= = =
que a fora encontrada num oscilador harmnico simples (sistema massa-
mola). Sempre que a fora proporcional e oposta ao deslocamento temos a
ocorrncia do MHS. A constante k denominada constante de mola ou
constante de fora do oscilador e a freqncia natural de oscilao do sistema,
m k
0
= , completamente independente da amplitude e fase do
movimento.
A velocidade mxima que um corpo em MHS pode atingir
0 max
A v = , de onde vemos que quanto maior for a amplitude do
movimento, maior ser a velocidade mxima. Da maneira que escrevemos x(t)
e v(t), notamos que para t = 0 temos x(0) = x
0
= Acos e v(0) = v
0
= -A
0
sen.
Assim, expandindo o co-seno existente em x(t) temos:
( ) ( ) t sen sen A t cos cos A t cos A t x
0 0 0
= + =
( ) t sen
v
t cos x t x
0
0
0
0 0

+ =
que a soluo mais geral para o movimento de um oscilador harmnico
simples sujeito s condies iniciais x(0) = x
0
e v(0) = v
0
.






Oscilaes


S. C. Zilio e V. S. Bagnato Mecnica, calor e ondas


178
9.2 O sistema massa-mola
Um dos sistemas mais simples que constitui um oscilador harmnico
o sistema massa-mola mostrado na Fig. 9.3. A equao de movimento dada
por:
kx
dt
x d
m F
2
2
= =
Chamando
2
0
m k = ficamos com a equao do MHS:
0 x
dt
x d
2
0
2
2
= +






Fig. 9.3 Sistema massa-mola.
Neste tipo de equao diferencial, que tambm aparece no caso do
pndulo e mesmo em outros sistemas, a raiz da constante que aparece
multiplicando o termo linear a freqncia natural de oscilao do sistema. A
soluo desta equao uma funo do tipo ( ) ( ) + = t cos A t x
0
, que
multiplicada por
2
0
e somada sua derivada segunda resulta num termo
nulo. Vimos que esta soluo pode ainda ser escrita como:
( ) t sen
v
t cos x t x
0
0
0
0 0

+ =
( ) t cos v t sen x t v
0 0 0 0 0
+ =
A fora restauradora F = -kx gera uma energia potencial V(x) dada por:
( )
2
x
0
kx
2
1
dx ) x ( F x V

= =
k
x
m






Oscilaes

S. C. Zilio e V. S. Bagnato Mecnica, calor e ondas


179
e assim a energia total do sistema :
2 2
kx
2
1
mv
2
1
E + =
Substituindo os valores de x(t) e v(t)encontramos:
2
0
2
0
mv
2
1
kx
2
1
E + =
que a energia inicial do sistema e que se mantm constante durante todo o
movimento, havendo apenas troca entre as energias cintica e potencial.
A conservao de energia permite outra maneira de encontrarmos a funo
x(t) sem resolver a equao diferencial. Como
( )
2
2 2
0
2 2
dt
dx
m
2
1
x m
2
1
mv
2
1
kx
2
1
E + = + =
podemos escrever a velocidade como funo de x:
( ) ( ) |

\
|

= =
2
2
0
2
0
2 2
0
2
x
m
E 2
x m
2
1
E
m
2
dt
dx

dt
x
m
E 2
dx
x
m
E 2
dt
dx
0
2
2
0
2
2
0
0
=

=
Integrando entre x(0) x
0
e x(t) = x e usando ,
a
arcsen
a
d
2 2

temos:
t
k / E 2
x
sen
x
sen
0
0 1
k
E 2
1
=
|

\
|

|
|

\
|


onde k m
2
0
= foi usado. Logo
( ) k / E 2 t x =
(

\
|
+

k / E 2
x
sen t sen
0
1
0







Oscilaes


S. C. Zilio e V. S. Bagnato Mecnica, calor e ondas


180
Se chamarmos k E 2 A = e ( ) 2 sen
k E 2
x
0
1
=

, obtemos a
j conhecida soluo:
( ) ( ) + = t cos A t x
0

As condies iniciais so agora dadas em termo de x
0
e E. Vamos
tomar dois exemplos de condio iniciais:
a)
0
2
0 2
1
0
x kx E 0 v , x x
k
E 2
= = = = , de onde tiramos que
( ) t cos x t x
0 0
= .
b)
0
0
2
0
2
1
0
v
k
E 2
mv v v , 0 x

= = = e assim, ( ) t sen t x
0
0
0
v
=

.
A energia potencial do sistema massa-mola,
2
2
1
kx ) x ( V = est
mostrada na Fig. 9.4. A energia determina completamente a amplitude do
movimento bem como a velocidade mxima que o corpo pode atingir.
A amplitude mxima, dada por , k E 2 A = determina o ponto de
retorno. Como ( ) ( ) , x k / E 2 x v
2 / 1
2
0
= vemos que a velocidade nula nos
pontos de retorno ( ) A . Por outro lado, a velocidade mxima para x = 0 e
vale m E 2 v
max
= .









Fig. 9.4 Energia potencial do sistema massa-mola.
-A A
K
V
E
x
V(x)






Oscilaes

S. C. Zilio e V. S. Bagnato Mecnica, calor e ondas


181
9.3 O sistema massa-mola com gravidade
Vamos analisar o que ocorre quando temos um corpo de massa m
pendurado numa mola vertical sob a ao do campo gravitacional, como
mostra a Fig. 9.5. Se no movimento horizontal a posio de equilbrio no
ponto x = 0, na presena da gravidade esta posio deslocada at o ponto em
que a fora peso equilibrada pela fora da mola, isto ,
k
mg
0 0
y ky mg = = .









Fig. 9.5 Sistema massa-mola sujeito ao campo gravitacional.
A 2
a
lei de Newton nos leva equao de movimento:
( )
0
2
2
y y k mg ky
dt
y d
m = + =
onde k / mg y
0
= a nova posio de equilbrio. Definindo y= y - y
0
, a
equao diferencial se torna:
0 ' y
dt
' y d
' ky
dt
' y d
m
2
0
2
2
2
2
= + =
cuja soluo j conhecida: y= y y
0
= A cos(
0
t + ). Isto quer dizer que o
oscilador harmnico sob a ao da gravidade tem o mesmo comportamento
que quando colocado na horizontal, apenas sua posio de equilbrio desloca-
se de y
0
. Podemos escrever a energia potencial do sistema como:
m
y = 0
k
m
y
0

k
mg






Oscilaes


S. C. Zilio e V. S. Bagnato Mecnica, calor e ondas


182
( ) mgy ky
2
1
y V
2
=
onde o zero da energia potencial gravitacional foi tomado em y = 0.
Completando os quadrados vemos que V(y) dada por:
( )
k 2
g m
) y y ( k
2
1
y V
2 2
2
0
=
A curva que descreve a energia potencial V(y) est mostrada na Fig.
9.6. Comparando com o oscilador horizontal, vemos que a ao da gravidade
o de deslocar o mnimo da curva de potencial para o ponto (y
0
, V(y
0
)) onde y
0

= mg/k e V(y
0
) = -m
2
g
2
/2k. Nem a frequncia
0
nem a amplitude do
movimento so alterados por influncia da gravidade.










Fig. 9.6 Energia potencial do sistema massa-mola sujeito gravidade.
9.4 O pndulo matemtico
O pndulo simples ou matemtico um outro exemplo bastante
importante de movimento harmnico. De acordo com a Fig. 9.7, o torque em
relao ao ponto O , sen mgL = sendo que o sinal negativo se deve ao
fato de que o torque est no sentido oposto ao que o ngulo aumenta.


k 2
g m
2 2

y
V(y)
y
0







Oscilaes

S. C. Zilio e V. S. Bagnato Mecnica, calor e ondas


183








Fig. 9.7 Pndulo simples ou matemtico.
Como = =
& &
I I , temos =

= sen mgL
dt
d
mL I
2
2
2
, que pode
ser escrito como:
0 sen
dt
d 2
0
2
2
= +


onde L / g
2
0
= . Esta equao diferencial muito difcil de ser resolvida mas
para ngulos pequenos ( )
0
15 < podemos aproximar sen por e assim,
0
2
0
= +
& &

cuja soluo ( ) ( ). t cos t
0 0
+ = Este resultado tambm pode ser
encontrado atravs de consideraes energticas. As energias cintica e
potencial so dadas respectivamente por:
2
2
2
2 2
1 mL
I K = =
& &

( ) ( ) = cos 1 mgL V
onde o zero da energia gravitacional foi escolhido na posio mais baixa da
massa. A energia total E = K + V uma constante de movimento e assim,
dE/dt = 0. Logo:
L
M

O
g m
r






Oscilaes


S. C. Zilio e V. S. Bagnato Mecnica, calor e ondas


184
( ) 0 cos 1 mgL
2
mL
dt
d
2
2
=
|

\
|
+
&

( ) 0 sen g L mL sen mgL mL
2
= + = +
& & & & & & &

0
L
g
2
0
= + = +
& & & &

onde a aproximao sen foi usada.
9.5 O pndulo fsico
Pndulo fsico o nome dado a qualquer corpo rgido suspenso por
um ponto diferente do centro de massa, que quando solto de um certo ngulo
oscila em torno da posio de equilbrio, como mostrado na Fig. 9.8.







Fig. 9.8 - Pndulo fsico.
Para se encontrar a equao diferencial do pndulo fsico, usa-se o
mesmo procedimento adotado para o pndulo simples. O torque
= sen mgD e, portanto,
= =
& &
I sen mgD
onde I / mgD
2
0
= a freqncia natural de oscilao. No caso do pndulo
matemtico,
2
mD I = e, portanto, D / g
2
0
= . No caso de uma barra delgada
de massa m e comprimento 2 / L D , ML I , L
2
3
1
= = a posio do centro de
massa e, portanto, L / g
2
3 2
0
= .
D

O
CM






Oscilaes

S. C. Zilio e V. S. Bagnato Mecnica, calor e ondas


185
9.6 Oscilao de dois corpos
Vamos considerar o caso em que dois corpos de massas M
1
e M
2
so
interconectados atravs de uma mola de constante k e comprimento livre l,
como mostra a Fig. 9.9. Seja x
1
(t) e x
2
(t) a posio dos corpos em relao a
uma origem arbitrria. A variao no comprimento da mola dada por:
x = (x
2
x
1
) l
de forma que se a mola estiver distendida x > 0 e se estiver comprimida x < 0.




Fig. 9.9 Corpos conectados por uma mola.
As foras sobre os corpos depender do sinal de x:
kx x M
kx x M
2 2
1 1
=
=
& &
& &

Podemos combinar estas equaes de movimento e obter resultados
interessantes. Por exemplo, somando as duas equaes temos:
( ) 0 a M M a M a M
dt
x d
M
dt
x d
M
CM 2 1 2 2 1 1
2
2
2
2
2
1
2
1
= + = + = +
Isto implica que a acelerao do centro de massa nula e, consequentemente,
a velocidade do centro de massa constante, pois no existem foras externas.
Podemos tambm analisar o movimento relativo ao centro de massa. Vamos
re-escrever as equaes de movimento como:
x
M
k
dt
x d
1
2
1
2
=
x
M
k
dt
x d

2
2
2
2
=
M
1

M
2

0
x
1

x
2







Oscilaes


S. C. Zilio e V. S. Bagnato Mecnica, calor e ondas


186
Subtraindo a primeira da segunda, obtemos:
( ) x
M
1
M
1
k x x
dt
d
1 2
1 2
2
2
|

\
|
+ =
0 x x
k
dt
x d
2
0
2
2
= +

= & &
onde
2 1
M
1
M
1 1 2
0
e / k + = =

chamada de massa reduzida. Desta forma,
encontra-se uma equao diferencial bastante conhecida que descreve entre os
dois corpos como funo do tempo. A introduo da massa reduzida faz com
que o oscilador constitudo de dois corpos seja equivalente ao sistema de
apenas uma massa e uma mola. Esta considerao bastante importante no
estudo de vibraes moleculares.
9.7 O sistema mola-cilindro
Um cilindro de massa m e raio R est ligado a uma mola de constante
k, como mostra a Fig. 9.10. Queremos encontrar a freqncia de oscilao do
sistema quando o cilindro roda sem deslizar. Chamaremos de x a coordenada
do centro de massa do cilindro com origem na posio em que a mola no est
distendida. Sendo F
at
a fora de atrito, as equaes de movimento so:






Fig. 9.10 Sistema mola-cilindro.
kx F x m
at
= & &
para a translao do centro de massa e
R F I
a
=
m
-kx
x
R
F
at







Oscilaes

S. C. Zilio e V. S. Bagnato Mecnica, calor e ondas


187
Para a rotao em torno do centro de massa. Como o cilindro roda sem
deslizar temos R / x& & = e assim,
x
2
m
F R F
R
x
2
mR
at at
2
& &
& &
= =
Substituindo na equao da translao,
0 x x kx x m
2
3 2
0
= + = & & & &
onde
m
k
3
2
2
0
=
9.8 Oscilaes amortecidas
De um modo geral, a existncia de atrito faz com que a energia de um
sistema oscilante seja dissipada. Como a energia de um oscilador simples est
diretamente ligada amplitude do movimento, a dissipao de energia acarreta
um decrscimo na amplitude. Consideremos um oscilador tipo massa-mola no
interior de um meio viscoso. Sua equao de movimento da forma:
kx x b x m = & & & 0 x x
m
b
x
2
0
= + + & & &
Como discutimos anteriormente, devido ao atrito, a amplitude deve
diminuir com o tempo de modo que podemos tentar uma soluo do tipo x(t) =
Ae
-t
cos(t + ). Substituindo esta funo na equao diferencial obtemos:
( ) ( ) ( ) ( ) 0 t sen Ae
m
b
2 t cos Ae
m
b t t 2
0
2 2
= +

+ +

+


onde . m / k
2
0
= Da equao acima tiramos:
m 2
b
m
b
2 = =
0
m
b
2
0
2 2
=

+
2
2
2
0
m 4
b
=






Oscilaes


S. C. Zilio e V. S. Bagnato Mecnica, calor e ondas


188
e assim a frequncia de oscilao modificada pela presena do atrito.
Considerando o caso em que m 2 / b
0
> (sistema sub-amortecido) vemos que
a soluo escrita como:
( ) ( ) + =
|

\
|

t cos Ae t x
m 2
bt

sendo A e determinados pelas condies iniciais. Um grfico x(t) est
mostrado na Fig. 9.11.











Fig. 9.11 Movimento harmnico sub-amortecido.
9.9 Oscilaes foradas
Vamos analisar agora o caso de um sistema massa-mola com
freqncia de ressonncia
0
submetido a uma fora externa do tipo:
t sen F F
0 ext
=
A equao diferencial que descreve o movimento :
t sen
m
F
x x
0
2
0
= + & &
Como o sistema est sendo forado a uma freqncia , ele oscilar nesta
freqncia, porm a amplitude do movimento no aumentar, pois o trabalho
m 2
bt
Ae


t
x(t)






Oscilaes

S. C. Zilio e V. S. Bagnato Mecnica, calor e ondas


189
realizado por F nulo em cada perodo. Podemos tentar uma soluo do tipo
x(t) = Asent. Substituindo na equao diferencial, encontramos o valor de A
dado por:
( )
2 2
0
0
m
F
A

=
Quando
0
< , A negativo e isto indica que a resposta do sistema est 180
o

fora de fase com o estmulo. A potncia fornecida pela fora F :
( )
t cos
m
F
t sen F Fv ) t ( P
2 2
0
0
0


= =
( )
t 2 sen
m 2
F
) t ( P
2 2
0
2
0


=
Quando calculamos a potncia fornecida ao sistema durante um perodo
completo temos:


= =
2
0
0 dt ) t ( P P
O grfico da amplitude de movimento como funo de est
mostrado na Fig. 9.12. Podemos ver que A tende a infinito quando
0
.
Porm, na prtica isto no acontece porque foras dissipativas impedem que
isto acontea. A equao diferencial para um sistema massa-mola amortecido
sujeito a uma fora do tipo F(t) = F
0 sent :







Fig. 9.12 Amplitude do movimento forado sem atrito como funo da frequncia de
excitao.

A()






Oscilaes


S. C. Zilio e V. S. Bagnato Mecnica, calor e ondas


190
t sen
m
F
x x
m
b
x
0
2
0
= + + & & &
Novamente o sistema obrigado a oscilar com freqncia , porm, devido ao
termo de amortecimento, pode haver uma parte da soluo que esteja fora de
fase com F(t). Portanto, vamos supor que a soluo seja do tipo:
x(t) = A
1
cost + A
2
sent
Substituindo na equao diferencial obtemos:

( )
t sen
m
F
A A
m
b
A t sen A
m
A b
A t cos
0
2
0 2 1 2
2 2
0 1
2
1
2
=
+

+
|

\
|
+

+

Como esta igualdade deve ser vlida para qualquer instante de tempo,
devemos ter:
( ) 0 A
m
b
A
2 1
2 2
0
=

+
( )
m
F
A A
m
b 0
2
2 2
0 1
= +


de onde podemos encontrar os valores de A
1
e A
2
e, conseqentemente, x(t). A
soluo pode ser colocada na forma:
( )
( ) ( )
( )
+
=

t sen
m / F
t x
2
2
2 2
0
0
m
b

( )
2 2
0
m
b
tg


=






Oscilaes

S. C. Zilio e V. S. Bagnato Mecnica, calor e ondas


191
Vemos agora que prximo da ressonncia (
0
), a amplitude do
movimento fica limitada ao valor
b
F
0
e, portanto, no diverge. Um grfico
desta amplitude est mostrado na Fig. 9.13.









Fig. 9.13 Amplitude do movimento forado com atrito como funo da frequncia de
excitao.
A soluo que acabamos de encontrar a chamada soluo particular
da equao diferencial. Existem tambm a soluo da equao homognea que
chamada de transiente e que desaparece com o passar do tempo. A soluo
geral da equao diferencial dada por:
( ) ( )
( ) ( )
( )
+
+ =

\
|

t sen
m F
t ' cos e ' A t x
2 2
2 2
0
0
m
b
m 2
bt


A()

R







Oscilaes


S. C. Zilio e V. S. Bagnato Mecnica, calor e ondas


192
Exerccios
1 - Nos sistemas mostrados na Fig. 9.14 no h atrito entre as superfcies do
corpo e do cho e as molas tm massa desprezveis. Encontre as
freqncias naturais de oscilao.




(a) (b) (c)
Fig. 9.14
2 - Composio de movimentos (Figuras de Lissajous) - Consideremos um
corpo sujeito a dois movimentos harmnicos em direes ortogonais:
( ) ( )
x x x
t cos A t x + =
( ) ( )
y y y
t cos A t y + =
a) Quando
y x
/ um nmero racional, a curva fechada e o
movimento repete-se em tempos iguais. Determine a curva traada pelo
corpo para
x
/
y
= 1/2, 1/3 e 2/3, tomando
y x y x
e A A = = .
b) Para
x
/
y
= 1/2, 1/3 e , A A
y x
= desenhe as figuras para
y x
=
0, /4 e /2.
3 - Considere um cilindro preso por duas molas que roda sem deslizar como
mostra a Fig. 9.15. Calcule a freqncia para pequenas oscilaes do
sistema.
4 - Considere um pndulo simples de massa m e comprimento L, conectado a
uma mola de contraste k, conforme mostra a Fig. 9.16. Calcule a
freqncia do sistema para pequenas oscilaes.


M
k
1

k
2

M
k
1

k
2

M
k
1

k
2







Oscilaes

S. C. Zilio e V. S. Bagnato Mecnica, calor e ondas


193







Fig. 9.15 Fig. 9.16
5 - Dois movimentos harmnicos de mesma amplitude mas freqncias
ligeiramente diferentes so impostos a um mesmo corpo tal que
( ) | | t cos A ) t ( x e t cos A ) t ( x
2 1
+ = = . Calcule o movimento
vibracional resultante.
6 - Considere um pndulo simples num meio viscoso com constante de fora
viscosa b. Calcule o novo perodo de oscilao de pndulo.
7 - Considere uma barra delgada de massa M e comprimento 2L apoiada no
centro de massa como mostra a Fig. 9.17. Ela presa nas duas
extremidades por molas de constante k. Calcule a freqncia angular para
pequenas oscilaes do sistema.
8 - Considere 2 pndulos (comprimento L e massa M) acoplados por uma
mola de constante k, conforme mostra a Fig. 9.18.
a) Encontre as equaes diferenciais para os ngulos
1
e
2
.
b) Defina as coordenadas normais de vibrao =
1
-
2
e =
1
+
2
.
Encontre as equaes diferenciais para e . Dica: some ou subtraia
as equaes de a)
c) Quais so as freqncias angulares dos modos normais de vibrao?





M
k k
R
a
a
L
M







Oscilaes


S. C. Zilio e V. S. Bagnato Mecnica, calor e ondas


194







Fig. 9.17 Fig. 9.18
9 - Considere um disco de massa M e raio R ( )
2
2
1
MR I = que pode rodar em
torno do eixo polar. Um corpo de massa m est pendurado em uma corda
ideal, que passa pelo disco (sem deslizar) e presa a uma parede atravs
de uma mola de constante k, como mostra a Fig. 9.19. Calcule a
freqncia natural do sistema.








Fig. 9.19


k k
2L
L
M

1

L
M

2
k
R
M
m
k






Movimento ondulatrio

S. C. Zilio e V. S. Bagnato Mecnica, calor e ondas


195






10.1 Introduo
O movimento ondulatrio bastante importante devido ao fato de que
a maior parte do intercmbio de informaes ou energia entre sistemas fsicos
d-se atravs de ondas. Uma onda nada mais do que uma perturbao que se
cria num meio material elstico e que se propaga por este meio. Em alguns
casos (e.g. ondas eletromagnticas) a propagao da onda deve-se a dois tipos
de energias (eltrica e magntica) que se auto-sustentam. Neste caso, no
necessria a presena de um meio material para que a onda se propague.
Vamos comear nosso estudo imaginando uma corda esticada, presa a
uma parede. Como mostra a Fig. 10.1, se um pulso (movimento para cima e
para baixo) for introduzido na extremidade livre da corda ele caminhar para a
outra extremidade e este efeito conhecido como propagao.








Fig. 10.1 Pulso propagando-se numa corda esticada.
Durante a propagao do pulso na corda no h transporte de massa
pois todas as partculas do sistema mantm suas posies originais aps a
10
MOVIMENTO
ONDULATRIO






Movimento ondulatrio


S. C. Zilio e V. S. Bagnato Mecnica, calor e ondas


196
passagem do pulso. Entretanto, existe transporte de energia ao longo da corda
j que cada poro dela recebe um acrscimo de energia potencial durante a
passagem do pulso.
Em geral, o pulso se alarga durante a propagao e este efeito d-se o
nome de disperso. Ao atingir a parede, o pulso no consegue continuar a
propagao na mesma direo porque o ponto da corda preso na parede fixo.
Assim, ele comea a propagar-se no sentido oposto, retornando para a
extremidade livre. Se ao invs da parede rgida tivssemos uma outra corda
mais pesada, o pulso seria parcialmente refletido e parcialmente transmitido
para outra corda. Dependendo da densidade de massa deste segundo meio, a
parte refletida pode ou no inverter o sinal (o pulso fica virado para baixo).
Esta onda que acabamos de ver tem a direo de propagao
perpendicular ao pulso, isto , cada poro da corda movimenta-se
perpendicularmente direo em que existe a transmisso de energia. Neste
caso, a onda denominada transversal. No caso em que as partes de um
sistema se movem paralelamente direo de propagao do pulso temos uma
onda do tipo longitudinal. Como exemplo, considere uma longa mola esticada
na qual um pulso de compresso se propaga. Embora no haja transporte de
massa, cada poro do meio material estar animada de movimento oscilatrio
durante a propagao do pulso.
Vamos a seguir analisar algumas propriedades de um pulso
propagando-se numa corda. Como primeira aproximao, vamos supor que
no h disperso, isto , o pulso mantm sua forma original. Consideremos um
referencial fixo O e um referencial O movendo-se com velocidade v junto
com o pulso, como mostra a Fig. 10.2.





Fig. 10.2 Propagao de um pulso sem disperso.
P
y
x
O
O
y
x






Movimento ondulatrio

S. C. Zilio e V. S. Bagnato Mecnica, calor e ondas


197
Como o pulso no muda de forma com o tempo, as coordenadas de
um ponto P do pulso so dadas por y = y, x = x+ vt. A forma do pulso uma
funo de x, de forma que para o tempo t = 0 temos y = f(x) enquanto que para
t > 0, y = f (x - vt). Isto representa uma translao da funo f(x) para a direita.
No caso em que y = f(x + vt) temos uma onda propagando-se para a esquerda.
A funo y = f(x - vt) chamada de funo de onda. Dois pulsos
propagando-se no mesmo meio tem como funo de onda total a soma das
funes de onda:
( ) ( ) ( ) vt x f vt x f t , x Y
2 1
+ + =
originando o fenmeno de interferncia, que pode ser construtiva ou
destrutiva.
10.2 Propagao de pulsos numa corda
A velocidade com que o pulso se propaga numa corda depende
essencialmente das propriedades da corda (tenso e densidade de massa) e no
da amplitude do pulso. Consideremos um pulso deslocando-se com velocidade
v numa corda de densidade linear de massa e tenso T. De acordo com a
Fig. 10.3, vamos tomar um elemento da corda com comprimento dx.









Fig. 10.3 Elemento de massa de uma corda quando h a passagem de um pulso.
y
x
O
(x+dx)
(x)
T
T
x x+dx






Movimento ondulatrio


S. C. Zilio e V. S. Bagnato Mecnica, calor e ondas


198
Vamos aplicar a 2
a
lei de Newton a este elemento da corda, cuja massa
de dm = dx. Como a tenso na corda T, a fora vertical dada por:
( ) ( ) x sen T dx x sen T F
y
+ =
Se considerarmos dx muito pequeno ( (x) muito prximo de (x +
dx)) e tambm considerando (x) pequeno tal que sen ~ tg ~ dy/dx
teremos F
y
dada por:
dx
x
y
T
dx
dy
T
dx
dy
T F
2
2
x dx x
y

=
+

Pela 2
a
lei de Newton temos
2
2
y
t
y
dm F

= . Desta forma,
0
t
y
T
x
y
dx
x
y
T
t
y
dx
2
2
2
2
2
2
2
2
=


Esta equao chamada de equao de onda e descreve totalmente o
movimento de um pulso numa corda de densidade linear de massa e tenso
T. Como y = f(x - vt) depende tanto de x como de t, as derivadas que
aparecem na equao so parciais, isto , deriva-se em relao a uma varivel
matando a outra constante. Para encontrar a velocidade de propagao do
pulso, fazemos:

x
y
v
t
y

x
y
v
t
y
2
2
2
2
2

0
t
y
v
1
x
y
2
2
2 2
2
=


onde a regra da cadeia
t
x
x
y
t
y

foi usada. Comparando esta equao com


a equao de onda, conclumos que

=
T
v .
10.3 Ondas sonoras
A onda na corda que acabamos de ver um exemplo de onda
transversal. Vejamos agora um exemplo de onda longitudinal. As ondas






Movimento ondulatrio

S. C. Zilio e V. S. Bagnato Mecnica, calor e ondas


199
sonoras correspondem a um processo de compresso e descompresso de um
meio gasoso (como o ar), lquido ou mesmo slido. Vamos considerar que
estas ondas so produzidas por um pisto oscilante, como mostrado na Fig.
10.4.









Fig. 10.4 Ondas sonoras produzidas por um pisto oscilante.

0
a densidade de massa e P a presso (P = F/A) do gs quando no existe
compresso. Queremos encontrar a velocidade v com que a compresso (onda
de presso) se desloca ao longo do tubo da Fig. 10.4. Para isto, vamos tomar
um referencial que se desloca com velocidade v junto com o pulso e analisar o
que acontece com o elemento de massa mostrado na Fig. 10.5.






Fig. 10.5 Comportamento de um elemento de massa de um meio gasoso quando h
a passagem de um pulso.
Para um observador no referencial mvel, este elemento caminha para
a esquerda com velocidade -v. Ao atingir a regio de compresso, a presso (e
conseqentemente a fora) no lado esquerdo aumenta, existindo assim uma

0
, P
compresso
descompresso
compresso
zona de
compresso
P+P P
P P
(v+v) t v






Movimento ondulatrio


S. C. Zilio e V. S. Bagnato Mecnica, calor e ondas


200
fora resultante para a direita, que produz uma reduo da velocidade do
elemento considerado. Ao sair da zona de compresso o gs torna a se
expandir e recupera a velocidade v.
O elemento de massa considerado m =
0
V =
0
Avt. A fora
resultante para a direita sobre m :
( ) PA PA A P P F = + =
Pela 2
a
lei de Newton,


=
t
v
t v A
t
v
m A P
0

onde j tomamos v como sendo negativo. Portanto,
( ) v / v
P
v
2
0

=
Por outro lado, o volume do elemento do gs :
V
V
v
v
t v A V t Av V

=

= =
ou seja, a variao fracional de velocidade a mesma que a variao fracional
de volume. Logo,
B
V
P
V
V / V
P
v
2
0
=

=
onde
V
P
V B

= uma caracterstica do gs chamado de mdulo de


compressibilidade volumtrica do gs. Desta forma,
0
B v =
Na tabela seguinte apresentamos a velocidade de propagao do som
em vrios materiais.








Movimento ondulatrio

S. C. Zilio e V. S. Bagnato Mecnica, calor e ondas


201
Meio T(
o
C) v (m/s)
ar 0 331
H
2
0 1286
O
2
0 317
H
2
O 15 1450
Pb 20 1230
Al 20 5100
Cu 20 3560
borracha 20 54

10.4 Ondas harmnicas
Se ao invs de darmos um pulso na corda fizermos com que ela oscile
devido a uma perturbao peridica, teremos a propagao de um trem de
pulsos peridicos e regulares. Quando esta perturbao da forma senoidal ela
deve estar necessariamente sendo produzida por um oscilador harmnico e,
portanto, denominada de onda harmnica. A Fig. 10.6 mostra uma onda
harmnica.






Fig. 10.6 - Onda harmnica.
A distncia entre dois mximos consecutivos chama-se comprimento
de onda () enquanto que a amplitude da onda mostrada y
0
. medida que
a onda se propaga pela corda, cada um de seus pontos executa um
movimento harmnico simples, cuja freqncia a mesma que a da
fonte. O perodo da onda o inverso da freqncia (T = 1/f). Ele o

v
x
y
y
0
crista






Movimento ondulatrio


S. C. Zilio e V. S. Bagnato Mecnica, calor e ondas


202
tempo que um ponto da corda demora executar uma oscilao completa
e tambm o tempo que a crista da onda demora para andar a distncia
. Portanto,
f v vT = =
A oscilao de cada ponto da corda descrita pela funo
( ) ( ) t kx sen y t y
0
=
onde f 2 = a freqncia angular e, assim,
k
2
v
2
v =

=
onde k chamado de vetor de onda.
10.5 Efeito Doppler
Imaginemos que ao invs de termos uma fonte parada em relao ao
observador, temos agora um movimento relativo entre os dois. Neste caso, se
a fonte estiver se aproximando do observador, este v uma freqncia maior
e se a fonte estiver se afastando, a freqncia observada ser maior. Este efeito
da variao da freqncia devido ao movimento relativo fonte-observador
chamado de efeito Doppler.
Vamos imaginar uma fonte emitindo sinais com uma freqncia f
0
,
isto , durante um segundo so emitidos f
0
pulsos completos. Se ao invs de 1s
tomarmos um intervalo de tempo t, sero emitidos N = f
0
t pulsos. Vamos
agora analisar os seguintes casos:
a) Observador e fonte parados
Durante o tempo t, o 1
o
pulso percorre uma distncia l = N = vt.
Portanto,
0 0
f
v
t f
t v
N
t v
=

=






Movimento ondulatrio

S. C. Zilio e V. S. Bagnato Mecnica, calor e ondas


203
1 2
u/f
0

0

relao esta que j conhecamos. O nmero de pulsos recebido pelo
observador ser igual ao emitido pela fonte e, conseqentemente, ele no
percebe alteraes na freqncia.
b) Fonte em movimento e observador parado
Digamos que a fonte est se movendo com velocidade u. Durante a
emisso de uma onda completa (ou de dois pulsos consecutivos) a fonte
desloca-se a uma distncia uT = u/f
0
, conforme mostra a Fig. 10.7.







Fig. 10.7 - Fonte em movimento e observador parado.
Desta forma, o comprimento de onda efetivo visto por um observador
estacionrio
0 0 0 0
f u f v f u = =
( )
v
u
1
0
+ =
Como a velocidade de propagao no muda, isto nos leva a uma variao de
freqncia. Fazendo , v f = temos:
v u 1
f
0
f

= fonte se aproximando (f
aumenta) e
v u 1
f
0
f
+
= fonte se afastando (f diminui)
c) Fonte parada e observador em movimento
No caso que acabamos de analisar, devido ao movimento da fonte, o
comprimento de onda emitindo se altera. Se a fonte estiver em repouso isto j
no acontece, porm se o observador estiver em movimento, o nmero de
ondas recebido por ele, por unidade de tempo, se altera. Se o observador






Movimento ondulatrio


S. C. Zilio e V. S. Bagnato Mecnica, calor e ondas


204
estiver parado, o nmero de ondas que passam por ele durante o tempo t
dado pela expresso:
t
N v
f t v N
0
0 0

= =
Por outro lado, se o observador estiver se aproximando da fonte com
velocidade u, o nmero de ondas que ele recebe durante o tempo t :
0 0
t u t v
' N

=
e a freqncia observada :
( )
v
u
1 f
u v
t
' N
f
0
0
+ =

+
=

= (aproximando, f aumenta)
e quando o observador estiver se afastando da fonte,

=
v
u
1 f f
0
(afastando, f diminui)
Quando u << v, podemos mostrar que a freqncia depende apenas da
velocidade relativa entre o emissor e o receptor, diferindo nos casos b) e c)
apenas por um termo da ordem (u/v)
2
<< 1, que chamado de efeito Dopper
de 2
a
ordem.
At agora apenas consideramos casos em que u < v. O que acontece se
u > v? Nesta condio, as ondas emitidas pela fonte ficam para trs, no
havendo nenhuma onda adiante. Imaginemos uma fonte emitindo ondas
bidimensionais que se propagam com velocidade v. Se a fonte estiver em
repouso observaremos um padro de ondas, como mostrado na Fig. 10.8, que
se espalha uniformemente no plano. Imaginemos agora a fonte deslocando-se
para a direita com velocidade u > v. No ponto 1 ela emite uma onda.
Decorrido um intervalo de tempo t, esta onda ter se propagado de maneira a
formar um crculo de raio vt em torno do ponto 1. Por outro lado, a fonte ter
se deslocado para o ponto 2, distante ut do ponto 1, tendo emitido vrias
ondas durante este tempo, conforme mostra a Fig. 10.9.







Movimento ondulatrio

S. C. Zilio e V. S. Bagnato Mecnica, calor e ondas


205






Fig. 10.8 - Fonte parada.
As ondas estaro confinadas num cone de ngulo dado por:
u v
t u
t v
sen =

=
Isto pode ser observado se um objeto anda na gua emitindo ondas.








Fig. 10.9 Fonte em movimento com velocidade maior que a da onda.
10.6 Ondas estacionrias
Quando as ondas esto confinadas numa determinada regio do
espao, ondas incidentes e refletidas no contorno desta regio interferem,
produzindo configuraes especiais de ondas denominadas estacionrias.
Estas ondas possuem uma propriedade importante: num certo ponto x, o meio
oscila em torno daquele ponto com amplitude constante (independente do
tempo). Assim, ondas estacionrias tm como caracterstica apresentarem
amplitudes de oscilaes dependentes da posio, mas independentes do

v t
1
2
u t






Movimento ondulatrio


S. C. Zilio e V. S. Bagnato Mecnica, calor e ondas


206
tempo. A seguir, vamos considerar ondas estacionrias em diversas situaes
distintas.
a) Corda com as duas extremidades fixas
Imaginemos uma corda que tem numa das extremidades um elemento
vibrador (tipo diapaso) e na outra uma parede fixa. As ondas geradas pelo
vibrador so de pequena amplitude e refletem na parede e no prprio vibrador,
ficando assim confinadas e formando uma onda estacionria. Se as ondas
geradas pelo vibrador tiverem a freqncia correta interferiro
construtivamente aps uma volta completa nesta cavidade ressonante e no
final, a amplitude de vibrao ser muito maior que a amplitude da prpria
fonte emissora. Esta situao conhecida como ressonncia.
Nas extremidades fixa, a corda no pode oscilar e tem, portanto,
amplitude nula. Estes pontos so chamados de ns e a distncia entre dois ns
consecutivos /2 nesta regio. Assim, a condio de onda estacionar numa
corda de comprimento L e extremidades fixas, dada por:
( )
2
n L

=
com n = 1, 2, 3,... Esta condio determina completamente. Para
encontrarmos as freqncias de ressonncia podemos utilizar a relao
= v f , onde = T v a velocidade de propagao na corda. Da
condio de ressonncia temos:
1 n
nf
L 2
nv
f
f
v
n n L 2 = = = =
onde f
1
a freqncia do modo fundamental (n = 1), que

= =
T
L 2
1
L 2
v
f
1

As freqncias f
n
so denominadas de freqncias naturais da corda.
Quando tentamos fazer o diapaso oscilar com freqncia diferente de f
n
, o






Movimento ondulatrio

S. C. Zilio e V. S. Bagnato Mecnica, calor e ondas


207
ventre
sistema no entra em ressonncia e as ondas geradas apresentam amplitude
muito pequena, do tamanho da do diapaso. Na Fig. 10.10 mostramos as
ondas estacionrias na corda com extremidades fixas. A freqncia f
1

chamada de fundamental e as freqncias f
n
so denominadas de harmnicos
de ordem n.



n = 1 (fundamental ou 1
o
harmnico)



n = 2 (2
o
harmnico)



n = 3 (3
o
harmnico)



n = 4 (4
o
harmnico)

Fig. 10.10 Ondas estacionrias numa corda com as extremidades fixas.
b) Corda com apenas uma extremidade fixa
Na verdade, no podemos ter uma corda com a extremidade
completamente livre devido ao fato de ser necessria a existncia de uma certa
tenso na corda para termos v 0. Na extremidade fixa, a onda refletida
sempre invertida enquanto que na extremidade livre isto j no ocorre e a
onda refletida soma-se incidente. Desta forma, a extremidade livre sempre
uma posio de mxima amplitude (ventre) da onda estacionria formada.






Movimento ondulatrio


S. C. Zilio e V. S. Bagnato Mecnica, calor e ondas


208
Assim, o comprimento da corda dever ser um mltiplo inteiro mpar de /4,
ou seja:

4
n L

=
com n = 1, 3, 5,...,de onde obtemos que
L 4
v
L 4
v
1 1 n
f onde , nf n f = = = a
freqncia do modo fundamental. A Fig. 10.11 mostra os modos de vibrao
da corda com extremidade livre.


n = 1 (fundamental)



n = 3 (3
o
harmnico)



n = 5 (5
o
harmnico)



n = 7 (7
o
harmnico)



Fig. 10.11 Ondas estacionrias numa corda com uma extremidade livre.
c) Ondas acsticas estacionrias
Ondas estacionrias no se restringem apenas s ondas em cordas, mas
manifesta-se em qualquer fenmeno ondulatrio, inclusive em ondas
acsticas. Imagine um microfone gerando ondas sonoras, colocado na boca de






Movimento ondulatrio

S. C. Zilio e V. S. Bagnato Mecnica, calor e ondas


209
um tubo fechado, de comprimento L. O som no pode propagar-se para fora
do tubo, pois de um lado existe uma parede rgida e do outro, o microfone e
neles so formados os ns da onda estacionria. Assim, da mesma maneira
que na corda com extremidade s fixas temos:

n
L 2
2
n L
n
=

=
com n = 1, 2, 3,...As freqncias de ressonncia so dadas por:
1 n
nf
L 2
nv
f = =
onde:
0
1
B
L 2
1
f

=
Se por outro lado o tubo for aberto numa das extremidades, as
freqncias de ressonncia so dadas por:
1 n
nf
L 4
v
n f = =
com n = 1, 3, 5, 7,... e , f
0
1
B
L 4
1

= que bem parecido com o caso de uma


corda com uma das extremidades livre. Finalmente, quando o tubo aberto
nos dois lados, as freqncias de ressonncia so as mesmas do que quando o
tubo completamente fechado. Entretanto, aparecero ventres ao invs de ns
nas extremidades do tubo.
10.7 Funes de onda no caso estacionrio
As ondas estacionrias so formadas pela superposio de duas ondas
que se propagam em direes opostas. Consideremos ondas que se propagam
para a direita e para a esquerda, com funes de ondas das por:
( ) t kx sen Y Y
0 D
=






Movimento ondulatrio


S. C. Zilio e V. S. Bagnato Mecnica, calor e ondas


210
( ) t kx sen Y Y
0 E
+ =
A funo da onda resultante consiste na soma de Y
D
e Y
E
:
( ) ( ) [ ] t kx sen t kx sen Y Y Y Y
0 E D
+ + = + =
kx sen t cos Y 2 Y
0
=
onde na ltima passagem usamos a relao:
( ) ( )
2
B A
cos
2
B A
sen 2 B sen A sen
+
= +
Imagine agora uma corda fixa em x = 0 e x = L. Temos, portanto, Y(0)
= Y(L) = 0, para qualquer tempo. Desta forma,
L
n
k n L k 0 kL sen
n n

= = =
Como , / 2 k
n n
= recuperamos a condio de ressonncia
2
n L
n

= n = 1, 2, 3, ....
Por outro lado, se a extremidade da corda for livre, temos: senkL = 1. Logo,
2
n L k
n

= com n = 1, 3, 5, 7, .... Usando , k
n
n
2

= obtemos
4
n
n L

=
10.8 Interferncia
Vamos imaginar duas fontes de ondas separadas por uma distncia d e
um observador localizado sobre a linha que une as duas fontes. As ondas, que
suporemos de mesma freqncia, podem se adicionar, pois representam
perturbaes geradas no meio que podem ser somadas se o meio for linear.
Isto conhecido como princpio da superposio. Assim, de acordo com a Fig.
10.12, temos:
( ) t kx sen Y Y
0 1
=






Movimento ondulatrio

S. C. Zilio e V. S. Bagnato Mecnica, calor e ondas


211
( ) + = t kx sen Y Y
0 2

onde = kd a diferena de fase que aparece devido ao fato das fontes
estarem separadas. A onda resultante :
( ) ( ) [ ] + + = + = t kx sen t kx sen Y Y Y Y
0 2 1

=
2
t kx sen
2
cos Y 2 Y
0





Fig. 10.12 Interferncia entre duas ondas.
Dependendo do valor de , a onda resultante poder ser m10ima ou
mnima. Assim, se ( ) ,... 2 , 1 , 0 n n
2
kd
2
= = =

, a onda ser da forma:


( ) + = n t kx sen Y 2 Y
0

que apresenta uma onda duas vezes mais intensa do que cada onda isolada.
Este efeito chamado de interferncia construtiva. Por outro lado, quando
( ) ( ) ,... 2 , 1 , 0 n n
2
1
2
= + =

teremos y = 0 e neste caso temos o que se


chama de interferncia destrutiva.
d
(1) (2)
observador






Movimento ondulatrio


S. C. Zilio e V. S. Bagnato Mecnica, calor e ondas


212
Exerccios
1 - Uma corda vibra de acordo com a equao y(x,t) = 15sen( ) ( ), t 30 cos .
4
x


sendo x e y medidos em cm e t em segundos.
a) Qual a velocidade de um elemento da corda na posio x = 2 cm no
instante t = 2 s?
b) Qual a velocidade de propagao desta onda?
2 - Discuta as evidncias experimentais (que voc observa) que nos leva a
admitir que a velocidade do som na faixa audvel deve ser a mesma para
todos os comprimentos de onda.
3 - Suponha que no efeito Doppler com o som, a fonte e o observador estejam
ambos em repouso, mas o meio est se movendo com relao a este
referencial. Haver alguma variao na freqncia recebida pelo
observador?
4 - Na Fig. 10.13, uma haste est fixa pelo centro a um vibrador. Um disco
preso extremidade da haste penetra num tubo de vidro onde foi
espalhado p de cortia. Na outra extremidade do tubo existe um pisto
mvel. Produzindo-se vibraes longitudinais na haste, observar que para
determinadas posies do pisto mvel, o p de cortia forma um
conjunto de ns e anti-ns. Se para uma destas posies do pisto,
conhecermos a distncia d entre os anti-ns e a freqncia f de vibrao,
mostre que a velocidade do som no gs v = 2fd. Este o mtodo de
Kundt para determinar a velocidade do som.



Fig. 10.13

d anti-ns






Movimento ondulatrio

S. C. Zilio e V. S. Bagnato Mecnica, calor e ondas


213
5 - Um tubo pode funcionar como filtro acstico, discriminando as vrias
freqncias dos sons que o atravessam, das suas freqncias prprias. O
silencioso de um automvel um exemplo disto.
a) Explique o funcionamento deste filtro.
b) Determinar a freqncia de corte abaixo da qual o som no
transmitido.
6 - O comprimento de uma corda de violino de 50 cm e sua massa de 2.0 g.
Quando ela presa pelos extremos a corda pode emitir a nota l (440 Hz).
Onde deve ser colocado o dedo para que a nota emitida seja o d (528
Hz)?
7 - Considere uma fonte que emite ondas de freqncia f
0
movendo-se com
velocidade v
f
sobre o eixo x. Considere um observador movendo-se com
velocidade v
0
tambm sobre o eixo x. Qual ser a freqncia percebida
pelo observador? Chame a velocidade de propagao da onda de v.
















Movimento ondulatrio


S. C. Zilio e V. S. Bagnato Mecnica, calor e ondas


214







Gravitao

S. C. Zilio e V. S. Bagnato Mecnica, calor e ondas


215






11.1 Introduo
A lei de Newton da gravitao comumente expressa pela relao:
12
2
2 1
12 r
r
M M
G F =
Esta lei refere-se fora entre duas massas pontuais. Uma questo que
pode ser colocada como aplic-la ao clculo da fora entre um pequeno
corpo e a Terra, ou entre a Terra e a Lua, etc., onde sabemos que as partculas
que formam estes corpos esto a diferentes distncias umas das outras e as
foras de atrao so de direes e mdulos diferentes. O prprio Newton
protelou por onze anos a publicao da sua lei at ficar convicto de sua
validade, porque no sabia provar matematicamente que o resultado da fora
de atrao exercida sobre ou por uma esfera homognea seria o mesmo se
considerasse a massa da esfera concentrada em seu centro. Para provar isso,
ele criou o clculo diferencial e integral.
Fazemos aqui a demonstrao dessa lei de uma forma simples. Vamos
comear calculando qual a fora que um anel de massa exerce sobre massas
pontuais colocadas sobre seu eixo. Tomemos um anel de raio r, largura t e
espessura y como mostra a Fig. 11.1.
A rea da seco transversal do anel ty. Vamos chamar de
densidade de massa do anel, ou seja, quanta massa existe por unidade de
volume. Seja m a massa de uma partcula colocada a uma distncia d do centro
do anel. Considerando um elemento especificado por um ngulo d do anel,
11
GRAVITAO






Gravitao


S. C. Zilio e V. S. Bagnato Mecnica, calor e ondas


216
localizado a distncia x da partcula de massa m, a fora que esta parte do anel
exerce sobre a partcula , usando a lei da gravitao, dada por:
2
x
mdM
G dF =






Fig. 11.1 - Geometria para o clculo da fora devido a um anel.
A massa dM do elemento do anel pode ser determinada como:
dM = dV = r d y t
de modo que a fora fica:
2
x
d r t y m
G dF

=
Mas, usando o teorema de Pitgoras, x
2
= d
2
+ r
2
, ficamos com:

= d
r d
r t y m
G dF
2 2

Essa fora, que um vetor, pode ser decomposto numa componente
paralela ao eixo do anel e numa componente perpendicular a ele. Como a todo
elemento de massa do anel, existe outro igual e diametralmente oposto (de
modo a cancelar a componente de fora perpendicular ao eixo), s devemos
levar em conta a componente paralela. Assim,
dF
//
= dF cos
e, sendo cos =
2 2
r d / d + ficamos com

d
m d
x
r
F d
r
t
y






Gravitao

S. C. Zilio e V. S. Bagnato Mecnica, calor e ondas


217
( )

+

= d
r d
d r t y
Gm dF
2
3
2 2
//

Essa a fora devido ao elemento de massa considerado. Como
queremos a fora exercida pelo anel como um todo, devemos somar as
contribuies de todos os elementos de massa. Este justamente o objetivo da
integrao, o de somar todas as contribuies, que em termos prticos
significa fazer o ngulo da Fig. 11.1 variar deste 0 at 2. Ento,
( ) ( )

+

=
+

= =

=
=
d
r d
d r t y m G
d
r d
d r t y m G
dF F
2
0
2
3
2 2
2
3
2 2
2
0
//
2
0

Realizando a integrao, temos:
( )
2
3
2 2
r d
r d t y m G 2
F
+

=
A direo desta fora justamente a do eixo do anel, e o sentido o de
atrao. Uma vez que conhecemos a fora exercida por um anel, vamos
calcular a fora exercida por uma casca esfrica de espessura x e raio r, sobre
uma massa m, distante d do centro da esfera, como mostrado na Fig. 11.2.









Fig. 11.2 Geometria para o clculo da fora gravitacional devido a uma casca
esfrica de massa.
r
d
m
d
x
F d
r
y

Rsen






Gravitao


S. C. Zilio e V. S. Bagnato Mecnica, calor e ondas


218
fcil ver que uma esfera pode ser formada por vrios anis
justapostos. A idia ento dividir a esfera em anis, usar a equao para cada
um desses anis e somar sobre todos eles, obtendo a fora total exercida pela
esfera.
Vamos considerar um determinado anel que forma a esfera, definido
por um ngulo d, na posio . Sua largura rd, o raio rsen e ele est a
uma distncia x da partcula de massa m. Assim, para utilizarmos o resultado
do anel, devemos fazer as seguintes substituies:
d x, r r sen, t rd
Com este procedimento, temos que a fora devido ao anel mostrado na
Fig. 11.2 sobre a partcula de massa m :
( )
2
3
2 2 2
2
sen r x
d sen r x y m G 2
dF
+

=
Como nesta expresso temos duas variveis (x e ), devemos deix-la
como funo de uma nica varivel, para que se possa realizar a integrao.
Como x = d r cos, vem que
dx = r sen d
Se substituirmos x na expresso x
2
+ r
2
sen
2
, obteremos:
x
2
+ r
2
sen
2
= d
2
+ r
2
2dr cos =
d
2
+ r
2
+ 2d(x-d) = r
2
d
2
+ 2dx
Assim, a fora dada por:
( )
2
3
2 2
xd 2 d r
dx x r y m G 2
dF
+

=
Para termos a fora devido toda distribuio esfrica de massa,
devemos somar (integrar) as contribuies de todos os anis entre x = d r e x






Gravitao

S. C. Zilio e V. S. Bagnato Mecnica, calor e ondas


219
= d + r, pois a esfera formada pela superposio de anis nesse intervalo.
Logo,
( )
2
3
2 2
r d
r d
r d x
r d x
xd 2 d r
xdx
yr Gm 2 dF F
+
= =

+

+ =
=

Essa integral pode ser facilmente calculada. No caso geral, temos a
integral:
( )

+ = dx b ax x I
n

Fazendo a substituio ax + b = , temos x = a / ) b ( , de modo que
dx = d/a e, portanto:
( )
( ) ( ) 1 n
a
b
2 n
a
1
d
a
b
a
a
d
a
b
I
1 n
2
2 n
2 2
n
2
1 n
n
+

=
(


=
+ + +


Substituindo por ax + b, ficamos com:
( )
( )
( ) ( )
( )

+
+

+
+
= +
= +
1 n
b ax
a
b
a
b ax
2 n
1
dx b ax x
1 n
2 2
2 n
n

Logo a nossa integral em dF identifica-o com a integral se fizermos: n = -3/2,
a = 2d e
2 2
d r b = . Substituindo estes valores, ficamos com:
( )
( ) ( )
( )
r d x
r d x
2
1
2 2
2
2 2
2
2
1
2 2
2
3
2 2
r d
r d
dx 2 d r
) d 2 (
d r 2
) d 2 (
x . d . 2 d r 2
dx
x . d 2 d r
x
+ =
=

+
(
(

+
=
+


Substituindo os limites de integrao e notando que:
( )
( )
( )

=
+ = +
= +
r - d x para , r d
r d x para , r d
dx 2 d r
2
1
2 2

temos:






Gravitao


S. C. Zilio e V. S. Bagnato Mecnica, calor e ondas


220
( )

=
+
r d
r d
2
2
3
2 2 d
r 2
dx 2 d r
xdx

e, portanto:
2
2
d
yr Gm
4 F

=
Como o volume da casca esfrica de raio r e espessura y V = 4r
2
y,
temos que M = 4r
2
y massa total contida na distribuio esfrica de massa,
de modo que
2
d
GmM
F =
lembrando novamente que esta uma fora de atrao. Assim, como se
tivssemos duas massa pontuais separadas por uma distncia d. Portanto, em
termos gravitacionais, uma distribuio homognea e esfrica de massa se
comporta como se toda sua massa estivesse concentrada no seu centro.
claro que fizemos os clculos para uma casca esfrica, mas o
resultado vale para uma esfera macia, pois esta pode ser vista como sendo
composta de vrias cascas esfricas de raios variando entre 0 e R, bastando
ento tomar a soma delas. O resultado obtido vale para pontos fora da esfera.
Se tivermos uma distribuio que uma casca esfrica de raio r e colocarmos
no seu interior uma partcula de massa m, ento devemos refazer as integrais e
obtermos que a fora total exercida sobre a partcula nula.
Assim, um outro resultado importante uma partcula de massa m
colocada no interior de uma casca esfrica com densidade de massa uniforme,
fica sujeita a uma fora nula.






Gravitao

S. C. Zilio e V. S. Bagnato Mecnica, calor e ondas


221
Exerccios
1- Calcule a fora gravitacional que uma partcula de massa m fica sujeita
quando colocada no interior da Terra, a uma distncia r de seu centro.
2- Consideremos duas cascas esfricas concntricas de densidades uniformes
de massa M
1
e M
2
como mostra a Fig. 11.3. Calcule a fora sobre uma
partcula de massa m colocada em a, b ou c.





Fig. 11.3
3- Faz-se uma cavidade esfrica numa esfera de chumbo de raio R tal que
sua superfcie toque a superfcie externa da esfera macia e passe pelo
centro dessa. A massa primitiva da esfera de chumbo M. Qual ser a
fora que a esfera com a cavidade atrair uma massa m a uma distncia d
do centro da esfera externa, de modo que a massa e o centro da esfera e da
cavidade estejam alinhados? (Questo retirada do exame olmpico da
Universidade Estatal de Moscow (1946)).
4- Mostrar que num tnel cavado atravs da Terra, ao longo de uma corda e
no ao longo de um dimetro, o movimento de um objeto ser harmnico
simples.
5- Mostrar atravs de argumentos geomtricos que uma partcula de massa m
colocada no interior de uma casca esfrica de densidade uniforme de
massa fica sujeira a uma fora nula, qualquer que seja a posio da
partcula. O que aconteceria se a densidade superficial de massa no fosse
constante?
c
M
2

M
1

a b






Gravitao


S. C. Zilio e V. S. Bagnato Mecnica, calor e ondas


222
6- Considere o movimento de um mssel intercontinental, lanado segundo
inclinao
0
como mostrado na Fig. 11.4, com velocidade v
0
, na posio
indicada. Calcule a trajetria do corpo.






Fig. 11.4
7- Trs corpos idnticos de massa M esto localizados nos vrtices de um
tringulo eqiltero de lado L. A que velocidade eles devem mover-se se
todos giram sob a influncia da gravidade mtua, em uma rbita circular
que circunscreve o tringulo, mantido sempre eqiltero?
8- Considere um anel macio de raio R e massa M. Colocamos uma partcula
de massa m a uma distncia d do plano do anel de modo que quando solto
o corpo tem trajetria sobre a reta perpendicular ao plano do anel
passando pelo centro do mesmo. Calcule o movimento do corpo de massa
m (<<M).
9- Um corpo de massa m colocado a uma distncia r
0
do centro de um
planeta de massa M e raio R. Calcule a velocidade como funo de r.
10- Considere duas massas m e 2m com atrao gravitacional. Com que
velocidade angular elas devem rodar tal que a distncia d entre elas fique
constante?
11- Um corpo de massa m colocado a uma distncia r
0
do centro de um
planeta de massa M e raio R. Calcule a energia potencial para 0 r .
Suponha que a densidade de massa do planeta seja uniforme e que a massa
R

0

x

0

y
v
0







Gravitao

S. C. Zilio e V. S. Bagnato Mecnica, calor e ondas


223
m possa entrar no seu interior atravs de um tnel. Considere v() = 0.
Calcule a velocidade como funo de r para r < R sabendo que v(r
0
) = 0.
12- Considere um anel de massa M e raio R e uma partcula de massa m
colocada no seu centro. Qual a freqncia para oscilaes de pequena
amplitude na direo perpendicular ao plano do anel?







Gravitao


S. C. Zilio e V. S. Bagnato Mecnica, calor e ondas


224







Mecnica dos fluidos

S. C. Zilio e V. S. Bagnato Mecnica, calor e ondas


225






12.1 Introduo
De um modo geral, denomina-se fludo o meio material cuja forma
geomtrica depende vizinhana com a qual ele se encontra em contato. Este
tipo de material pode ainda escoar de um lugar para outro quando sujeito a
foras externas. De acordo com estas propriedades, podemos notar que os
gases e lquidos so classificados com fludos.
Nosso estudo de mecnica dos fludos comear pela hidrosttica, que
se refere ao caso particular em que o fludo se encontra em repouso.
Posteriormente, trataremos o caso em que o fluido encontra-se em movimento,
descrito pela hidrodinmica. Em ambos casos, ao invs de tratarmos a fora
atuante sobre o sistema, usaremos o conceito de presso, que definida como
a fora aplicada por unidade de rea:
|

\
|

=

S
F
lim P
0 S

onde F a fora agindo sobre o elemento de rea S. importante notar que
a presso se transmite s superfcies de um recipiente ou atravs de seces
arbitrrias de fludo sempre perpendicularmente a estas superfcies. Por
exemplo, se colocarmos um cubo de alumnio dentro de uma panela com gua,
as foras provocadas pelo fludo sero perpendiculares s suas faces. A
presso uma grandeza escalar, com unidades de N/m
2
(Pascal, Pa), dyn/cm
2
,
bar (= 10
5
N/m
2
), atm (1.01 x 10
5
N/m
2
), etc. Em mecnica dos fludos muito
comum trabalharmos com densidade de massa, definida como = dM/dV
onde dM a massa contida no volume dV.
12
MECNICA DOS
FLUIDOS






Mecnica dos fluidos


S. C. Zilio e V. S. Bagnato Mecnica, calor e ondas


226
12.2 Hidrosttica
Iniciamos o estudo deste tpico pela lei de Stevin, que estabelece a
presso de um fludo sujeito gravidade. Considerando um fludo em repouso,
vamos analisar um pequeno elemento de volume de rea A e espessura dy,
como mostrado na Fig. 12.1.






Fig. 12.1 Elemento de volume usado para a demonstrao da lei de Stevin,
A massa desta poro de fludo dada por M = V = Ay. As vrias
foras perpendiculares rea lateral cancelam-se mutuamente, pois o meio
isotrpico. O equilbrio de foras ao longo da vertical estabelece que:
[P(y + y) - P(y)]A + mg = 0
[P(y + y) - P(y)]A = -Agy
( )
g
dy
dP
y
) y ( P y y P
lim
0 y
= = |

\
|

+


Assim, conclumos que a presso diminui com a altura (taxa negativa) de uma
maneira proporcional a e g. No caso particular em que e g independem de
y (portanto constantes), a equao acima pode ser integrada entre dois pontos
quaisquer, resultando em:
P
2
= P
1
- g(y
2
y
1
)
Um dos casos mais comuns que aparece na literatura quando y
1
est
na superfcie e y
2
no interior do lquido tal que y
2
< y
1
. Chamado y
1
y
2
= h e
P
1
= P
a
(presso atmosfrica), obtemos:
P(h) = P
a
+ gh
A
P(y)
P(y+y)
y
y






Mecnica dos fluidos

S. C. Zilio e V. S. Bagnato Mecnica, calor e ondas


227
Como exemplo da aplicao desta lei, vamos analisar como varia a
presso do ar atmosfrico como funo da altitude. A densidade varia com y,
como veremos a seguir, mas suporemos que a temperatura seja constante.
Partindo da equao dos gases ideais (equao de Clapeyron) temos:
M
T K
M
T K
V
m
P T K
M
m
T NK PV
B B
B B
= = = =
onde m a massa de gs contida no volume V, M a massa de cada molcula
e K
B
a constante de Boltzmann. Como supusemos que T constante, temos:
a
0
P
P
=
onde P
a
a presso na superfcie e
0
, a densidade de massa neste ponto.
Como vimos anteriormente,
P
P
g
g
dy
dP
a
0

= =
dy
P
g
P
dP
a
0

=
Integrando esta expresso de y = 0 at y = h obtemos:
)
`


= y
P
g
exp P P
a
0
a

Tomando P
a
= 1atm, g = 9.8 m/s
2
e
0
= 1.2 Kg/m
3
, temos
0
g/P
a
= 0,116 Km
-1

e assim podemos fazer um grfico de P(y) como mostrado na Fig. 12.2.
Um outro exemplo comum onde a lei de Stevin aplicada o dos
vasos comunicantes, mostrados na Fig. 12.3, onde so colocados dois lquidos
no miscveis, de densidades diferentes. De acordo com o princpio de Pascal,
a presso aplicada a um fluido contido num recipiente transmitida
integralmente a todos os pontos do fludo e s paredes do recipiente.
Como a presso na altura y
0
, definida pela linha horizontal na altura L
a mesma nos dois lados do recipiente, temos:






Mecnica dos fluidos


S. C. Zilio e V. S. Bagnato Mecnica, calor e ondas


228
P
a
+
1
gh
1
= P
a
+
2
gh
2

1
h
1
=
2
h
2

0 10 20 30 40 50
0.0
0.2
0.4
0.6
0.8
1.0
P
r
e
s
s

o

(
a
t
m
)
y (Km)

Fig. 12.2 Variao da presso atmosfrica com a altura.





Fig. 12.3 Vasos comunicantes.
Um terceiro exemplo deste tpico o de um fludo em rotao.
Consideremos um liquido colocado dentro de um recipiente cilndrico, que
roda com velocidade angular constante, , em torno do eixo de simetria do
cilindro, de acordo com a Fig. 12.4. Queremos determinar o formato da
superfcie do cilindro.
Vamos supor que as coordenadas de um ponto qualquer desta
superfcie sejam r e y, onde r tem sua origem no eixo de simetria do cilindro e
y no ponto mais baixo da superfcie. Vamos tomar um elemento de volume no
interior do lquido, na forma de um anel de raio r e espessura r. Chamaremos
a rea lateral deste anel de A. A diferena entre as foras existentes nas reas
laterais, externa e interna do anel deve ser igual fora centrpeta. Portanto,
[P(r + r) - P(r)] A = m
2
r = Ar
2
r
h
2
y
0
L

h
1






Mecnica dos fluidos

S. C. Zilio e V. S. Bagnato Mecnica, calor e ondas


229









Fig. 12.4 Fluido em rotao.
( ) ( )
r
dr
dP
r
r P r r P
lim
2
0 r
= =
(

+


Logo, P(r) = P
a
+
2 2
2
1
r ,

pois o ponto r = 0 pertence superfcie, cuja
presso a atmosfrica (P
a
). Por outro lado, a presso independe da direo e
se olharmos para a presso no ponto r ao longo da vertical, ela ser dada pela
lei de Stevin:
P(r) = P
a
+ gy
onde y a altura da coluna de lquido sobre o ponto r. Igualando as presses
calculadas nas direes radial e vertical temos:
( )
g 2
r
r y
2 2

=
de onde conclumos que a superfcie do lquido forma uma parbola.
12.3 Princpio de Arquimedes
Um corpo imerso num fludo qualquer tem seu peso aparentemente
diminudo. Isto pode ser explicado pelo princpio de Arquimedes, que veremos
a seguir. Vamos considerar um cubo de aresta L imerso num fludo como
mostra a Fig. 12.5.
y
r
y = 0
y

r
r
A
P(r)
P(r+r)






Mecnica dos fluidos


S. C. Zilio e V. S. Bagnato Mecnica, calor e ondas


230
O peso do corpo dirigido para baixo e vale
Vg Mg
c
= =







Fig. 12.5 Cubo imerso num fluido.
Por outro lado, sabemos que a presso na parte inferior do cubo maior que
na superior e vale:
P
2
= P
1
+ gL
Desta forma, temos uma fora dirigida para cima, chamada de forca de
empuxo, cujo valor :
F = (P
2
P
1
)A = gLA = Vg
sendo, portanto, igual ao peso do volume de lquido deslocado. Assim, todo
corpo imerso ou parcialmente imerso sofre uma fora oposta gravidade, que
igual ao peso do fludo deslocado. Este resultado conhecido como
princpio de Arquimedes. A fora exercida sobre corpos submersos aparece
devido ao fato de que o fludo exerce presso em todos os pontos do corpo,
mas as regies de maior profundidade esto sujeitas a presses mais elevadas.
Assim, existe sempre uma fora de empuxo na vertical e de baixo para cima.
Esta fora no depende do formato do corpo, embora ns tenhamos usado um
cubo para deduzir sua expresso. Poderamos, por exemplo, ter usado uma
esfera para esta deduo. Neste caso, para a esfera de raio R mostrada na Fig.
12.6, a presso funo do ngulo :
P() = P
0
+ R(1 - cos )g
P
2

P
1

L






Mecnica dos fluidos

S. C. Zilio e V. S. Bagnato Mecnica, calor e ondas


231
A rea mostrada na figura : dA = 2Rsen Rd e a fora exercida sobre ela :










Fig.12.6 Fora de empuxo sobre uma esfera.
dF = P() dA = P() 2R
2
sen d
As componentes horizontais desta fora se cancelam mutuamente, restando
apenas a componente vertical dF
y
= dF cos. Logo, a fora resultante na
direo y :
| | + = =


d cos sen g ) cos 1 ( R P R 2 cos dF F
0
0
2
0
y

Fazendo cos = u temos du = - sen d e assim:
| | du u g R 2 du u g ) u 1 ( R P R 2 F
1
1
2 3
1
1
0
2
y


= + =
Vg g R
3
4
F
3
y
= =
12.4 Dinmica dos fludos
Para descrever-se o movimento dos fludos usual analisar-se o
comportamento de elementos infinitesimais de volume deste fludo, isto ,
necessrio conhecer-se ( ) ( ) t , r v e t , r
r r
para cada ponto e instante de tempo.
No caso particular em que v independe do tempo, temos um fluxo estacionrio
R
d

P()






Mecnica dos fluidos


S. C. Zilio e V. S. Bagnato Mecnica, calor e ondas


232
(ou lamelar). Caso contrrio, temos um fluxo no estacionrio ou turbulento.
Alguns outros conceitos importantes na anlise da dinmica dos
fluidos devem ser introduzidos. Por exemplo, se variar com r
r
ou t, o fluido
compressvel e quando isto no ocorre temos ento um fludo
incompressvel. Analogamente ao atrito, o escoamento de um fluido pode ser
viscoso. A viscosidade introduz foras tangenciais entre camadas do fludo
que possuem movimento relativo, resultando em dissipao de energia.
Quando o fluxo lamelar, a trajetria de um determinado elemento do
fludo chamada de linha de corrente. Da mesma forma que na mecnica da
partcula, a velocidade do fludo tangente linha de corrente. Se tomarmos
as linhas de corrente que tangenciam uma dada rea, como mostrado na Fig.
12.7, teremos um tubo de corrente ou tubo de escoamento. No h transporte
de matria pela superfcie lateral deste tubo, j que por definio, o
movimento do fluido sempre tangente a esta superfcie.








Fig. 12.7 Tubo de corrente.
Durante um certo intervalo de tempo t, a quantidade de massa transportada
atravs das superfcies A e B, de reas S
A
e S
B
:
t v S m
A A A A
=
t v S m
B B B B
=
O fluxo de massa definido por t / m = . Como a massa no est
sendo criada nem destruda, m
A
= m
B
. Logo:
A
B






Mecnica dos fluidos

S. C. Zilio e V. S. Bagnato Mecnica, calor e ondas


233
B B B A A A
S v S v =
ou seja, vS = = constante ao longo do tubo de corrente. Quando
constante (fludo incompressvel) temos = Sv = constante, que chamada
de vazo.
12.5 Teorema de Bernouilli
Os conceitos que vamos discutir agora so importantes para a anlise
do empuxo sobre as asas de um avio, para a medida da velocidade de um
fludo, etc. Consideremos o escoamento de um fludo no viscoso e
incompressvel atravs de um de corrente mostrado na Fig. 12.8.









Fig. 12.8 Tubo de corrente usado para a demonstrao do teorema de Bernouilli.
No lado esquerdo do tubo, temos uma rea S
1
e fludo escoando com
velocidade v
1
a uma altura y
1
e presso P
1
. No lado direito, estas grandezas so
dadas respectivamente por S
2
, v
2
, y
2
e P
2
.
Inicialmente vamos calcular o trabalho feito pelas foras F
1
e F
2
sobre
o volume do fludo compreendido entre (1) e (2) durante um intervalo de
tempo t.
t v S P t v S P F F W
2 2 2 1 1 1 2 2 1 1
= = l l
como Svt = m/ (= volume), podemos re-escrever W como:
1
2
U = 0
v
1

v
2

l
1

l
2

F
1
= P
1
S
1

F
2
= P
2
S
2

y
1

y
2







Mecnica dos fluidos


S. C. Zilio e V. S. Bagnato Mecnica, calor e ondas


234
( )
2 1
P P
m
W

=
Este trabalho produz uma variao na energia mecnica do sistema,
que pode ser a como se a regio achurada em (1) da Fig.12.8, tenha sido
promovida a (2). Assim,
1
2
1 2
2
2 1 2
mgy mv
2
1
mgy mv
2
1
E E E + = =
Igualando E com W e cancelando m, obtemos:
( )
1 2
2
1
2
2 2 1
gy gy v v P P
1
2
1
2
1
+ =


2
2
2 2 1
2
1 1
gy v P gy v P
2
1
2
1
+ + = + +
de onde conclumos que a grandeza P + gy v
2
2
1
+

constante ao longo do
tubo de corrente. O termo gy vem da lei de Stevin e
2
2
1
v

a presso
dinmica, enquanto que P a presso esttica. A igualdade acima conhecida
como teorema de Bernouilli. Se o meio for viscoso, temos que incluir nesta
equao um termo representando a dissipao de energia. A seguir, vamos
analisar alguns exemplos onde a equao de Bernouilli se aplica.
O tubo de Venturi, mostrado na Fig. 12.9, utilizado para a medida da
velocidade de escoamento de um fludo.








Fig. 12.9 Tubo de Venturi.
h
1

h
2

h
v
rea a
rea A
1
2






Mecnica dos fluidos

S. C. Zilio e V. S. Bagnato Mecnica, calor e ondas


235
A presso esttica no ponto 1 dada por P
1
= P
a
+ gh
1
e no ponto 2,
P
2
= P
a
+ gh
2
. Levando-se em conta que os pontos 1 e 2 esto mesma altura,
o teorema de Bernouilli nos d:
2
2 2
2
1 1
v P v P
2
1
2
1
+ = +
onde v
1
= v e v
2
= Av/a. Esta ltima igualdade vem do fato que a vazo
constante (Av = av
2
). Portanto,
2
2
2
2 a
2
1 a
v
a
A
gh P v gh P
2
1
2
1
+ + = + +
( ) ( ) gh h h g h h g
a
A
1 v
2 1 1 2
2
2
2
2
1
= = =
|

\
|

e consequentemente,
( ) 1
a
A
gh 2
v
2

=
O empuxo dinmico a denominao dada fora que surge sobre um
corpo que se desloca dentro de um fludo (ao contrrio do empuxo esttico do
princpio de Arquimedes). Consideremos uma bola movimentando-se com
velocidade v dentro de um fludo. Por convenincia, usaremos um referencial
solidrio a bola, no qual ela est em repouso e o fludo desloca-se com
velocidade -v. Se a bola estiver rodando com velocidade angular , os pontos
de sua periferia arrastaro as molculas do ar, modificando sua velocidade. De
acordo com a Fig. 12.10 as velocidades nos pontos (1) e (2) so dadas por v
1
=
v
aR
- R e v
2
= v
aR
+ R. Aparecer, portanto, uma diferena de presso dada
por:
( ) ( ) ( ) | |
R v 2 P P
v v R v v v v P P
aR 2 1
2
R aR
2
aR
2
1
2
1
2
2
2
1
2 1
=
+ = =







Mecnica dos fluidos


S. C. Zilio e V. S. Bagnato Mecnica, calor e ondas


236
presso maior
presso menor
bordo de ataque
bordo de fuga








Fig. 12.10 Bola com rotao movendo-se num fluido.
Esta diferena de presso produz uma fora F, que obriga a bola a descrever
uma trajetria curva. Note que se = 0, esta fora ser nula e a trajetria da
bola retilnea.
Um outro exemplo de empuxo dinmico ocorre com a asa de um
avio ou com o aeroflio de um carro de corrida. Na asa do avio, mostrada na
Fig. 12.11, o ar percorre uma distncia maior na parte de cima, tendo portanto,
maior velocidade naquela regio. Como conseqncia, a presso na parte de
baixo da asa maior do que em cima e isto d sustentao ao avio. J no
aeroflio de um carro de corrida, temos uma asa invertida que provoca uma
fora para o cho.









Fig. 12.11 Asa de avio.
Como um ltimo exemplo do teorema de Bernouilli, vamos considerar

v
ar

v
ar
-R
1
2
v
ar
+R
R
F
r








Mecnica dos fluidos

S. C. Zilio e V. S. Bagnato Mecnica, calor e ondas


237
o escoamento de um lquido atravs de um orifcio situado na lateral de um
grande reservatrio, mostrado na Fig. 12.12. Numa primeira etapa,
desprezaremos o movimento da superfcie do lquido no tanque.






Fig. 12.12 Fluido vazando de um reservatrio.
Nestas condies, teremos entre os pontos (1) e (2):
2
2 2
2
1 1
v
2
1
0 g P v
2
1
gh P + + = + +
Como estamos considerando
a 2 1 1
P P P e 0 v = = temos:
gh 2 v v
2
= =
Vamos agora levar em conta a velocidade da superfcie superior de
rea A para calcular a velocidade com que o lquido emerge do orifcio de rea
a. Como a vazo constante temos A v
1
= a v
2
v
1
= v
2
a/A

= v a/A.
Substituindo na equao de Bernouilli dada acima ficamos com:
2
2
2
v
A
a
v gh
2
1
2
1
=
|

\
|
+
( )
2
A / a 1
gh 2
v

=
Como em geral a << A, podemos expandir o denominador em srie de Taylor:
( )
(

+ + .....
A
a
2
1
1 gh 2 v
2

h
1
2
v






Mecnica dos fluidos


S. C. Zilio e V. S. Bagnato Mecnica, calor e ondas


238
12.6 Viscosidade
Vamos considerar um fludo colocado entre duas placas, onde a
superior desloca-se com velocidade v como mostra a Fig. 12.13. Nesta
situao surge sobre a placa inferior uma tenso de cisalhamento (fora/rea
da placa) tentando arrast-la junto com a outra.




Fig.12.13 Placa arrastada num fluido.
necessrio, portanto, uma tenso para mant-la em repouso.
Experimentalmente determina-se que:
y
v

=
onde a constante chamada de viscosidade do fludo. Note que a
viscosidade est relacionada com o poder que um fludo em movimento tem
de arrastar os corpos em contato com ele ou as camadas vizinhas do fludo.
Isto bastante semelhante ao atrito existente entre dois corpos em contato,
com movimento relativo. Nem todos os fludos obedecem a expresso acima;
aqueles que obedecem so chamados de fludos newtonianos. Um outro ponto
importante que a expresso acima s vlida para fluxos estacionrios
(escoamento lamelar).
Vamos considerar o escoamento lamelar de um fludo viscoso atravs
de um tubo de dimetro D (Fig. 12.14). Analisaremos o balano de foras
sobre um elemento de volume de raio r.
O deslocamento deste elemento de volume no interior do fludo gera a
tenso de cisalhamento indicada na figura, que funciona como atrito sobre a
poro considerada. Por outro lado, existe uma variao de presso ao longo
do tubo e como o fluxo estacionrio, a somatria das foras sobre o
elemento de volume nula:
y

v






Mecnica dos fluidos

S. C. Zilio e V. S. Bagnato Mecnica, calor e ondas


239








Fig. 12.14 Escoamento por um tubo.
= rL 2 P r
2

dr
dv
L 2
r
P = =
onde nesta ltima passagem usamos o fato de se tratar de um fludo
newtoriano e que v diminui com r. Logo,
( ) rdr
L
P
2
1
dv

=
Para encontrarmos v(r) faremos uma integral de r = 0 at r. No centro
do tubo (r = 0) a velocidade mxima (v
max
). Portanto,
v(r) = v
max
-
2
r
4
1
L
P

\
|

onde P/L a queda de presso por unidade de comprimento. Para
encontrarmos o valor de v
max
, levamos em conta que v = 0 para r = D/2.
Assim,
2
max
2
D
4
1
L
P
v
|

\
|

\
|
=
e, portanto,
( )
(

\
|
=
2
max
2 / D
r
1 v r v = ( ) ( )
(

\
|

2
2
2 / D
r
1
2
D
4
1
L
P

L
P +P
P


r






Mecnica dos fluidos


S. C. Zilio e V. S. Bagnato Mecnica, calor e ondas


240
que representa a parbola mostrada na Fig. 12.15.







Fig. 12.15 Distribuio radial de velocidades de um fluido escoando por um cano.
Desta forma, o perfil de velocidades para o escoamento lamelar de um
fluido numa tubulao parablico, sendo mximo no centro, como mostra a
Fig.12.16. como se fosse uma antena de carro sendo esticada.






Fig. 12.16 Fluido escoando por um cano.
Na prtica, estamos sempre interessados em encontrar a vazo
como funo da viscosidade, dimetro e comprimento da tubulao para um
certo valor de diferena de presso aplicada. Considerando o elemento de rea
em forma de anel mostrado na Fig. 12.17, temos:
d = v(r) dA = v
max
( ) rdr 2
D
r 2
1
2


Para encontrarmos o fluxo total, integramos de r = 0 at r = D/2:




v
max
r
v(r)
D/2
r






Mecnica dos fluidos

S. C. Zilio e V. S. Bagnato Mecnica, calor e ondas


241








Fig. 12.17 Elemento de rea usado para clculo da vazo.
2
A
v
8
D
v
max
2
max
= =
onde A a rea do tubo. Como A = R
2
, podemos ainda escrever:
|

\
|

=
L
P
R
8
4

de onde vemos que a vazo proporcional quarta potncia do raio. A
equao acima leva o nome de lei de Poiseuille.
Exerccios
1- Uma bola de madeira de densidade
m
est presa a uma profundidade h
num lquido de densidade
L
. Soltando-se a bola do repouso, determine
que altura acima da superfcie ela atingir.
2- Trs recipientes com fundos falsos (Fig. 12.18) foram colocados na gua,
a uma mesma profundidade. Colocando-se nos trs frascos a mesma
quantidade de leo, qual dos trs fundos cair primeiro? Justifique.
3- Um depsito retangular sem tampa, com as dimenses dadas na Fig.
12.19, move-se com acelerao a e contm gua at uma altura h (quando
a = 0). Para que o valor da acelerao a gua comear a escoar para fora?
4- Um cubo de um certo material flutua num recipiente contendo mercrio
v(r)
dr






Mecnica dos fluidos


S. C. Zilio e V. S. Bagnato Mecnica, calor e ondas


242
(
Hg
= 13.6g/cm
3
) tal que 1/4 de seu volume fica submerso.
Acrescentando-se ao sistema gua suficiente para cobrir o cubo (
gua
=
lg/cm
3
), que frao de seu volume ainda permanecer imersa no mercrio?




Fig. 12.18 Fig. 12.19
5- Uma tbua de comprimento L est apoiada numa pedra e parcialmente
imersa na gua. Conforme mostra a Fig. 12.20, uma poro de
comprimento a encontra-se acima do ponto de apoio. Sendo d a densidade
da madeira, que parte da tbua encontra-se submersa?





Fig. 12.20 Fig. 12.21
6- Dentro de um recipiente cnico coloca-se leite. Com o passar do tempo
ocorre formao de nata, que sendo menos densa fica no topo. Durante
este processo no h variao de volume, ist , h permanece constante
(Fig. 12.21). O que acontece com a presso no fundo do recipiente?
Justifique sua resposta.
7- Numa lata cilndrica de rea A coloca-se gua at uma altura h. Determine
a velocidade v com que a gua sai por um orifcio de rea a localizado no
fundo. Que quantidade de gua deve ser adicionada lata por unidade de
nata
h
a
H
h
a
L






Mecnica dos fluidos

S. C. Zilio e V. S. Bagnato Mecnica, calor e ondas


243
tempo tal que v seja constante?
8- Caso no se adicione gua na lata do problema anterior e a altura variar,
calcule a vazo como funo do tempo.
9- Com um sifo retira-se gua de um recipiente como indicado na Fig.
12.22. A rea do cano constante ao longo de seu comprimento e a
velocidade da superfcie do lquido desprezada. a) Qual a velocidade
da gua na sada do cano? b) Qual a presso no ponto mais alto do sifo?
c) Qual a mxima altura h para a qual ainda possvel sifonar a gua?




Fig. 12.22
10- Monta-se uma caixa dgua sobre um vago que pode se mover no plano
horizontal sem atrito (Fig. 12.23). Na parede da caixa existe um orifcio de
rea A a uma profundidade H, pelo qual sai gua paralelamente ao plano
horizontal. A massa total inicial do sistema (caixa, gua e vago) M
0
e a
velocidade da superfcie da gua desprezada. Se o vago est
inicialmente em repouso quando o orifcio aberto, qual ser a acelerao
inicial do sistema?



Fig. 12.23

11- Um tubo de gua roda com velocidade em torno de um eixo vertical
conforme mostra a Fig. 12.24. Calcule a presso como funo de r, usando
H
H
h






Mecnica dos fluidos


S. C. Zilio e V. S. Bagnato Mecnica, calor e ondas


244
P(r = 0) = P
0
.




Fig. 12.24
12- Um rotmetro (medidor de vazo) consiste num tubo de vidro cnico e
vertical com uma esfera metlica de massa m e raio r no seu interior como
mostra a Fig. 12.25. Calcule o fluxo de um gs de viscosidade como
funo da altura h. Considere bem pequeno. Nota: F
stokes
= 6rv.









Fig. 12.25

r

h







Termologia e termodinmica

S. C. Zilio e V. S. Bagnato Mecnica, calor e ondas


245






13.1 Introduo
Em geral, para caracterizarmos um sistema com N partculas
necessrio especificarmos a posio, velocidade e acelerao de cada partcula
como funo do tempo. No caso de um sistema macroscpico, o nmero de
partculas extremamente grande e esta tarefa se torna muito difcil. Uma
alternativa para abordar este problema a de trabalharmos com valores
mdios, que representam o comportamento do sistema como um todo. Vamos
comear esta abordagem definindo as grandezas macroscpicas que
determinam o estado do sistema.
Consideremos um gs constitudo de N molculas num recipiente de
volume V. Microscopicamente, o movimento de cada partcula retilneo
uniforme at que ela se choque com a outra molcula ou com as paredes do
recipiente. Este tipo de movimento, mostrado na Fig. 13.1, chamado de
browniano. A distncia mdia que a partcula percorre entre duas colises
sucessivas denomina-se caminho livre mdio.





Fig. 13.1 Representao do movimento browniano.
Os choques das partculas com as paredes do recipiente implicam em
transferncias de momentum e, conseqentemente, numa fora mdia exercida
sobre as paredes. Esta fora por sua vez d origem presso que o gs exerce
13
TERMOLOGIA E
TERMODINMICA






Termologia e termodinmica


S. C. Zilio e V. S. Bagnato Mecnica, calor e ondas


246
sobre as paredes e esta uma grandeza macroscpica (com origem
microscpica) que descreve uma propriedade mdia do sistema global. Alm
da presso, existem outras grandezas macroscpicas importantes para a
descrio do sistema: volume V, energia interna e temperatura, ambas
associadas ao movimento translacional, vibracional e rotacional das
molculas. O objetivo da termodinmica relacionar estas grandezas
macroscpicas, que podem ser medidas experimentalmente.
Quando as propriedades macroscpicas de um sistema no se alteram
com o tempo, dizemos que ele est em equilbrio termodinmico. Neste caso,
o sistema de interesse deve ser mantido em contato com um segundo sistema,
chamado de reservatrio ou banho trmico, que determina os parmetros do
equilbrio. O conjunto das grandezas macroscpicas associadas a um sistema
em equilbrio tem o nome de estado macroscpico. Convm notar que o
estado microscpico do sistema determina o estado macroscpico, porm a
recproca no verdade porque a partir de valores mdios impossvel
especificar-se r
r
e p
r
para todas as partculas do sistema.
As grandezas macroscpicas esto de alguma forma interconectadas.
Para verificarmos isto, podemos tomar um pisto contendo gs, como
esquematizado na Fig. 13.2, e aquec-lo. Neste caso, a temperatura do sistema
aumentar. Se mantivermos a posio do pisto fixa, haver um aumento de
presso. Se por outro lado deixarmos o pisto solto, haver um aumento de
volume. Assim, tanto o aumento da presso como o de volume so
conseqncias do aumento da temperatura, de onde conclumos que estas
grandezas esto, de alguma forma, relacionadas.





Fig. 13.2 Cilindro com pisto contendo gs.
P,V,T






Termologia e termodinmica

S. C. Zilio e V. S. Bagnato Mecnica, calor e ondas


247
Se tivermos dois sistemas trmicos em contato, importante sabermos
a maneira pela qual um interage com o outro. Esta interao freqentemente
feita atravs de paredes, como a mostrada na Fig. 13.3. Se a parede for fixa e
variarmos a temperatura de um dos sistemas, podemos ter duas situaes: (i) a
temperatura do outro sistema no se altera e, neste caso, temos uma parede
perfeitamente isolante (tambm chamada parede adiabtica) ou (ii) a
temperatura do outro sistema acompanha as mudanas do primeiro e, neste
caso, temos uma parede diatrmica.






Fig. 13.3 Interao entre dois sistemas atravs de uma parede.
No caso (ii), as temperaturas dos dois sistemas evoluem at atingirem
um valor comum. Quando as temperaturas dos dois sistemas forem iguais,
dizemos que eles esto em equilbrio trmico.
13.2 Medida da temperatura
Normalmente, a temperatura medida atravs da observao de
alguma grandeza sensvel sua variao. O sistema utilizado para este fim
chamado de termmetro, para o qual definida uma escala de temperatura. Os
exemplos mais comuns de termmetro so:
(i) Termmetro de Mercrio Coloca-se um certo volume de mercrio num
tubo capilar de vidro e observa-se a dilatao trmica do mercrio como
funo da temperatura. O comprimento da coluna de mercrio varia de forma
aproximadamente linear com a temperatura, e assim podemos escrever:
T = aL + b
Sistema 1

Sistema 2
isolante trmico

parede







Termologia e termodinmica


S. C. Zilio e V. S. Bagnato Mecnica, calor e ondas


248
onde T a temperatura, geralmente em graus Celsius, L o comprimento da
coluna de mercrio e a e b so duas constantes que dependem das
temperaturas de referncia escolhidas. Convencionalmente so escolhidas as
temperaturas de fuso do gelo (ponto trplice) como 0
0
C e de ebulio da
gua como 100
0
C, de forma a termos:
g V
g
g V
L L
L 100
b e
L L
100
a

=
(ii) Termmetro a gs J neste caso, a presso de um gs utilizada como
grandeza termomtrica. Conforme mostra a Fig. 13.4, o volume do gs
mantido constante movendo-se a coluna da direita e a medida da temperatura
est diretamente ligada leitura da altura h desta coluna.








Fig. 13.4 - Termmetro a gs.
Como veremos adiante, a presso e a temperatura de um gs so
proporcionais de forma que T = AP + B, com:
g V
g
g V
P P
P 100
B e
P P
100
A

=
onde P
g
a presso do gs no ponto trplice da gua (0
0
C) e P
V
a presso no
ponto de ebulio da gua (100
0
C

).
Informalmente, utilizamos acima a escala de temperatura Celsius (ou
centgrada), que bastante comum no nosso dia a dia. Esta escala usa como
h
T






Termologia e termodinmica

S. C. Zilio e V. S. Bagnato Mecnica, calor e ondas


249
referncia o ponto trplice da gua (0
0
C) e o ponto de ebulio da gua (100
0
C).
Uma outra escala de temperatura muito importante, principalmente do
ponto de vista de aplicaes cientficas, a escala Kelvin (ou absoluta). Ela
baseada em propriedades microscpicas da matria, como veremos adiante. O
zero desta escala corresponde ao ponto em que toda energia (exceto a de ponto
zero) retirada do sistema. Esta escala ser relacionada com a Celsius atravs
da expresso: T
K
= T
c
+ 273.15.
Uma outra escala que bastante utilizada em alguns pases a escala
Fahrenheit, que se relaciona com a Celsius atravs da expresso: T
F
= 9/5T
c
+
32
13.3 Equao de estado
A equao de estado a relao matemtica existente entre as vrias
grandezas macroscpicas que definem o estado de um sistema. De um modo
geral, o conhecimento da equao de estado permite o conhecimento de todas
as propriedades termodinmicas do sistema.
Para gases com presses muito baixas, as interaes entre as
molculas do sistema podem ser desprezadas. Neste caso, o gs chamado de
ideal e o relacionamento entre as grandezas macroscpicas que definem seu
estado termodinmico dado pela equao de Clapeyron:
PV = NK
B
T
onde T a temperatura absoluta (em K). N o nmero de molculas contidas
no volume V e K
B
a constante de Boltzmann (K
B
= 1.38 x 10
-23
J/K).
A equao acima tambm pode ser escrita em termo do nmero de
moles, n = N/N
A
, onde N
A
= 6.02 x 10
23
o nmero de Avogadro. Neste caso,
PV = nRT
onde R = N
A
K = 8.314 J/mol
0.
K = 0.082 atm.l/mol
0
K chamada de constante
universal dos gases.






Termologia e termodinmica


S. C. Zilio e V. S. Bagnato Mecnica, calor e ondas


250
A equao de Clapeyron s vlida para gases ideais. Entretanto, num
caso real notamos que para T = 0, V no pode ser zero porque as molculas
tm seu prprio volume. Assim, quando T = 0, V = b, que o volume somado
de todas as molculas. Por outro lado, como as molculas possuem interao
atrativa em si, a presso nula mesmo antes de T = 0. Desta forma, para gases
reais devemos usar a equao de van der Waals:
( ) nRT b V
V
a
P
2
=

+
onde a e b so constantes determinadas experimentalmente para cada gs
especfico.
13.4 Interpretao microscpica da temperatura
Vamos considerar um gs contido num reservatrio de volume V que
satisfaz as seguintes hipteses:
(i) o gs constitudo de um nmero grande de partculas que colidem
elasticamente entre si e com as paredes do recipiente;
(ii) no h foras atrativas (aproximao de gs ideal);
(iii) o movimento completamente aleatrio, no havendo direo ou posio
privilegiada.
Sendo o movimento completamente aleatrio (hiptese (iii)), as
velocidade mdias so as mesmas nas direes x, y e z. Logo:
z y x v v v = =
Olhando para uma pequena poro do gs nas proximidades da parede
(Fig. 13.5), podemos imaginar que um grande nmero de partculas colidiro
com esta parede. Num intervalo de tempo t, todas as partculas contidas no
volume Av
x
t e movimentando-se para a direita colidiro com a superfcie.
Como metade das partculas caminham para a esquerda, o nmero de colises
no intervalo de tempo t dada por:






Termologia e termodinmica

S. C. Zilio e V. S. Bagnato Mecnica, calor e ondas


251
A t v
2
1
x
=
onde = N/V o nmero de partculas por unidade de volume.






Fig. 13.5 Molculas colidindo com a parede de um recipiente.
Como as colises com a parede so elsticas, cada partcula transfere
uma quantidade de movimento 2mv
x
em cada coliso, sendo m a massa da
partcula. O momentum total transferido durante o intervalo de tempo t :
A t mv
V
N
mv 2 p
2
x x
= =
Assim, a fora mdia e a presso exercidas sobre a parede so:
2
x
v m A
V
N
t
p
F =

=
2
x
mv
V
N
A
F
P = =
Por outro lado, como o movimento isotrpico, temos:
2 2
x
2
x
2
z
2
y
2
x
2
v
3
1
v v 3 v v v v = = + + =
e, portanto, , mv P
2
V
N
3
1
= que pode ser escrito como:
2
mv
N PV
2
3
2
=
Comparando este resultado com a equao de Clapeyron, temos:
A

v
x
V
x
t






Termologia e termodinmica


S. C. Zilio e V. S. Bagnato Mecnica, calor e ondas


252
2
B
mv
2
1
T K
2
3
=
de onde conclumos que a temperatura est associada energia translacional
das molculas do gs ideal. Esta expresso tambm conhecida como teorema
da equipartio de energia. De uma maneira geral, associamos a cada grau de
liberdade de um sistema, o termo 1/2 KT. No exemplo acima, temos 3 graus
de liberdade, que correspondem a translaes nas direes x, y e z. Se
tivermos vibraes ou rotaes de uma molcula, tambm associamos um
termo 1/2 KT a cada um destes graus de liberdade.
Para vermos como o teorema da equipartio de energia til, vamos
considerar um resistor R sujeito a uma certa temperatura T. Se associarmos
1/2 KT potncia mdia dissipada por ele em 1 segundo teremos:
2
KTR
V KT
2
1
R
V
2
= =
isto , aparece uma pequena voltagem nos terminais do resistor, que
conhecido como rudo Jonhson. Para um resistor de 1 temperatura
ambiente (T ~ 300 K), . nV 4 . 0 V 10 5 . 4 ~ V
10
=

Esta uma voltagem
extremamente pequena, mas em medidas de alta preciso ela deve ser levada
em conta.
13.5 Dilatao trmica
Ao aquecermos um slido, em geral ele muda de tamanho. Isto se
deve ao fato de que a energia potencial entre seus tomos, ou molculas,
possui termos no harmnicos, como mostrado na Fig. 13.6. Ao aumentarmos
a temperatura, damos mais energia ao sistema e os tomos do slido
(idealizados como estando conectados por molas) vibram com grande
amplitude, produzindo em mdia uma separao maior entre os constituintes
do sistema.
A variao do comprimento de um slido segue a lei:







Termologia e termodinmica

S. C. Zilio e V. S. Bagnato Mecnica, calor e ondas


253








Fig. 13.6 Energia de interao entre dois tomos.
T L L =
onde chamado de coeficiente de dilatao linear e caracterstico de cada
material, como mostra a tabela.

Material
o
(C
-1
)
alumnio 23 x 10
-6

ao 11 x 10
-6

invar 0.7 x 10
-6

vidro 9 x 10
-6

pirex 32 x 10
-6

Como cada dimenso do slido varia desta forma, temos:
L
1
= L
1
t L
1
= L
10
+ L
10
T = L
10
(1 + T)
L
2
= L
20
(1 + T) e L
3
= L
30
(1 + T)
Para calcularmos a dilatao superficial de um corpo, fazemos: A = L
1

L
2
= L
10
(1 + T) L
20
(1 + T) = L
10
L
20
+ 2 L
10
L
20
T + L
10
L
20

2
T
2
.
Como muito pequeno, podemos desprezar o tempo
2
e assim temos:
T A T 2 A A
0 0
= =
r
1 r
V(r)
r
2
T
1
T
2






Termologia e termodinmica


S. C. Zilio e V. S. Bagnato Mecnica, calor e ondas


254
onde
20 10 0
L L A e 2 = = . Da mesma forma, temos para a dilatao
volumtrica:
T V V
0
=
onde = 3 chamado de coeficiente de dilatao volumtrica e em geral
dependente da temperatura.
Os lquidos e os gases tambm sofrem, obviamente, variaes de
volume com a temperatura. Neste caso, bastante comum trabalharmos com a
densidade do fluido ao invs do volume:
T V
V
m
V
V
m

V
m
2 2
= = =
Logo, T
0
=
Em geral positivo e a densidade do fluido diminui com a
temperatura. Uma exceo a esta regra o caso da gua (vide a Fig. 13.7) que
abaixo de 4
0
C possui < 0 e assim, entre 4
0
C e 0
0
C a densidade aumenta com
a temperatura. Isto explica porque os lagos congelam-se a partir da superfcie
durante o inverno, no hemisfrio norte.
Fig. 13.7 Variao da densidade da gua com a temperatura.
0 2 4 6 8 10
0.996
0.998
1.000


(
g
/
c
m
3
)
T (
0
C)






Termologia e termodinmica

S. C. Zilio e V. S. Bagnato Mecnica, calor e ondas


255
13.6 Calor e trabalho
Quando colocamos dois corpos com temperaturas diferentes em
contato trmico, existe transferncia de energia de um corpo para o outro at
que as duas temperaturas evoluam para um valor comum. No caso trmico, a
energia transferida tem o nome de calor. Da mesma forma que no trabalho
mecnico, no podemos dizer que um corpo a uma dada temperatura possui
uma certa quantidade de calor. Podemos apenas dizer que calor a variao de
energia entre um ponto e outro.
Analogamente inrcia mecnica, um corpo possui uma certa inrcia
trmica, chamada de capacidade calorfica, que a capacidade que um corpo
tem de reter energia trmica. A definio da capacidade calorfica :
dT
dQ
C =
A unidade de calor J ou erg, mas tambm muito comum o uso da
caloria, que a quantidade de calor necessria para elevar em 1
0
C a
temperatura de 1 g de gua entre 14.5
0
C e 15.5
0
C. O equivalente mecnico da
caloria 1 cal = 4.184 J. A unidade de capacidade calorfica , portanto, cal/

0
C, J/
0
C ou erg/
0
C.
Ao invs da capacidade calorfica, comum utilizar-se o calor
especfico, definido como:

=
C g
cal

m
C
c
0

onde m a massa do sistema. Desta forma, podemos escrever:
dQ = mcdT
significando que ao dar-se uma certa quantidade de calor ao sistema, ocorre
um aumento na temperatura. Esta expresso, entretanto, no sempre vlida.
Em transies de fase, de slido para lquido ou de lquido para gs, a






Termologia e termodinmica


S. C. Zilio e V. S. Bagnato Mecnica, calor e ondas


256
temperatura no muda ao fornecermos calor ao sistema. Para uma certa massa
m de material, o calor fornecido para ocorrer a transio de fase :
Q = mL
onde L chamado de calor latente de fuso ou de vaporizao.
Quando vrios corpos so colocados em contato trmico, o calor flui
de um para outro de tal maneira que:
0 Q
N
1 i
i
=

=

Isto se deve conservao de energia e esta propriedade importante para a
determinao do calor especfico de algum dos corpos. Quando o calor para de
fluir, os corpos esto todos em equilbrio trmico e, neste caso, vale a lei zero
da termodinmica: se um corpo A est em equilbrio trmico simultneo com
os corpos B e C, ento B est em equilbrio com C (Fig. 13.8).





Fig. 13.8 Corpos em contato trmico.
A injeo de calor num sistema pode ser feita atravs da realizao de
trabalho mecnico, dissipao eltrica ou iluminao. A quantidade de energia
(translacional, rotacional e vibracional) contida num sistema chamada de
energia interna. A conservao de energia estabelece que o calor fornecido a
um sistema usado para alterar a energia interna e/ou realizar trabalho
mecnico (W). Este princpio conhecido como 1
a
lei da termodinmica e
pode ser expresso matematicamente como:
Q = U + W
Voltaremos a enfocar este assunto no prximo captulo.
A
B
C






Termologia e termodinmica

S. C. Zilio e V. S. Bagnato Mecnica, calor e ondas


257
13.7 Transmisso de calor
O calor pode ser transmitido de trs formas distintas: pela conduo,
radiao ou convexo. Na conduo, embora seja necessrio um meio material
para haver a transmisso de calor, no existe transporte de massa. O que
acontece que ao aquecer uma parte de um slido, as molculas ali contidas
vibram com amplitudes grandes e transmitem esta vibrao para molculas
vizinhas. No caso dos metais, os eltrons de conduo tambm so
importantes no mecanismo de transmisso de calor.
J no caso da convexo, o calor transportado devido ao movimento
de massa. Ao se aquecer parte de um fluido, variaes de densidade e/ou
presso fazem com que a matria se movimente, transmitindo o calor. Um
exemplo tpico, j citado anteriormente, o congelamento dos lagos a partir da
superfcie, pois a gua fria (abaixo de 4
0
C) menos densa e sobe. Quando o
fluido obrigado a se mover devido ao de algum agente externo como por
exemplo, um ventilador, temos o que se chama de convexo forada.
A terceira maneira pela qual o calor transmitido atravs da
radiao. Neste caso, no necessria a presena de um meio material para
transmitir a energia. Esta transmisso feita por intermdio de ondas
eletromagnticas, do tipo radiao infravermelha, que emitida sempre que
um corpo aquecido.
Neste captulo daremos uma nfase maior transmisso de calor pela
conduo. Vamos considerar uma barra de seco transversal A e
comprimento L, cujas extremidades esto em contato trmico com dois corpos
de temperaturas T
1
e T
2
(T
1
> T
2
), como mostra a Fig. 13.9. Para uma
determinada posio x ao longo da barra, a quantidade de calor por unidade de
tempo (corrente trmica) cruzando a rea A naquela posio depende dos
seguintes fatores:
(i) tipo de material da barra tem materiais que conduzem calor melhor que
outros, como por exemplo o cobre conduz calor melhor que o ao. A
facilidade que um material possui para conduzir calor dada pela
condutividade trmica K.






Termologia e termodinmica


S. C. Zilio e V. S. Bagnato Mecnica, calor e ondas


258
(ii) a rea da barra quanto maior a rea da barra maior ser a corrente
trmica, pois teremos naquele plano mais tomos participando do
processo de conduo.
(iii) o gradiente de temperatura a corrente trmica depender da diferena de
temperaturas entre as camadas de tomos adjacentes ( esquerda e
direita) ao plano caracterizado pela posio x.






Fig. 13.9 Conduo de calor por uma barra isolada lateralmente.
Pelos argumentos expostos acima, podemos escrever a seguinte
expresso para a corrente trmica H:
dx
dT
KA
dt
dQ
H = =
Se a barra for isolada termicamente, como o caso da Fig. 13.9, existe
conservao da corrente, ou seja, todo calor que entra numa extremidade da
barra sair pela outra, pois no existem perdas. Nesta situao, H independe de
x e, conseqentemente, dx / dT constante. Assim,

L
T T
dx
dT 1 2

=
e consequentemente,
L
T T
KA H
2 1

=
Neste caso, a distribuio de temperatura uma reta, como mostra a
Fig. 13.10(a). Por outro lado, se a superfcie lateral da barra no for isolada,
haver perdas de calor por conveco e a corrente trmica diminui conforme x
T
1
T
2
0

L

x
x
H






Termologia e termodinmica

S. C. Zilio e V. S. Bagnato Mecnica, calor e ondas


259
aumenta. Neste caso, dx / dT tambm diminui e, como conseqncia, temos a
distribuio de temperatura mostrada na Fig. 13.10(b).







Fig. 13.10 Distribuio de temperatura ao longo de uma barra isolada (a) e no
isolada (b) lateralmente.
A seguir, vamos analisar dois exemplos onde a equao da
condutividade se aplica. No primeiro exemplo, vamos considerar duas barras
de mesma seco transversal, porm de materiais diferentes e condutividade
diferentes, como mostra a Fig. 13.11. As barras so isoladas termicamente em
suas laterais. Queremos determinar qual a temperatura do ponto de juno.





Fig. 13.11 Barras de materiais diferentes colocadas em srie.
Como as barras esto isoladas, a corrente trmica constante e,
portanto:
2
2
2
1
1
1
L
T T
A K
L
T T
A K H

=

=
de onde obtemos:
2 1 1 2
1 2 1 2 1 2
L K L K
T L K T L K
T
+
+
=
x
0 L
T
2
T
1
T
L
1
L
2
T
1
T
2
< T
1 K
2
K
1
K
1






Termologia e termodinmica


S. C. Zilio e V. S. Bagnato Mecnica, calor e ondas


260
e aps substituir na expresso para H temos:
( )
2 1
2 1 1 2
2 1
T T
L K L K
A K K
H
+
=
e no caso particular em que K
1
= K
2
recuperamos o resultado j conhecido da
barra isolada. A distribuio de temperatura ao longo das barras depende da
razo entre K
1
e K
2
. Se K
1
> K
2
temos a distribuio de temperatura mostrada
na Fig. 13.12.







Fig. 13.12 - Distribuio de temperatura ao longo duas barras de materiais diferentes
colocadas em srie.
Como segundo exemplo, vamos considerar um cilindro oco, de raios a
e b, como mostra a Fig. 13.13. A parte interna do cilindro mantida a uma
temperatura T
1
, enquanto que a externa mantida em T
2
(T
2
< T
1
). O
comprimento do cilindro L e a condutividade K. A corrente trmica
radial. A rea dada por rL 2 A = e, portanto:







Fig. 13.13 Cilindro oco com conduo trmica radial.
T
x
0 L
1
+ L
2
T
2
T
1
L
1
L
a
b






Termologia e termodinmica

S. C. Zilio e V. S. Bagnato Mecnica, calor e ondas


261
dr
dT
rL 2 K
dr
dT
KA H = =
Como H constante, pois no h perdas, podemos integrar esta igualdade para
b r a e no caso em que
2 1
T T T . Como resultado temos:
( )
( )
2 1
T T
a / b n
LK 2
H

=
l

Para finalizar este captulo, vamos abordar rapidamente os outros dois
tipos de transmisso de calor mencionados no incio. Na convexo, estamos
interessados no seguinte tipo de problemas: dado um corpo a uma temperatura
T envolvido pelo ar atmosfrico (mais frio) tal que o corpo est mais quente
T quando comparado com o ar, quanto calor ele perde por unidade de
tempo? A corrente trmica do corpo para o ar tem uma forma similar a da
conduo trmica:
H = hAT
onde A a rea onde est havendo perda de calor e h um nmero que
depende de T (em geral
4
1
T h ), da geometria do corpo e sua orientao
no espao, j que a conveco se deve ao fato que o ar quente sobe. Portanto,
uma placa deitada tem um h diferente ao de uma placa em p.
A transmisso de calor por radiao proporcional a T
4
, como T
sendo a temperatura absoluta (Kelvin):
R = eT
4

Aqui, R a corrente trmica emitida por unidade de rea, e a emissividade
do corpo ( ) e 1 e 0 a constante de Stefan-Boltzmann
(
( )
4
0 2
K m
W
8
10 7 . 5

= ).






Termologia e termodinmica


S. C. Zilio e V. S. Bagnato Mecnica, calor e ondas


262
Exerccios
1- Na perfurao de um bloco de lato (
C g
cal
0
1 . 0 c = ) de 500 g, fornecida
uma potncia de 300 W durante 2 minutos. Qual o aumento de
temperatura do bloco se 75% do calor gerado o aquece? O que acontece
com os 25% restantes?
2- Suponha que o calor especfico de um corpo varia com a temperatura de
acordo com a expresso c = A + BT
2
, onde A e B so constante e T em
0
C.
Compare o valor mdio de c entre T = 0 e T = T
1
, com seu valor em T
1
/2.
3- Considere um corpo slido com momento de inrcia I. Mostre que devido
a uma pequena variao de temperatura T, este momento varia de I =
2T, onde o coeficiente de dilatao linear. Com este resultado,
calcule de quanto varia o perodo de um pndulo fsico sujeito a uma
variao de temperatura T.
4- Mostre que a corrente trmica em uma substncia de condutividade K
situada entre as superfcies de duas esferas concntricas dada por:
( )
1 2
2 1
2 1
r r
r kr 4
T T H
dt
dQ

= =
onde r
1
e r
2
so respectivamente os raios das superfcies interna e externa e
T
1
> T
2
.
5- Um sistema termodinmico levado do estado inicial A a outro B e
depois trazido de volta a A pelo ponto C, como ilustra o diagrama da Fig.
13.14. Calcule o trabalho realizado pelo sistema para efetuar o ciclo
completo.









Termologia e termodinmica

S. C. Zilio e V. S. Bagnato Mecnica, calor e ondas


263







Fig. 13.14
6- Uma barra com coeficiente de dilatao trmica e mdulo de Young Y
( )
L
L
Y
A
F
=
est presa entre duas paredes, conforme mostra a Fig. 13.15.
Calcule a tenso na barra quando a temperatura acrescida de T.






Fig. 13.15
7- Qual a quantidade de calor necessria para transformar 1g de gelo a 10
0
C (c
gelo
= 0.55 cal/g
0
C, L
f
= 80 cal/g) em vapor a 100
0
C (L
V
= 540
cal/g)?
8- Coloca-se uma barra de metal (C = 0,2 cal/g
o
C) a 100
o
C sobre um grande
bloco de gelo a 0
o
C. Qual a massa da barra se quando o sistema atingir
o equilbrio trmico 500 g de gelo se derreteram?
9- Coloca-se um bloco de gelo a 20
0
C dentro de um recipiente
hermeticamente fechado com 200g de vapor de gua a 100
0
C. Se a massa
do gelo 500 g, qual ser a temperatura final do sistema?
P (N/m
2
)
V(m
3
)
A

B

1

2

C

0

20

40

0







Termologia e termodinmica


S. C. Zilio e V. S. Bagnato Mecnica, calor e ondas


264
10- Encontre o gradiente de temperatura e a corrente trmica numa barra de
condutividade K, comprimento L e seco transversal irregular, como
mostra a Fig. 13.16.






Fig. 13.16
12- Duas barras de materiais diferentes, com comprimentos, mdulos de
Young e coeficientes de dilatao trmica dados respectivamente por L
1
,
L
2
, Y
1
, Y
2
,
1
e
2
, esto presas entre duas paredes como mostra a Fig.
13.14. Calcule a distncia percorrida pelo ponto de juno das barras
quando o sistema aquecido de T. Qual a tenso nas barras?






Fig. 13.17

L
L/2
A 2A T
2
T
1
>T
2

L
1 L
2

1
, Y
1

2
, Y
2





Termodinmica do gs ideal

S. C. Zilio e V. S. Bagnato Mecnica, calor e ondas


265






14.1 Introduo
Consideremos um gs ideal contido num cilindro com pisto como
mostrado na Fig. 14.1. Mediante a movimentao de mbolo, possvel
comprimir ou expandir tal gs e neste processo haver variao de presso
e/ou temperatura j que estas variveis esto vinculadas pela equao do gs
ideal (PV = NKT).
Imaginemos que a presso P do gs maior que a presso atmosfrica.
Neste caso, a tendncia do gs empurrar o pisto para fora do cilindro. Se o
pisto desloca-se lentamente uma distncia dx, o trabalho realizado pelo gs
ser:
W = Fdx = P Adx = PdV
onde A a rea do pisto expanso e dV = Adx corresponde variao de
volume durante a expanso.






Fig. 14.1 Gs ideal num cilindro com pisto.
Assim, se imaginarmos o gs expandindo-se de um volume V
1
at um
volume V
2
, o trabalho total realizado :
14
TERMODINMICA
DO GAS IDEAL
P P
a






Termodinmica do gs ideal


S. C. Zilio e V. S. Bagnato Mecnica, calor e ondas


266
V
1
V
2
P
V
V
1
V
2
1
2

=
2
1
V
V
PdV W
Se acompanharmos a evoluo da presso com volume num diagrama PV,
como o da Fig. 14.2, o trabalho realizado pelo gs ser a rea sobre a curva.
Esta rea depende de como o gs levado do ponto 1 ao ponto 2.





Fig. 14.2 Diagrama PV de um gs.
A Fig. 14.3 mostra dois possveis caminhos que podem levar o gs do
ponto inicial ao final. No processo (a), inicialmente o sistema sofre uma
transformao isobrica (presso constante), seguida de uma transformao
isocrica (volume constante). J no processo (b) temos inicialmente uma
transformao isocrica seguida de uma isobrica.






Fig. 14.3 Trabalho realizado em dois processos diferentes.
Analisando as reas sob as curvas (a) e (b), vemos que no primeiro
caso, o trabalho realizado P
2
(V
2
- V
1
) enquanto que no segundo, o trabalho
P
1
(V
2
V
1
). Embora os processos sejam diferentes, eles produzem a mesma
variao da energia interna do gs U = U
2
U
1
, j que esta s depende dos
estados inicial e final do sistema. O trabalho realizado diferente em cada
processo e portanto, pela 1
a
lei da termodinmica, o calor transferido tambm
o ser.
1

2

P
V
P
2
P
1
(a)
(b)





Termodinmica do gs ideal

S. C. Zilio e V. S. Bagnato Mecnica, calor e ondas


267
P
V
1
2
Vamos agora imaginar um processo (Fig. 14.4) no qual o sistema
conduzido de um estado 1 at um estado 2 atravs de um caminho A e trazido
de volta a 1 por um outro caminho B, fechando desta forma o ciclo de
transformao.




Fig. 14.4 Trabalho realizado num ciclo completo do gs.
Durante o ciclo completo, a variao total da energia interna nula e
portanto
W = Q
de modo que todo o calor absorvido pelo gs foi usado na realizao de
trabalho, no contribuindo em nada para a alterao da energia interna do
sistema.
A energia interna de um gs ideal, como j discutimos anteriormente,
provm essencialmente da energia cintica de seus constituintes, de modo que:
T NK
2
3
U
B
=
e assim, nos processos onde no h alterao de temperatura, U = 0. Esta
relao foi verificada experimentalmente por Joule, atravs de uma expanso
adiabtica (sem troca de calor) do gs, como mostrado na Fig. 14.5. Um gs
isolado termicamente expande no vcuo, mudando seu volume de V
1
para V
1
+
V
2
, aps a remoo da partio mostrada. Embora haja variao de volume, o
trabalho realizado pelo gs nulo, pois a presso da interface nula. Por outro
lado, devido ao fato da parede ser isolante, Q = 0 e pela 1
a
lei da
termodinmica U = 0. Assim, se U = U(T), a no variao da energia interna
implica na no variao da temperatura. De fato, Joule observou que a
temperatura indicada pelo termmetro T acoplado ao sistema no muda
durante o processo.






Termodinmica do gs ideal


S. C. Zilio e V. S. Bagnato Mecnica, calor e ondas


268





\

Fig. 14.5 Expanso livre de um gs.
14.2 Capacidade trmica
Como o gs ideal pode sofrer transformaes tanto isocricas quanto
isobricas, definimos dois tipos de capacidade trmica:
(i) volume constante: C
v
= (dQ/dT)
v
. Como V constante, dV = 0 e dW = 0.
Logo: C
v
= dU/dT.
(ii) presso constante:
( ) ( )
p
V
p
p
p
dT
dW
C
dT
dW
dT
dU
dT
dQ
C + = + =

=
. Para um
gs ideal, PV = nRT e, portanto, PdV + VdP = nRdT. Como estamos tratando
de um processo onde a presso constante, dP = 0. Logo, dW = PdV = nRdT
e consequentemente,
C
p
= C
V
+ nR
14.3 Tipos de expanses
(a) expanso quase esttica - O processo denominado de quase esttico
quando executado lentamente, de modo que a variao das grandezas
termodinmicas infinitesimalmente pequena entre dois passos consecutivos
do processo. Obviamente, o experimento de Joule discutido anteriormente no
uma expanso quase-esttica.
(b) expanso isotrmica - Neste caso, a temperatura mantida constante.
Sabemos que para o gs ideal PV = nRT, logo P = (nRT)(1/V) e esta curva no
gs
vcuo
V
1
V
2

termmetro
isolante





Termodinmica do gs ideal

S. C. Zilio e V. S. Bagnato Mecnica, calor e ondas


269
diagrama PV da Fig. 14.6 chamada de isoterma.







Fig. 14.6 Expanso isotrmica.
Na isoterma de um gs ideal (PV = nRT) temos: PdV + VdP = nRdT
= 0 dW = PdV = - VdP. Entretanto,
V
dV
nRT dW dV
V
nRT
dP
V
nRT
P
2
= = =
e assim, o trabalho realizado quando o gs vai de V
1
a V
2
:
( )
1 2
V
V
2 1
V / V n nRT
V
dV
nRT W
2
1
l = =


(c) expanso adiabtica - Nesta transformao o sistema encontra-se
termicamente isolado tal que Q = 0. Neste caso, h variao da temperatura do
gs, como pode ser visto pela Fig. 14.7, de modo que o processo inicia-se
numa isoterma T
1
e termina em outra isoterma T
2
.





Fig. 14.7 - Expanso adiabtica.
Queremos calcular qual a relao entre P e V durante o processo

(P
2
,V
2
)
(P
1
,V
1
)
expanso adiabtica
P
v
(P
2
,V
2
)
(P
1
,V
1
)
P1/V (isoterma)
P
v






Termodinmica do gs ideal


S. C. Zilio e V. S. Bagnato Mecnica, calor e ondas


270
adiabtico. Como dQ = 0, dU = C
V
dT e dW = PdV, a 1
a
lei da termodinmica
pode ser escrita como: 0 = C
V
dT + PdV, e portanto, PdV = -C
V
dT.
Da lei dos gases ideais temos que PdV + VdP = nRdT. Multiplicando
os dois lados por Cv obtemos:
Cv(PdV + VdP) = nR(Cv dT) = -nRPdV
PdV(C
V
+ nR) = -Cv VdP C
p
PdV = -Cv VdP
Definindo = C
p
/C
V
temos: P / dP V / dV = . Integrando, obremos:


2
1
1
2
1
2
1
2
P
P
n
V
V
n
P
P
n
V
V
n l l l l

= =

2 2 1 1
2
1
1
2
V P V P
P
P
V
V

isto , num processo adiabtico PV

= constante. Como para o gs ideal PV =


nRT, obtemos uma expresso alternativa:
TV
-1
= constante
Para o gs ideal , dT C dV e nRdT dU nRT U
V
2
3
2
3
= = = logo
. nR C
2
3
V
= Por outro lado nR C nR C C
2
5
p V p
= + = e, portanto,
3
5
V
P
C
C
= = para o gs ideal.
14.4 Mtodo de Rchhardt para determinao de
O mtodo de Rchhardt mostrado na Fig. 14.8, permite a medida da
razo
V P
C / C = de um gs. Consideremos um gs contido num recipiente
grande, de volume V. Conectado a este recipiente existe um tubo, cuja rea da
seco transversal A, no qual uma bola de metal de massa m (que se encaixa
perfeitamente no tubo) pode deslizar formando uma sistema equivalente a um
pisto. Devido compresso e descompresso do gs, esta massa oscila em
torno de sua posio de equilbrio (y = 0). A presena da esfera metlica faz





Termodinmica do gs ideal

S. C. Zilio e V. S. Bagnato Mecnica, calor e ondas


271
com que a presso interna seja P = P
a
+ mg/A na posio de equilbrio, onde
P
a
a presso externa (atmosfrica). Chamaremos de o perodo de oscilao.









Fig. 14.8 - Mtodo de Rchhardt para determinao de .
Dando um deslocamento y na esfera, a variao do volume do gs
V = yA, de forma a haver uma variao de presso P acompanhando V.
Este incremento na presso produz uma fora restauradora F = PA. Supondo
que o processo seja quase esttico e adiabtico temos:
0 P V V PV const. PV
1
= + =


Usando P = F/A e V = yA, obtemos:
0 A / F V yA PV
1
= +


de onde tiramos: F =
y
V
PA
2

, que nos leva seguinte equao de


movimento:
0 y
V
PA
dt
y d
m
2
2
2
=

+
que a equao diferencial de um movimento harmnico simples de
frequncia:
mV
PA
2
2
0

=
y = 0

y > 0

A
mg
P P
0
+ =
y < 0

V

A







Termodinmica do gs ideal


S. C. Zilio e V. S. Bagnato Mecnica, calor e ondas


272
e perodo:
2
PA
mV
2

=
Desta forma, conhecendo-se P, V, m e A, podemos medir e obter como
sendo
2 2
2
P A
mV 4

=
Exerccios
1- Um gs ideal, inicialmente com presso P
1
e volume V
1
, expande-se
adiabaticamente at a presso P
2
e volume V
2
. Mostre que o trabalho
realizado W = (P
1
V
1
P
2
V
2
)/( - 1) onde = C
p
/C
v

2- Calcule o rendimento (
1
Q / W = , Q
1
= calor recebido pelo sistema) do
ciclo do Otto (Fig. 14.9).
3- Calcule o rendimento do ciclo de Carnot (Fig. 14.10).
4-







Fig. 14.9 Fig. 14.10
5- Explique porque o calor especfico a volume constante menor que o calor
especfico a presso constante.
6- Calcule o coeficiente de dilatao volumtrica de um gs ideal presso
constante.
V
2

P
V
V
1

adiabticas
T
2
adiabtica
P
V
isoterma
T
1





Termodinmica do gs ideal

S. C. Zilio e V. S. Bagnato Mecnica, calor e ondas


273
7- Calcule o coeficiente de dilatao volumtrica de um gs ideal durante uma
expanso adiabtca.
8- Calcule a compressibilidade ( )
T dP
dV
V
1
K = de um gs ideal.
9- A velocidade mdia das molculas de um gs ideal 500 m/s. Se o gs
mantiver a mesma temperatura e as massas moleculares forem duplicadas,
qual ser a nova velocidade mdia?
10- Mostre que para um gs diatmico
5
7
C
C
V
p
= =
11- Calcule o trabalho realizado na expanso isotrmica de V
1
a V
2
de um gs
real.
12- Considere o ciclo trmico mostrado na Fig. 14.11. a) Calcule o calor
fornecido ao sistema na isoterma 1 2, b) Calcule o trabalho na isobrica
2 3 e c) Calcule o calor fornecido ao sistema na isocrica 3 1








Fig. 14.11

isotrma
V
2

V
1

(isoterma)
P
v
P
1

P
2







Respostas dos exerccios


S. C. Zilio e V. S. Bagnato Mecnica, calor e ondas


275






Captulo 1

1.1 5 2 , k

5 j

4 i

3 r + + =
r

1.5 a) 4 b . a =
r
r
, k

7 j

b a + + = +
r
r
, k

5 i

3 b a + =
r
r
,
k

10 i

17 b a =
r
r

b) k

6
j

4
i

a
7 7 7
2
//
+ =
r
, c) k

a
7
22
7
25
7
16
+ + =

r

1.6 b) 29 , cos = 4/ 29 , tg = 3/2, c) ( ) k

3 a
//
+ + =
r
, d)
k

a + =

r

1.8 0 b . a =
r
r
, k

8 j

4 i

16 b a =
r
r

1.9 k

6
j

4
i

a
7 7 7
2
//
+ =
r
, k

a
7
22
7
25
7
16
+ + =

r

1.10
) j

sen i

(cos r r + =
r

) j

cos i

sen ( v v
0
+ =
r

k

rv r v
0
=
r r

1.11 0 ) cos sen cos sen ( v r r . v
0
= + =
r r

15
RESPOSTAS DOS
EXERCCIOS

r
r
x
y
v
r






Respostas dos exerccios


S. C. Zilio e V. S. Bagnato Mecnica, calor e ondas


276
1.12 a) 6x, b)
3 2
x
x sen 2
x
x cos
, c) ) x x 4 x 2 1 ( e
3 2 x
+ + +
d)
( )
2
2
2 4
3 x
x 6 x 6 x
+
+

1.13 a) 0, b)
2
1

, c) 1, d) 1/16
1.14 a) 1/x, b) lnx
1.15 a) 20 x
4
, b) 6x
2
+8x-5, c) cosx senx, d) 2x , e) senx + xcosx
f) 2/x
3
, g)
( )
( )
2
2
2
1 x
1 x
2
+

, h) (1+x) e
x
, i) x sec cos y
2
= & ,
j)
x 2
1
y = & , k)
2
3
x 2
1
y = &
1.16 a) sec
2
x, b) a, c) 2, d) n x
n-1
senx, e) senx cos(cosx), f) 1
1.17 /4
1.18 a) 3x
2
/2 + C, b) 7 x
3
/3 + x
4
2x + C, c) 15/11 x
11
+ 8/3 x
3
+ C
1.19 a) 6, b) 34/3, c) (e
2
-1)/2, d) 1/4
1.20 a) 4 / 1 x
o
= , b) 8 / 25 y
o
= , c) ( ) 8 / 25 4 / 1 x 2 y
2
+ = ,
d) 1, -3/2, f) 19/6
Captulo 2
2.1 Demonstrao
2.2 Em relao ao solo: y
1
= 0, y
2
= 5/9 h, y
3
= 8/9 h, y
4
= h, com h = 2m
2.3 a) x
max
= (4/27) a
3
/b
2
em t = 2a/3b, b) v(t) = 2at-3bt
2
, v = 0 para t =
2a/3b, c) a = 2a 6bt, a = 0 para t = a/3b






Respostas dos exerccios


S. C. Zilio e V. S. Bagnato Mecnica, calor e ondas


277
2.4
0
3 / 2
0
v
A 2
v 3
5
3
d
|

\
|
=
2.5 a) Ver figura ao lado
b) t
C
= 4 s, c) 8 m
d) v
B
= 4 m/s
e) 2 s


Captulo 3
3.1 = (a/R) , = (a/2R) t
2

3.2 | | t ) ( cos r r 2 r r v v
a b b a b a
2
a
2
a
2
b
2
b
2
a b
+ =
r r

3.3
( )
2 2 2
R 4 h
h g R 2
a
+

,
( )
2 2 2
2
z
R 4 h
h g
a
+
=
,
( )
( )
2
2 2 2
2
r
R 4 h
R / t h g 2
a
+

=

3.4 ( )
|
|

\
|
+ + =
2
0
h
d
1 1 gh v
3.5 Para R > v
0
2
/g
3.6

= g cot
g 2
sen v
x
2 2
0
max
,
g 2
sen v
z
2 2
0
max

= ,

=
sen g
sen v 2
t
0

3.7 a) sen = g/20 30
0
, b) y
max
= g/2 5 m, c) R = 2 v
0
cos
20 3 m, d) 60
0

3.8
x 0
2
x 0
y 0 x 0
v
g 2
v
H
g
2
g
v v
d |

\
|
+ + = ,
g 2
v
H y
2
y 0
max
+ =
3.9 a) t
col
= 2 s, b) H = 85 m, c) x
col
= 100/ 3 m, y
col
= 80 m
0 1 2 3 4 5
0
2
4
6
8
10
P
o
s
i

o

(
m
)

Tempo (s)
A
B






Respostas dos exerccios


S. C. Zilio e V. S. Bagnato Mecnica, calor e ondas


278
3.10 H =
2
0
g
2 sen v
2
g
|

\
|


3.11 b) |

\
|

=
g 2
2 sen v
x
cos v
g
tg
2
0
2 2
0
, c)
g 2
2 sen v
x
2
0
max

= , d)
g
2 sen v
R
2
0

=
Captulo 4
4.1 = 30
0
4.2
3
1
M
M
2
1
=

4.3 2g/15 (para cima)
4.4 M
A
= 4 Kg
4.5
g
M 4 M
M 2 M
2 a
1 2
1 2
A
|

\
|
+

=
,
g
M 4 M
M 2 M
a
1 2
1 2
B
|

\
|
+

=
,
( )
g
M 4 M
2 M M
T
1 2
1 2
+
+
=

4.6 a
R
= g tg. Neste caso, N = mg/cos
4.7
g
sen cos
sen cos
a
max

+
=
,
g
sen cos
sen cos
a
min
+
+
=

4.8 a) a = (2x/L-1) g, b) x = L/2
4.9 a) F = 3Mg, b)
|

\
|

+
=
e
e
max
1
1
Mg 3 F
,
|

\
|
+

=
e
e
min
1
1
Mg 3 F

4.10
b 3
mv 2
d
2
3
0
=
4.11 a) a = 2/5 g, b) T = 3/5 g






Respostas dos exerccios


S. C. Zilio e V. S. Bagnato Mecnica, calor e ondas


279
4.12
g
M M M M M M 4
M M 2
a
3 2 3 1 2 1
3 2
1
+ +
=
,
g
M M M M M M 4
M M 2
a
3 2 3 1 2 1
3 1
2
+ +
=
,
( )
g
M M M M M M 4
M M M
a
3 2 3 1 2 1
3 2 1
3
+ +
+
=
,
g
M M M M M M 4
M M M 2
T
3 2 3 1 2 1
3 2 1
+ +
=

4.13 a) tg = F/3Mg, b)
( ) ( )
2
2
Mg
3
F
T + =
,
c)
Mg 3
F
Mg 3
F
1
2
+
|

\
|
+ =

4.14
F
n
1 i n
Ti
+
=

4.15 a)
(
(

\
|
+ = 1
v
gL 2
1
gL 2
v
cos
2
2
2
, b) T = Mg/cos
4.16 a = g cotg. Neste caso, T = mg/sen
4.17 a) tg
0
=
e
, b)
2
e
e
min
1
Mg
F
+

=

4.18 a) ( ) ( ) g M M F
2 1 c e max
+ + = , b)
|

\
|
+ =
2
1
c
2
2
M
M
2 1 g
M
F
a
, a
1
=

c
g
4.19 a) F = 2Mg cos, b) a = g(cos - sen)
4.20 a)
|

\
|

+
+
=
tg 1
tg
mg
M
M m
F , b)

=
sen cos
mg
N
Captulo 5
5.1 Demonstrao

5.2 W = (F-2Mg) x








Respostas dos exerccios


S. C. Zilio e V. S. Bagnato Mecnica, calor e ondas


280
5.3





b) 0
dr
dU
= para r = r
0
, c)
|

\
|
= =
7
6
0
13
12
0
r
r
r
r
C 12
dr
dU
) r ( F ,
d) E = C
5.4 d = 4/5 l

5.5 a) gR 5 v
0
= , b) = 20.3
0
5.6





b) x sen
dx
dU
) x ( F = =
c) v
0
=2/ M

5.7 a) K = mg(h-2R) = 1/2 mv
P
2
,
m
N
g
R
v
a
2
p
C
+ = = , c) N = 0 h =
5/2R, d) ( ) = cos 3 2
R
h 2
mg ) ( N
5.8 a) 1 m, b) 0.79 m, c) H = L sen 30
0
= 1.72 m
5.9
2
v
gh 2
1 d ' d =
5.10
|

\
|
+
=
g cot 1
1
g 2
v
H
2
0

5.11 gR 5 v
0
=
a)
20
-0.8
0.0
0.8
a) U(r)
r
0
-1
x
U(x)
1
1






Respostas dos exerccios


S. C. Zilio e V. S. Bagnato Mecnica, calor e
ondas


281
5.12 a) ) cos 1 ( gR 2 v ) ( v
2
0
+ = , b)
R
mv
) 2 cos 3 ( mg ) ( N
2
0
=
,
c)
gR 3
v
3
2
cos
2
0
+ =

5.13 a) mgy
2
ky
) y ( U
2
= , b)
k 2
g m
k
mg
y
2
k
) y ( U
2 2
2

\
|
= , c)
k
mg
y
eq
= , d)
k
mg 2
y
max
= , e)
k
m
g v
max
=
5.14 a) ) cos 1 ( gL 2 v ) ( v
2
0
= , b)
) cos 3 2 ( mg
L
mv
) ( T
2
0
=
,
c) ( ) gL 5 v
min 0
=
5.15 a) mgx kx
2
1
k 2
g m
) x ( E
2
2 2
K
= ,
b) ( )
2
2 2
2
K
1
k 2
g m
k
mg
x
2
k
) x ( E + +
|

\
|

+ = , c)
k 2
mg
x
max
=
d) 3/4
5.16 a) E
i
= 2mgR + 1/2 kR
2
,
b)

E() = 1/2 mv
2
+ mgR (1+cos) + 1/2 kR
2

c) v
2
() = 2gR (1-cos), d) N() = kR + mg (3cos2)
d) k = 5mg/R
Captulo 6
6.1
( )
+

+
=
2
sen
m
M
cos
m
M
1
gh 2
V

6.2 t = 1.48 s
6.3 v = 57.14 cm/s






Respostas dos exerccios


S. C. Zilio e V. S. Bagnato Mecnica, calor e
ondas


282
6.4


6.5
(

\
|

=
2
2
0
Mg
v
g 2
1
h
6.6 ( ) t Kg tgh
K
g
v =
6.7 v = 61.6 m/s
Captulo 7
7.1
( )
2 1
2 2
2 1
2 1 1
f 1
m m
v m 2
m m
m m v
v
+
+
+

=
( )
2 1
1 2 2
2 1
1 1
f 2
m m
m m v
m m
v m 2
v
+

+
+
=

7.2
B
= 90
0
e tg
A
= , E
K
= E
Ki
(3-5m
B
/m
A
)
7.3
( )
gL 2
v
M m
m
1 cos
2
2
2
+
=
7.4 a)
g M
v m
32
9
h
2
2 2
= , b) ( )
M
m 9
15
16
1
E
E
i
=


7.5 a)
M m
MV mv
V
0
CM
+
+
= ,
b) ( )
0 f 1
v V
M m
mM
p
+
= , ( ) V v
M m
mM
p
0 f 2

+
=
c)
( )
m M
m M v MV 2
v
0
f 1
+

= ,
( )
m M
m M V mv 2
v
0
f 2
+

=
7.6 a) v
f
= 3 m/s, b) E = -9 J
t
F(t)
mg






Respostas dos exerccios


S. C. Zilio e V. S. Bagnato Mecnica, calor e
ondas


283
7.7 a) v
0
= 2 x
m
k
, b) I = x km
2
3
, c)
4
3
E
E
i
=


7.8 Demonstrao
7.9 a) V
CM
= v
0
, b) ( )
0
v
M 2
m 3
1 V + = , d)
i Kr
E
M 4
m 9
E = ,

e)
0 CM
v
M m
M m
2
3
V
+
+
= , c)
7.10
( )
x
m m m
k m
V v
2 1 1
2
CM 1
+
= ,
( )
x
m m m
k m
V v
2 1 2
1
CM 2
+
+ =
7.11
( )
( )
2 1
2 1
2 1
max
v v
m m k
m m
x
+
=


3/2 v
0

3m/2M v
0

You might also like